Anda di halaman 1dari 188

BUCUR

dr. Andrie Ronggani, Sp. OG



Ceritakan kasus pasien / ceritakan pasien dari datang – tatalaksana, sikap
sikap terhadap pasien kamu apa?
baca resume dan sikap kenapa
diagnosis itu? tatalaksananya apa?

1. kenapa kamu yakin usia gestasi-nya 42 minggu? [pasien yakin akan HPHT, siklus haid 28 hari, tidak
minum pil KB selama 3 bulan terakhir] 


2. kelompok RESTI pada ibu hamil menurut MDGs? [Usia (16-35 tahun), grande multipara, riwayat
persalinan, penyakit infeksi, penyakit penyerta (DM, HT), dll] 

Ñ• mene Kan V. cava
3. posisi terbaik pada ibu hamil dan kenapa? [uterus
membesar dan menekan aorta abdominalis di sebelah 0
kanan vertebrae, jadi kalo terlentang dan miring kanan
akan kena] Anjuran tirah baring miring ke kiri (30
derajat) untuk mencegah hypotensive syndrome

4. 7 cardinal movement biasa VS sungsang


5. Ekstensi
✓ 1. Engagement biparietal
Do :

6
Putarp hear
Engagement & Descent Desensus
.

2.
.

3. Fleksi 7 .

Ekspulsi
4 . Putar p. da 1am
Bokong akan memasuki panggul dengan diameter
bitrokanter dalam posisi 0
☐ oblik. Pinggul janin bagian
anterior mengalami penurunan lebih cepat dibanding
pinggul posterior à pinggul depan akan mencapai pintu
tengah panggul terlebih dahulu
normal =
kepaia masukpanggul diameter biparietal
,

Putar Paksi dalam

Tahanan dinding panggul dan kekuatan yang mendorong ke bawah (kaudal) menghasilkan putaran
paksi dalam 450 à sakrum menjadi arah transversal (pukul 3 atau 9), diameter bitrokanter menjadi
AP di pintu bawah panggul
normal di spina ischiadika , kepalamenjddiarah anterior /posterior he arahsimfisis
=

Putar paksi luar

Pinggul bawah menekan perineum. Lalu lahirlah bokong dengan posisi diameter bitrokanter AP, diikuti
putaran paksi luar. Putaran paksi luar akan membuat posisi diameter bitrokanter dari AP menjadi
transversal. Punggung mengarah ke anterior, bahu salah satu diameter oblik pelvis. Bahu turun
dengan cepat dan mengalami putar paksi dalam, dengan diameter bisakromial di sisi AP. Setelah bahu,
kepala defleksi tajam secara normal kea rah thorax, memasuki pelvis pada salah satu diameter oblik,
berotasi sehingga posterior leher di bawah simfisis pubis. Kepala kemudian lahir dalam posisi fleksi.

5. kenapa hamil edemanya di kaki dulu?

Peningkatan fisiologis cairan selama kehamilan akibat turunnya osmolaritas dari 10 mosm/kg yang
diinduksi oleh makin rendahnya ambang rasa haus dan sekresi vasopressin. Saat aterm +3,5 L cairan
berasal dari janin, plasenta, dan cairan amnion, sedangkan 3 L lainnya berasal dari akumulasi
peningkatan volume darah ibu, uterus, dan payudara sehingga minimal tambahan cairan selama
kehamilan adalah 6,5 L. penambahan tekanan vena di bagian bawah uterus dan mengakibatkan oklusi
parsial vena kava yang bermanifestasi pada adanya pitting edema di kaki dan tungkai terutama pada
akhir kehamilan. Penurunan tekanan osmotic koloid di interstitial juga akan menyebabkan edema
pada akhir kehamilan. Mfi-hidira-bumil-B.tv osmolarity bambang halls
g- sekresivasopresin
6. Indikasi VT obstetrik dan ginekologik? & 3,5L ,

janin uterus menekan v. cava


-


-

Indikasi VT obstetric:
-

placenta
-

payudara &
-

amnion -9 V01 darah


edema
-

-0-0
OBS : 1. Untuk menentukan fase persalinan dan diagnosis letak janin
-0
i. fasepersalinan 2. Pada saat ketuban pecah untuk menentukan ada tidaknya prolapse bagian kecil janin atau tali

#
pusat
,

Utakjanin 3. Pada primigravida gravid > 37 minggu untuk mengevaluasi kapasitas panggul dan menentukan
2. prolapse -@
tali pusat adanya kelainan pada jalan lahir yang dapat mengganggu jalannya proses persalinan
pervaginam
3. primi →
Kap 4. Saat inpartu digunakan untuk menilai apakah kemajuan proses persalinan sesuai dengan yang
.

② ② ②
panggue diharapkan
4.
klmajuan
pergalinah
Indikasi VT ginekologik:

1.
2.
Pemeriksaan bentuk, arah, besar, dan konsistensi uterus
Pemeriksaan adneksa dan parametrium
6YNI.letak.konsislensi.dk uterus
2. Adneksa , parametrium
3. Pemeriksaan ballottement 3. lnfeksi
4. Konfirmasi kehamilan intra atau ekstrauterin 4. Ballolemen
5. Konfirmasi peradangan atau infeksi 5. Tumor fluor albus PUA
, ,

6. Pemeriksaan fluor albus, perdarahan, tumor pelvik 
 6. Kehamilan intra


.
eksfraultrin

PLASENTA
pelepasan
7. mekanisme pelepasan plasenta ✗ pengeluaran
v Pelepasan Plasenta: Ukuran uterus akan menyusut disertai pengurangan bidang tempat
implantasi plasenta
à plasenta akan menebal dan menekuk à tegangan pada plasenta
akan menyebabkan lapisan desidua yang paling lemah à terjadi pelepasan plasenta dan
ukuran tempat implantasi di bawahnya mengecil. Saat terjadi pelepasan plasenta terbentuk
hematoma diantara plasenta yang terpisah dan desidua yg tersisa.
v Ekstruksi Plasenta: setelah plasenta terpisah dari tempat implantasinya, tekanan oleh dinding
uterus menyebabkan plasenta menggelincir turun menuju ke segmen bawah uterus atau
bagian atas vagina.
v Mekanisme ekstruksi plasenta: pelepasan plasenta dari dinding uterus ini dapat dimulai dari
1) Tengah (sentral menurut Schultze) à paling banyak
2) Pinggir (marginal Mathew – Duncan)
3) Kombinasi 1 dan 2

Bila terjadi pemisahan plasenta tipe sentral, atau tipe biasa, hematoma retroplasenta à mengalami
inversi dan dibebani oleh hematoma à mendorong plasenta menuju ke rongga uterus, pertama
bagian tengah dan kemudian sisanya. plasenta tersebut, kemudian turun. Karena membran di
sekitarnya menempel kaku pada desidua, plasenta hanya dapat turun dengan menyeret membran
secara perlahan-lahan; kemudian membran-membran tersebut mengelupas bagian perifernya.
Akibatnya, kantong yang terbentuk oleh membran tersebut mengalami inversi, dan yang muncul di
vulva adalah amnion yang mengilap di atas permukaan plasenta atau ditemukan di dalam kantong
inversi. Pada proses ini yang dikenal sebagai ekspulsi plasenta secara mekanisme Schultze, darah dari
tempat plasenta tercurah ke dalam kantong inversi tersebut dan tidak mengalir keluar sampai setelah
ekstrusi plasenta. Mekanisme Duncan: pemisahan plasenta pertama kali terjadi di perifer, dengan
akibat darah mengumpul di antara membran dinding uterus dan keluar dari plasenta. Plasenta turun
ke vagina secara menyamping, dan permukaan ibu adalah yang pertama kali terlihat di vulva.

8. maneuver pelepasan plasenta?

Perasat Kustner

Tali pusat diregangkan dengan satu tangan, tangan lainnya menekan perut di atas simfisis. Kalau tali
pusat masuk, menandakan plasenta belum lepas, kalau tetap atau keluar berarti plasenta sudah lepas
(Tanda biasanya terlihat 5-10 menit setelah bayi lahir).
Bila diyakini plasenta telah lepas, tentukan
apakah kontraksi uterus baik. Penderita disuruh bernapas panjang, dan dengan tekanan pada fundus
uteri, uterus yang berkontraksi baik akan mendorong plasenta ke bawah dan keluar. Tekanan pada
fundus uteri hanya boleh dilakukan setelah kontraksi-uterus baik agar tidak menimbulkan inversio
uteri. Jangan meremas-remas uterus, karena tindakan ini akan menimbulkan kontraksi uterus yang
tidak merata dan akan mengganggu pelepasan plasenta.

Cara melahirkan plasenta secara Calkin

1) Letakkan tangan di atas fundus uteri segera setelah bayi lahir, untuk mengetahui tanda
lepasnya 
plasenta 


2) Lakukan masase uterus dengan segera setelah ada tanda lepasnya plasenta. Cara masase
dengan 
menempatkan jari-jari di belakang uterus sedang ibu jari di depan. 


3) Bila bentuk uterus berubah dari memanjang menjadi bulat dan disertai pengeluaran darah dari
vagina 
 masase sampai kontraksi uterus baik, kemudian tekan fundus uteri ke bawah, dengan
demikian plasenta akan lahir dengan mudah. 


9. tanda-tanda pelepasan plasenta?

- Perubahan bentuk dan tinggi uterus : Fundus uteri naik karena plasenta yang sudah lepas
masuk ke segmen bawah rahim dan vagina sehingga fundus terdorong ke atas. Uterus menjadi
membulat dan keras.
- Tali pusat memanjang 

- Sudden gush of blood 


ANATOMI
so
10. ligament penampang uterus dan masing-masing kegunaannya [7 biji] origo, insersio

- Lig. Cardinal (mackendrot) sinistra et dextra : terpenting , untuk mencegah uterus tidak turun,
mengikat serviks dan vagina ke lateral dinding pelvis 

- Lig. Sakrouterina sin et dext : menahan uterus tidak banyak bergerak. Dari belakang serviks
kiri dan kanan melalui dinding rectum ke os sacrum kiri kanan 

- Lig. Rotundum sin et dex (round ligament) : menahan uterus dalam posisi antefleksi.
Mengikat dari sudut fundus uteri kiri kanan ke inguinal kiri kanan 


97¥
- Lig latum sin et dext (broad ligament) : ligamentum yang meliputi tuba. Berjalan dari uterus
ke arah lateral 

- Lig. Infundibulopelvikum/ lig ovarii proprium : menahan tuba fallopi, dari infundibulum ke
dinding pelvis 

- Lig. ovarii proprium sin et dext: menahan ovarium. Pada sudut kiri dan kanan belakang fundus
uteri 



11. anatomi panggul, penopang panggul dan organ, kelenjar kewanitaan? (Bartholine dll)

Jalan lahir terdiri dari:

- Jalan lahir bagian tulang: tulang-tulang panggul (os coxae 3 bagian: os ilium, os ischium, os
pubis; os sacrum; os koksigis) dan sendi-sendinya (4 sendi panggul: 2 artikulasio sakroiliaka,
simfisis pubis, dan artikulasio sakrokoksigeal)
- Jalan lahir bagian lunak: otot2, jaringan, ligamen2 (ligament pada artikulasio sakroiliaka,
ligament sakrotuberosum: sacrum-tuber iskhii, lig. Sakrospinosum: sacrum-spina iskhiadika)

Panggul secara fungsional dibagi 2:

1. Pelvis mayor pelvis diatas linea terminalis 


2. Pelvis minor dibatasi PAP, PBP
PAP: bidang yang dibatasi di sebelah posterior oleh promontorium,
di lateral oleh linea terminalis dan di anterior oleh pinggir atas simfisis

• Diameter anteroposterior/ konyugata obstetrika: dari promontoriurn sampai ke tengah


permukaan posterior simfisis
• Konyugata diagonalis: jarak bagian bawah simfisis sampai ke promontorium. Pada panggul
normal promontorium tidak teraba dengan jari yang panjangnya 12 cm.
• Konyugata vera: jarak pinggir atas simfisis dengan promontorium diperoleh dengan
mengurangi konyugata diagonalis dengan 1,5 cm.
• Diameter transversa: jarak terjauh garis lintang pintu atas panggul, biasanya sekitar 12,5 - 13
cm.
• Diameter oblikua: persilangan konyugata transversa ke artikulasio sakroiliaka, yang
panjangnya sekitar 13 cm.

Ruang panggul = saluran di antara pintu atas panggul dan pintu bawah panggul. Dinding anterior
sekitar 4 cm terdiri atas os pubis dengan simfisisnya. Dinding posterior dibentuk oleh os sakrum dan
os koksigis, sepanjang ± 12 cm. Ruang panggul berbentuk saluran dengan sumbu melengkung ke
depan. Sumbu ini adalah garis yang menghubungkan titik temu konyugata vera dengan diameter
transversa di pintu atas panggul dengan titik-titik sejenis di Hodge II, III, dan IV. Arah sumbu ini sesuai
pula dengan arah tarikan cunam atau vakum pada persalinan dengan tindakan

Pintu bawah panggul = batas atas pintu bawah panggul setinggi spina iskhiadika. Jarak antara kedua
spina ini disebut diameter bispinosum adalah sekitar 9.5 - 10 cm. Batas bawah pintu bawah panggul
berbentuk segi empat panjang, di sebelah anterior dibatasi oleh arkus pubis, di lateral oleh tuber iskhii,
dan di posterior oleh os koksigis dan ligamen sakrotuberosum. Pada panggul normal besar sudut
(arkus pubis) ±90 ̊. Jika kurang dari 90 ̊, lahirnya kepala janin lebih sulit karena memerlukan lebih
banyak tempat ke posterior. Diameter anteroposterior pintu bawah panggul diukur dari apeks arkus
pubis ke ujung os koksigis


Otot : m.sphinter ani internus &externus, m.bulbokavernosus, m.transversus perinei superfisialis, m.
iliokoksigeus, m. transversus perinei profunda, m. iskiokoksigeus, m. koksigeus, m. levator ani,
m.konstriktor uretra.
Organ Genitalia Eksterna Organ Genitalia Interna

• Vulva 
 • Vagina 

• Mons pubis 
 • Uterus: fundus, korpus, serviks 

• Labia mayora 
 • Tuba falopii: pars intertisialis, ismus, ampularis, infundibulum
• Labia minora • ovarium
• Klitoris
• Vestibulum
• Bulbus vestibule
• Introitus vagina
• hymen
• perineum
Pada kiri dan di kanan bawah lubang kemih terdapat dua ostia Skene. Saluran Skene (duktus
parauretral) analog dengan kelenjar prostat pada laki-laki. Di kiri dan kanan bawah di dekat fossa
navikulare, terdapat kelenjar Bartolin. Kelenjar ini berukuran diameter lebih kurang 1 cm, terletak di
bawah otot konstriktor kunni dan mempunyai saluran kecil panjang 1,5 - 2 cm yang bermuara di
vestibulum, tidak jauh dari fossa navikulare. Pada koitus kelenjar Bartholin mengeluarkan getah.

HIPERTENSI KEHAMILAN
12. definisi, etiologi dan patofisiologi PEB

PEB
n bukan
a. Preeklampsia à peningkatan tekanan darah ≥140/≥90 mmHg setelah 20 minggu kehamilan
dan menghilang sebelum 12 minggu pasca persalinan disertai proteinuria: protein urin ≥
300 mg/24 jam/dipstick +1/rasio protein:kreatinin ≥ 0.3 atau tanda-tanda preeklampsi
lainnya: Trombosit <100.000, ↑2x dari baseline serum transaminase, kreatinin serum ≥ 1.1
mg/dL, gejala SSP (sakit kepala, penglihatan kabur), edema paru 


b. Etiologi 


1. Kelainan vaskularisasi plasenta

Remodeling A. spiralis Abnormal.
Tidak terjadi invasi trofoblas ke dalam lapisan otot A. spiralis dan
ke jaringan matriks sekitar A. spiralis à lapisan otot tsb tetap kaku dan keras à vasokontriksi
relative A. spiralis (d=200mikron) à penurunan aliran darah uteroplasenta hipoksia à dan
iskemia plasenta

2. Iskemik plasenta, radikal bebas, disfungsi endotel


Iskemik plasenta akan menghasilkan radikal bebas/oksidan yang merupakan senyawa penerima
elektron. Salah satu radikal bebas penting, yakni hidroksil. Hidroksil sangat toksik, dan menyerang
secara khusus terhadap membrane sel endotel sehingga menyebabkan “toxaemia”. Selain itu,
hidroksil juga merusak membrane sel yang banyak asam lemak tidak jenuh sehingga menjadi
peroksida lemak yang nantinya akan merusak membrane sel itu sendiri, merusak nucleus dan
protein sel endotel sehingga menyebabkan disfungsi endotel.
SFLT I anti

angiogenic
-

Disfungsi endotel dapat menyebabkan :


Tromboxan A2
- gangguan metabolism prostaglandin/PG (prostasiklin / PGE2 vasodilator 
kuat) 

4 vasokonstriksi

- agregasi trombosit pada endotel rusak à trombosit kemudian memproduksi



tromboksan à vasokonstriktor kuat 


- perubahan khas pada sel endotel kapiler glomerulus (glomerular capillary 
endotheliosis) 


- peningkatan permeabilitas kapiler 


- peningkatan produksi endotelin (vasokonstriktor/vasopressor) 


- penurunan produksi NO (vasodilator) 


- peningkatan faktor koagulasi 


3. teori imunologi

Ekspresi HLA-G mengalami penurunan à (Immune-maladaptation)à HLA-G berkurang di desidua


daerah plasenta à hambat invasi sel trofoblas ke dalam jaringan desidua ibu à Remodeling A.
spiralis Abnormal.

Fungsi HLA G:

- modulasi respon imun sehingga si ibu tidak menolak hasil konsepsi (plasenta) yang bersifat
asing. 

natural killer cell

- melindungi trofoblas janin dari lisis oleh a
NK cell ibu sehingga mempermudah invasi sel
trofoblas ke dalam jaringan desidua ibu. Jadi, HLA-G adalah faktor prakondisi invasi
trofoblas.


4. Adaptasi Kardiovskular

Terjadi gangguan metabolism PG à Sehingga pembuluh darah kehilangan daya refrakter terhadap
vasopressor sehingga menjadi sangat peka. 


5. Genetic 


Ada faktor keturunan dan familial dengan model gen tunggal. Telah terbukti bahwa ibu dengan pre-
eklampsia 26% anak perempuannya akan mengalami pre-eklampsia pula, sedangkan hanya 8% anak
menantu yang mengalami pre-eklampsia.
4 sperm a sua mi
jgpengaruhin

6. Diet defisiensi gizi


- Konsumsi minyak ikan, minyak hati ikan halibut dapat mengurangi risiko à minyak ikan
mengandung banyak asam lemak tidak jenuh à hambat aktivasi trombosit à hambat
produksi tromboksan (vasokonstriktor kuat) à hambat vasokonstriksi
- Defisiensi kalsium meningkatkan risiko à suplementasi kalsium perlu 

htpxsemua ibu hamil diberi Ca spy cegah PE

hanygdefis punyaefkbimrls-2.n

7. Inflamasi

Faktor risiko dari proses inflamasi berlebihan adalah banyaknya sel trofoblas plasenta, misal pada
plasenta besar dam hamil ganda 


13. Penyakit ibu hamil dengan hipertensi dan proteinuria selain PEB apa?
Hipertensi + kelainan ginjal (nefrotik, nefritik, gagal ginjal)
14. Bagaimana cara bedain hipertensi kronis dan gestasional?
- Onset: hipertensi kronis terdiagnosis sebelum hamil / usia kehamilan < 20 minggu ;
hipertensi gestasional setelah 20 minggu
- Pada hipertensi kronis dapat ditemukan end organ damage
15. Penyebab proteinuria pada PEB
Proteinuria terjadi akibat kerusakan sel glomerulus yang menyebabkan permeabilitas membran
basalis meningkat sehingga terjadi kebocoran protein ke dalam urine 



16. Kalau PEB tidak mau dirawat bagaimana?
Tidak bisa diberikan MgSO4 karena harus monitoring ya

Dexa juga tidak bisa diberikan karena sediaan IM


Hanya diberikan antihipertensi oral à Nifedipine 1x30 mg
17. Patofisiologi HELLP syndrome? Kenapa saran cek LDH?
Hemolisis, peningkatan liver enzyme, low platelet count
Peningkatan enzim hepar: perdarahan pada sel periportal lobus perifer à nekrosis sel hepar dan
peningkatan enzim hepar
Low platelet count: karena increased consumption (platelet nempel di tempat endotel vaskular
rusak)
LDH untuk liat hemolisis. LDH adalah enzim yang banyak terdapat di eritrosi. Apabila terjadi
hemolisis, LDH naik.
18. BPP itu apa? Bisa menggunakan alat apa saja? Apa saja yang dinilai?
S-yaratBA-D-ol.AE ☒


of labor after section
= trial C -

-
④ insisiklasik di uterus
-

vagina , y.mn a, , , , g. gecy.cm , y , ammgge ., ge , gaming gygyem


-
usia
-
pendataran serviks
-
riwayatkelahiran normal -
dilatasi serviks
-
alasanscpertama
kesejahkraan
BPP = biophysical profile. Tujuannya untuk memeriksa kesehatan janin. Pemeriksaan
membutuhkan waktu 30-60 menit. Yang diperiksa: heart rate acceleration, breathing,
movements, tone, amnionic fluid volume CpaKai al 6) at Us

6C-16 hub . DH dgn gerakan janin

→ terminus i

→ gawat janin


MIOMA UTERI

19. Definisi, jenis/klasifikasi (letak), gejala khas, tatalaksana
Mioma (leiomoma/fibromyoma) : tumor jinak yang berasal dari sel otot. Sebagian besar
leiomyoma mengandung jaringan fibroid (dari degenerasi sel otot polos)
Pertumbuhan myoma tergantung gonadal steroid. Ditemukan ada peningkatan reseptor steroid
di myoma dibandingkan myometrium normal. Mioma jarang sebelum menarche, dan
kebanyakan mengecil setelah menopause (penurunan esterogen). Kadang membesar saat hamil
dan penggunaan kontrasepsi oral
Faktor risiko:
- Peningkatan usia
- Menarche awal n
paparan estrogen
>>
- Low parity
- Penggunaan tamoxifen estrogen
4th
u
modulator
pay
/ Kanker
udara

- Obesitas
- High fat diet
Klasifikasi berdasarkan anatomic relationship:
- Intramural
- Subserosa à bikin kontur menonjol pada
pemeriksaan pelvis
à bisa tumbuh wandering ke peritoneum. Bisa
lepas dari uterine blood supply dan dapet intramural
secondary blood supply dari organ lain seperti
omentum (PARASITIC MYOMA)
- Submukosa à bisa sebabkan abnormal vaginal
bleeding atau distorsi uterine cavity jd bisa
sebabkan infertilitas dan keguguran
à dapat membesar dan menjadi bertangkai
(pedunculated) à uterus berusaha keluarin dan
myoma bisa keluar melalui ostium cervix
eksterna
Patogenesis:


Treatment:
Surgery à paling efektif untuk fibroid simptomatik.
Obat:
- Ulipristal acetate à selective progesterone receptor modulator yang berikatan dengan
reseptor progesteron di target tissue
Kontrol gejala, mengecilkan ukuran tumor, improve quality of life
- Aromatase inhibitor à blok sintesis esterogen
Dapat mengecilkan ukuran dan menurunkan gejala (menstrual volume, durasi mens, urinary
retention)
• Emboli arteri uterina à risk IUGR, preterm, abortus lebih besar.
Kalau mau pertahankan fertilitas lebih baik miomektomi dibanding emboli
• Ablasi endometrium à mainly used to manage heavy uterine bleeding
Untuk wanita dengan ukuran uterus yang normal, diameter kurang dari 3 cm
High risk aborsi, ektopik, placental disorder
• Myolysis: destruksi fibrosis atau blood supply via ultrasoud, laser, cryotherapy
Alternatif untuk yang mau pertahankan uterus tp no fertility
Kandidat: small fibroids (£ 5 cm), largest fibroid £ 10 cm
20. MEIG’s syndrome, apa itu?
MEIG’s syndrome: trias ascites, efusi pleura, benign ovarian tumor (fibroma ovarium (terutama
ini), fibrothecoma, tumor Brenner). Akan resolve setelah reseksi tumor
Merupakan diagnosis eksklusi (bisa diagnosis setelah eksklusi kanker ovarium)
Ascites : karena solid ovarian tumor iritasi permukaan peritoneum à stimulasi pembentukan
peritoneal fluid. Hanya tumor yang diameternya lebih dari 10 cm yang bisa bikin ascites
Efusi pleura : patof blm jelas. Hipotesis karena cairan ascites transfer via transdiaphragmatic
lymphatic channels. Banyaknya efusi pleura sangat tergantung dari banyaknya ascites

KETUBAN PECAH DINI



21. Definisi, klasifikasi, faktor risiko, etiologi KPD (pada kasus), patofisiologi, komplikasi?
Definisi : pecahnya selaput ketuban sebelum persalinan, kalo <37 minggu PPROM (preterm
premature ruprture of membrane) kalo 37 keatas PROM
Faktor risiko:
- Kulit hitam
- Sosioekonomi rendah
- Merokok
- Riwayat infeksi menular seksual
- Riwayat persalinan prematur
- Riwayat ketuban pecah dini
- Perdarahan pervaginam
- Distensi uterus (multipel, polihidramnion)
- Dilakukan amniosentesis
- Infeksi/inflamasi koriodesidua
- Penurunan jumlah kolagen membran amnion
Etiologi: penurunan kekuatan membran (degradasi kolagen akibat peningkatan MMP) dan/atau
peningkatan tekanan intrauterin (kontraksi uterus, polihidramnion)
Patof : infeksi, peregangan selaput ketuban, kontraksi berulang à mengeluarkan mediator
prostaglandin, sitokinin, MMPs (matrix metalloproteinase) “Matrix degrading enzyme “
à selaput ketuban inferior rapuh
Komplikasi:
Maternal: Chorioamnionitis à karena penurunan imunitas. Harusnya ada imunitas di cairan
amnion
Abruptio placenta à bisa terjadi kalo
Postpartum endometritis
Fetal: Hyaline membrane disease à risiko meningkat kalau PPROM
Nonreassuring fetal status (deselerasi variabel)
Pulmonary hypoplasia à sering kalo usia < 26 wks
Cerebral palsy à karena chorioamnionitis, intrapartum fetal acidosis & hypoxia
Anomali kongenital
Fetal deformities (facial & skeletal) à karena prolonged PROM dan oligo
22. Kalau KPD kan faktor risikonya hubungan seksual à apakah tidak boleh hubungan seksual jika
sudah 38-39 minggu?
Boleh. Karena hubungan seks rangsang kontraksi uterus à inpartu
With penetration, the lower uterine segment is stimulated. This stimulation results in a local
release of prostaglandins. Female orgasms have been shown to include uterine contractions
sexually active would have significantly shorter active phase and second stage, a normal pattern
of labor, and a higher rate of spontaneous deliveries.
23. Kalau KPD kan faktor risikonya sakit gigi, apa hubungannya?
Periodontitis à meningkatkan MMP à KPD
24. Kalau KPD kan faktor risikonya rokok, apa hubungannya?
Merokok à abnormal collagen cross-link

25. sampe kapan KPD dipertahankan, kenapa dilahirin?
Bila ibu mengalami tanda-tanda korionamnitis dan janin mengalami distress perlu dilahirkan, bila
tidak dapat menimbulkan komplikasi
Bila umur kehamilan 24-37 minggu kehamilan dapat dipertahankan selama mungkin
26. zat apa pada kertas nitrazine (lakmus) yang bisa bikin kertasnya dari merah jadi biru?
pH basa akan bikin kertas lakmus jadi biru
27. tatalaksana KPD sesuai usia kehamilan?
PNPK:
<24 minggu à konseling keluarga, bisa induksi kelahiran/menajemen ekspektatif, evaluasi pasien
per 24-48 jam dengan pemberian antibiotic (USG fetal, monitor suhu perhari), kortiko idak
disarankan rineuroprokktif
24-34 minggu à manajemen ekspektatif, magnesium diberi bila persalinan <24 jam, beri kortiko,
beri antibiotic, NST harian, USG periodic
34-37 minggu à bila fetus sudah matur bisa dipertimbangkan induksi, pertimbangkan kortiko,
pemberian antibiotic, NST harian, USG periodic
>37 miggu à pemberian antibiotic khususnya group beta streptococcus, lahirkan
28. kapan harus terminasi kehamilan? Bila sudah aterm, atau ibu mengalami korionamnitis atau
fetus mengalami distress
29. Kalau KPD 32 minggu gimana?
manajemen ekspektatif, magnesium diberi bila persalinan <24 jam, beri kortiko, beri antibiotic,
NST harian, USG periodic
30. Kalau gravid 33-34 minggu, dengan KPD, tatalaksana gimana?
manajemen ekspektatif, magnesium diberi bila persalinan <24 jam, beri kortiko, beri antibiotic,
NST harian, USG periodic
31. Kalau KPD 34-35 minggu mau diapain? [rawat konservatif 2 hari, tidak perlu pemotongan paru]
32. Apa yang perlu diobservasi saat perawatan konservatif KPD? [Maternal : tanda korioamnionitis,
CRP > (terminasi walau tidak ada tanda infeksi), janin : DJJ]
33. Apa yang perlu diobservasi pada KPD tanpa adanya perubahan TTV/gejala? [CRP >2.7] Cek CRP
tiap berapa lama? (diguideline engga dibilang berapa lama ceknya, tapi ibu perlu diskrining
perharinya)
34. Cara kerja deksametasone pada pasien KPD dan oligohidramnion? [bekerja pada reseptor di
paru untuk menstimulasi sekresi surfaktan, bikin lepcitin]
35. Deksametason apakah efektif untuk janin <28 minggu?
Pembentukan surfaktan baru dimulai 24 minggu dan berakhir sekitar minggu 32-34 minggu.
Apabila diberikan kortiko pada minggu <24 minggu, tidaklah efektif. Steroid berfungsi utuk
pematangaan pneumosit tipe 2 dan supresi terbentuknya septa sekunder

HIDRAMNION

36. Definisi, etiologi
Peningkatan volume cairan amnion yang abnormal. Diklasifikasikan sebagai mild (AFI 25-29,9 cm),
moderate (AFI 30-34,9 cm), dan severe ( AFI >34,9cm)
Etiologi : DM, kongenital abnormalitas (abnormalitas sistem saraf pusat, obstruksi GIT bagian atas,
kelainan toraks fetus, multifetal gestasi, idiopatik

37. Pengukuran AFI dan single deepest pocket? Range normal AFI?
Pengukuran SDP adalah nilai tertinggi dari pengukuran cairan amnion pada 1 dari 4 kuadran.
Pengukuran AFI adalah penjumlahan dari SDP di 4 kuadran.
Range normal SDP 2-8 cm.
Range normal AFI 5-24 cm.
5

CA CORPUS ENDOMETRIUM

38. Tiga tanda gejala Ca corpus endometrium?
• Abnormal uterine bleeding: dapat berupa intermenstrual bleeding, menorrhaghia
• Nyeri pelvis
• Bloating, mudah kenyang, abnormal vaginal discharge

39. Staging Ca endometrium dan tata laksananya?



Tata laksana:


• Surgical
Pasien dengan kanker endometrium sebaiknya dilakukan histerektomi, BSO, and
surgical staging (termasuk pelvic washings dan limfadenektomi). Histerektomi yang
biasa dilakukan adalah tipe 1 atau simple, bisa juga tipe 3 atau radikal jika terdapat
penyebaran ke serviks. Operasi dilakukan secara laparotomy. Pelvic washing dilakukan
dengan menuang 50-100 mL normal saline ke rongga pelvik dan mengambil specimen
sitology. Jika ditemukan massa dalam bentuk apa pun, eksisi dan lakukan biopsy.
Pengawasan POST-OPERATIF
o Pemeriksaan pelvik tiap 3-6 bulan pada 2 tahun pertama dan tiap 6-12 bulan
untuk tahun berikutnya. Bisa dilakukan dengan CT-scan dan MRI juga.
o Pengukuran CA125 serum (terutama jika sudah ada peningkatan sebelum
diterapi)
o Pap’s smear tidak wajib
o Wanita dengan kanker stadium lanjut: + radiasi atau kemoterapi post-op dan
pengawasan post-op lebih ketat
• Chemotherapy
Ada tiga obat sitotoksik yang digunakan untuk terapi kanker endometrium: Paclitaxel
(Taxol), doxorubicin (Adriamycin), dan cisplatin (Platinol) à TAP. Kemoterapi adalah
salah satu terapi adjuvan dalam mengobati kanker endometrium stadium lanjut
bersama dengan terapi operasi dan radiasi.
o Doxorubicin + cisplatin à tidak sebaik TAP, tapi less toksik
o Paclitaxel + carboplatin à sebaik TAP
o TAP à lebih toksik (neuropati perifer)
• Targeted therapy
o Bevacizumab (Avastin) adalah inhibitor dari vascular endothelial growth factor
(VEGF) dan menghambat terjadinya angiogenesis.
o Fibroblast growth factor receptor pathway
o mTOR pathway à metformin + paclitaxel untuk kanker endometrium lanjut
atau rekuren. Metformin mampu menghambar proliferasi sel pada kanker
endometrium melalui jalur mTOR.
• Radiation
o Primary Radiation Therapy à jarang dilakukan, kecuali pada pasien dengan
prognosis operasi buruk. Metode: intracavitary brachytherapy (Heyman
capsules) dengan atau tanpa radiasi pelvik. Survival rate 10-15% < terapi
operasi.
o Adjuvant Radiation Therapy à pada wanita dengan risiko rekurensi tinggi.
Metode: vaginal brachytherapy atau whole pelvic radiation
§ Kanker endometrium stadium 1 dengan moderate-high risk: (1) dengan
3 faktor risiko (tumor grade 2 atau 3, invasi limfovaskuler, dan invasi
2/3 bagian luar myometrium); (2) usia >50 tahun dan 2 dari 3 faktor
risiko; dan (3) usia >70 years dan 1 dari 3 faktor risiko.
§ Kanker endometrium stadium 3: boleh jika + chemotherapy. Terapi
radiasi juga disarankan jika terdapat metastasis ke area paraaortik.
§ Kanker endometrium stadium 4: radioterapi sekedar paliatif.
• Hormonal therapy: tamoxifen + progestin
Kanker endometrium sensitive pada pemberian hormone sehingga pada wanita dengan
kontraindikasi operasi, terapi hormonal berupa progestin kronik atau levonorgestrel-
releasing IUD dapat menjadi pilihan. Tamoxifen meningkatkan ekspresi reseptor
progesterone sehingga baik jika dikombinasikan dengan terapi progestin.

CEPHALOPELVIC DISPROPORTIONAL

40. Penyebab CPD
• Absolut: kontraktur panggul, tumor di jalan lahir, makrosomia, janin dengan
hidrosefalus
o Kontraktur pintu atas panggul: menyebabkan kepala janin tidak dapat turun.
§ Konjugata vera < 10 cm, atau
§ Diameter transversa < 12 cm, atau
§ Konjugata diagonalis < 11 cm
o Kontraktur midpelvis: lebih sering karena kapasitas midpelvis < pintu atas
panggul, dan defleksi/posisi abnormal kepala janin sering terjadi pada daerah
midpelvis.
§ Jumlah antara spina interischial dan diameter sagittal posterior
midpelvis ≤ 13 cm (normal 15 cm).
o Kontraktur pintu bawah panggul: biasanya berhubungan dengan kontraktur
midpelvis.
§ Diameter intertuberosum ≤ 8 cm.
• Relatif: presentasi janin (presentasi alis, wajah) dan posisi janin (persistent oksiput
posterior, kepala defleksi)
41. Grading CPD
Berdasarkan pemeriksaan klinis dan pelvimetri:
• Severe disproportion: konjugata vera <7.5 cm
• Borderline disproportion: konjugata vera antara 9.5-10 cm, diameter anteroposterior
pelvik <10 cm dan diameter transversal pelvik <12 cm
42. Pemeriksaan fisik CPD
Pemeriksaan abdomen (obturator test) dan pemeriksaan abdominovaginal (Muller-Munro
Kerr).
Interpretasi pemeriksaan Obturator:
• Tidak CPD, jika kepala dapat didorong masuk ke pelvis tanpa tumpang tindih antar
tulang parietal pada simfisis pubis.
• CPD sedang, jika kepala dapat didorong sedikit, terdapat sedikit tumpang tindih,
dikonfirmasi dengan pemeriksaan Muller-Munro Kerr.
• CPD berat, jika kepala tidak dapat didorong masuk.
Interpretasi pemeriksaan Muller-Munro Kerr:
• Tidak CPD, kepala dapat didorong, setinggi spina ischial, tidak terdapat tumpang tindih.
• CPD ringan-sedang, kepala dapat didorong sedikit, tidak setinggi spina ischial, dan
terdapat sedikit tumpang tindih.
• CPD berat, kepala tidak dapat didorong.
43. Cara mengetahui CPD
• Pemeriksaan fisik:
o Pemeriksaan abdomen (obturator test)
o Pemeriksaan abdominovaginal (Muller-Munro Kerr).
• Pemeriksaan penunjang:
o X-ray pelvimetri (x-ray lateral pada posisi berdiri)
o Cephalometry: ultrasound, MRI.

44. Sikap terhadap CPD
Tatalaksana CPD pada pintu atas panggul:
• Kontraktur minor → persalinan spontan pervaginam.
• Kontraktur sedang-berat:
o Induksi persalinan preterm
o Seksio sesarea saat aterm
o Trial labor
Tatalaksana CPD pada midpelvis dan pintu bawah panggul:
• Seksio sesarea elektif
• Persalinan pervaginam menggunakan vakum atau forsep pada kontraksi minimal

DM GESTATIONAL

45. Definisi DM gestasional
Kumpulan kelainan metabolik dengan kondisi hiperglikemia yang diderita pasien atau
terdiagnosis saat kehamilan berlangsung.

46. Kalau ada pasien ibu hamil, gimana dicurigai DMG?
Sering BAK, sering haus, mudah lapar, mudah lelah, berat badan turun, mata buram, mual, luka
sulit sembuh, kesemutan

47. Cara diagnosis dan tata laksana DMG?
Diagnosis DMG ditegakkan berdasarkan kriteria satu dari nilai kadar glukosa darah dibawah ini
pada saat dilakukan tes toleransi glukosa oral (TTGO)
TTGO dilakukan dengan prosedur sebagai berikut.
• Minta ibu untuk makan makanan yang cukup karbohidrat selama 3 hari, kemudian
berpuasa selama 8-12 jam sebelum dilakukan pemeriksaan.
• Periksa kadar glukosa darah puasa dari darah vena di pagi hari, kemudian diikuti
pemberian beban glukosa 75 gram yang dilarutkan dalam 200 ml air diminum dalam
waktu paling lama 5 menit
• Dilanjutkan pemeriksaan kadar glukosa darah 1 jam lalu 2 jam kemudian

Tes Kadar glukosa (mmol/L) Kadar glukosa (mg/dL)
Glukosa darah puasa 5.1-6.9 92-125
Glukosa darah 1 jam pasca >10 180
pembebanan glukosa 75
gram
Glukosa darah 2 jam pasca 8.5-11 153-199
pembebanan glukosa 75
gram


Tatalaksana:
• Non farmakologis (gaya hidup):
o Terapi nutrisi (diet)
Kebutuhan kalori = 35 kkal/kgBB x BBI
BBI = (TB-100) – 10%(TB-100)
Jika IMT >30 kg/m2 à 25 kkal/kgBB
Karbohidrat 30-35% dari kalori total
o Aktivitas fisik
Intensitas sedang 150 menit/minggu
o Menjaga berat badan
Peningkatan BB -7 kg atau -18 kg jika IMT <18,5 kg/m2
Wanita obese peningkatan BB tidak boleh lebih dari 11,4 kg
• Farmakologis:
o Insulin (first line)
Perlu disesuaikan dosisnya
Umumnya dosis insulin yang dibutuhkan 0.7 U/kg pada trimester dan
ditingkatkan menjadi 1.1 U/kg saat kehamilan term. Dalam memulai terapi
insulin, dilakukan penghitungan total dosis harian 0.7 hingga 1.0 unit per kg.
Setengah dari total kebutuhan harian diberikan menggunakan satu dosis insulin
kerja panjang, dan separuh lainnya diberikan dalam tiga dosis terbagi pada
waktu makan menggunakan insulin kerja cepat.



o Metformin
Lebih dipilih terutama jika gula darah dapat dikontrol, risiko hipoglikemia dan
peningkatan berat badan lebih kecil
o Sulfonilurea mglimepiride
Inferior dibandingkan obat lain, meningkatkan risiko hipoglikemia bayi dan
makrosomia, belum ada data aman jangka panjang



48. Etiologi DMG?
Kekurangan insulin yang cukup untuk menjaga homeostasis gula darah.
• Gangguan produksi insulin
• Gangguan reseptor glukosa pada pankreas
• Gangguan reseptor insulin pada jaringan perifer
Terlebih lagi, pada ibu hamil terdapat perubahan metabolisme karbohidrat:
• Peningkatan resistensi insulin
o Produksi human placental lactogen
o Peningkatan produksi kortisol, estriol, dan progesteron
o Peningkatan destruksi insulin
• Peningkatan lipolisis
o Glukosa untuk kebutuhan janin, sedangkan energy ibu didapat dari
pembakaran lemak
• Perubahan gluconeogenesis
o Alanin dan asam amino lain à janin, sumber glukoneogenesis
ibu à berbeda

SECTIO CAESAREA

49. Kenapa pasien di SC? Coba jelaskan VBAC itu apa?
VBAC SCORE FLAMM GEIGER digunakan untuk memprediksi keberhasilan persalinan spontan
pervaginam (VBAC) pada pasien dengan riwayat seksio sesarea.


Makin I score nya →
keberhasilan V13 Act




LETAK LINTANG

50. Kalau 31-32 minggu dengan letak melintang apa yang dilakukan dengan kondisi kamu ada di
pedalaman? Trus kalau sudah bukaan lengkap bagaimana? Kalau belum bukaan lengkap gimana?

external
cephalic
version



51. Primigravida, 26-27 minggu, IUFD, dengan letak lintang, kamu satu-satunya dokter yang ada di
pulau tersebut. Apa yang akan kamu lakukan?
Pasien primigravida dengan IUFD dan letak lintang sangat sulit untuk dilakukan persalinan
pervaginam dan dapat menyebabkan rupture uteri akibat uterus yang terus berkontraksi namun bayi
sulit untuk dikeluarkan. Selain itu terdapat penyulit lain dimana letak lintang biasanya disertai dengan
plasenta previa yang dapat menyebabkan prolaps tali pusat. Menurut Williams, persalinan spontan
pervaginam dapat dicapai apabila ukuran fetus yang kecil (< 800 gram) dan pelvis yang besar. Apabila
pasien dengan letak lintang datang dalam keadaan inpartu maka pasien tersebut merupakan indikasi
untuk dilakukan SC. Tapi apabila pasien datang sebelum inpartu atau awal-awal kala I fase laten,
dengan syarat selaput ketuban belum pecah dan tidak ada komplikasi lainnya maka dapat dilakukan
versi luar. Apabila kepala janin dapat diputar dengan manuver pada abdomen maka perlu ditahan
pada saat kontraksi-kontraksi berikutnya dengan tujuan untuk memfiksasi kepala janin di pelvis. SC
yang disarankan adalah insisi vertikal pada uterus untuk memduahkan ekstraksi janin.
Menurut Dudenhausen, letak melintang dapat dilahirkan pervaginam dengan indikasi :
• Stillbirth
• Persalinan preterm dengan EFW < 500 gram
• Anak kedua dari gemelli
• Tidak ada indikasi lainnya untuk SC seperti plasenta previa
Kalau menurut saya yang pertama dilakukan adalah versi luar lalu mengukur TFU dan menentukan TBJ
apabila memungkinkan untuk persalinan pervaginam tanpa adanya komplikasi lainnya maka
dilanjutkan dengan cervical ripening dan diinduksi untuk persalinan pervaginam.

GESTATIONAL TROPHOBLASTIC DISEASE

52. Jelaskan GTD dan GTN itu apa beserta patofisiologi dan jenis-jenisnya
Ugestasional trophoblast's neoplasia

GTD (gestational trophoblastic disease)


merupakan kelompok tumor yang dikenal dengan
abnormalitas proliferasi dari trofoblas. Trofoblas
menghasilkan hCG, sheingga pemeriksaan hormone hCG
sangat penting dalam menunjang diagnosis, tatalaksana,
dan surveilans. GTD dibagi menjadi 2 yaitu mola
hidatidosa (adanya villi) dan non molar trophoblastic
neoplasm (villi sedikit). Nonmolar trophoblastic
neoplasma (GTN) merupakan GTD dalam bentuk ganas,
terdiri dari choriocarcinoma, placental site trophoblastic
tumor (PSTT), dan epitheloid trophoblastic tumor (ETT).
Ketiga tipe keganasan ini dibedakan dari tipe trofoblas
yang dimiliki. GTN juga dikenal dengan GTD ganas dan
GTD persisten (bertahan minggu hingga tahunan
seringkali terjadi setelah mola hidatidosa).

MOLA HIDATIDOSA

53. Definisi, etiologi dan faktor risiko mola hidatidosa
Mola hidatidosa merupakan plasenta imatur yang
banyak mengalami pembengkakan. Etiologi berasal dari
komposisi fertilisasi yang abnormal.
Faktor risiko :
• Usia (usia ekstrim > 40 tahun)
• Riwayat mola sebelumnya
(mola komplit 1,5% berisiko
rekuren, parsial 2,7%)
• Ras asian, hispanik, amerika
india

54. Jenis-jenis mola
Mola hidatidosa dibagi
menjadi 3 yaitu mola komplit, mola
parsial dan mola invasive. Mola
invasive dianggap malignan karena
penetrasi nya ke dalam dan merusak
miometrium serta memiliki
kemampuan untuk bermetastasis.

55. Definisi mola destruens
Mola invasive merupakan GTN paling umum yang muncul setelah kejadian mola hidatidosa,
dan hampir semua mola invasive muncul dari mola komplit atau parsial. Chorioadenoma destruens
merupakan mola invasive yang dikarakterisasi dengan invasi jaringan yang luas oleh trofoblas dan
seluruh villi. Terdapat penetrasi ke dalam miometrium dan terkadang melibatkan peritoneum,
paramterium sekitarnya, atau vagina. Walaupun mola invasive agresif secara lokal, mola invasive lebih
jarang bermetastasis dibandingkan dengan choriocarcinoma.

56. Mola high risk? Faktor risiko mola menjadi ganas?

GTNT
Skor P →
kemungkinan



57. Tatalaksana mola? Low risk vs high risk

GTN terbaik ditatalaksana oleh onkologis dengan kemoterapi sebagai tatalaksana utama, dan
evakuasi berulang tidak dianjurkan oleh kebanyakan ahli karena dapat menyebabkan perforasi uterus,
perdarahan, infeksi, atau adhesi intrauterine.
Pada neoplasia non metastatic atau risiko rendah metastatic cukup diberikan kemoterapi
agen tunggal dengan metotreksat atau aktinomisin D (keduanya sama-sama efektif namun secara
umum MTX lebih kurang toksik dibandingkan aktinomisin). Pemberian obat ini diulang terus hingga
kadar beta hCG tidak dapat dideteksi.
Pada risiko tinggi dapat diberikan kemoterapi kombinasi dengan menggunakan EMA-CO
(etoposide, metotreksat, aktinomisin D, cyclophosphamide, dan oncobin (vincristine). Terapi bedah
adjuvant dan radioterapi dapat juga diberikan bersamaan. Baik kelompok risiko rendah atau tinggi,
ketika serum beta hcg tidak dapat terdeteksi, serosurveilans dilanjutkan untuk 1 tahun. Pada saat
masa serosurveilans ini, pasien tidak boleh hamil untuk mencegah efek teratogenic dari kemoterapi
terhadap janin dan mencegah kebingungan dari peningkatan beta hcg akibat superimposed kehamilan
(jadi terlihat gagal padahal peningkatan beta hcg bukan berasal dari GTN tapi dari kehamilannya)







58. Obat mola apa aja?

anesthesia utero tonika 4 Kurt


paracervical
+
n



59. Jika kamu menemukan kasus mola dengan perdarahan sangat banyak, dan tidak ada
transportasi untuk merujuk, apa yang akan kamu lakukan?
(Kalau saya) Evakuasi mola dengan menggunakan suction curettage, dilatasi servikal
preoperatif dengan menggunakan agen osmotic jika serviks berdilatasi minimal. Pemberian anestesi
adekuat, pemasangan jalur IV, dan transfuse darah jika perdarahan hebat. Dilatasi serviks dapat
dilakukan secara mekanik dengan pemasangan kuret suction berukuran 10-14 mm. Ketika evakuasi
sedang berjalan, oksitosin di drip untuk menghentikan perdarahan. Pemeriksaan beta hcg awal
dilakukan 48 jam setelah evakuasi sebagai baseline.

60. Skrining terapi mola? Follow up nya?
Surveilans biokimia dilakukan dengan pemeriksaan serum beta hcg serial untuk mendeteksi
prolifersi trofoblastik persisten atau yang baru. Pemeriksaan beta hcg awal didapatkan 48 jam setelah
evakuasi sebagai baseline dibandingkan dengan kuantifikasi setelahnya setiap 1 hingga 2 minggu
sampai kadar beta hcg menurun secara progresif hingga tidak dapat terdeteksi. Waktu median untuk
resolusi pada mola parsialis 7 minggu dan mola komplit 9 minggu. Ketika beta hcg sudah tidak dapat
dideteksi, dikonfirmasi kembali setiap bulan selama 6 bulan. Apabila tetap tidak terdeteksi,
pemeriksaan dapat dihentikan dan kehamilan dapat terjadi.
Terdapat faktor risiko untuk perkembangan GTN setelah proses evakuasi mola :
• Mola komplit memiliki 15-20% insiden dari sekuele malignan (parsial 1-5%)
• Identifikasi mola secara dini dan evakuasi dini tidak menurunkan risiko malignansi
• Usia tua
• Kadar beta hcg > 100.000 mIU/mL
• Ukuran uterus lebih besar dari usia gestasi
• Kista techa lutein > 6 cm
• Penurunan lambat dari kadar beta hcg
USG postevakuasi juga dilakukan, nodul miometrium atau hipervaskularitas dapat menjadi predictor
untuk keganasan.

SUNGSANG

61. faktor risiko sungsang

Faktor resiko :
- early gestasional age
- abnormal amniotic fluid volume
- mutifetal gestation
-
hydrocephali, anencephali
- uterine abnormalities
- placenta previa, fundal placental implantation -
pelvic tumor
- multiparitas
- riwayat sunsang sebelumnya

62.jenis-jenis sungsang, disuru pake boneka peragakan

a. Complete breech! kedua sendi paha fleksi dan kedua sendi lutut fleksi


b. Incomplete breech! satu atau dua sendi paha fleksi, satu sendi lutut fleksi dan satu lagi ekstensi


c. Frank breech! kedua sendi paha fleksi namun kedua sendi lutu ekstensi
4 marni

d. Footling! satu atau kedua kaki terletak di bawah bokong

63. Emang sungsang boleh partograf? lambang sungsang apa di partograf? [sungsang 
gak pake
Upd penurunan pakaiw
Otp
bukan
partograf] 
 umumnya pakai modified partograf

64. Kasus = inpartu, presenrasi bokong Hodge III, mulai kelelahan, His lemah

Resusitasi + augmentasi (D5% + oksitosin 5IU mulai 20 tpm) [D5% untuk energi]

Oksigen via nasal kanul.

à
Setelah His bagus Bracht, Loveset tengan
64 lahirkan bahu +

à
Saat mau lahirin kepala, kalo tiba-tiba His ibu jelek lagi Mauriceau + 1 orang tekan suprapubic + 1
orang dorong bayi dari perut ibu

à
Setelah bayi lahir suction, klem tali pusat + potong, resusitasi neonatus

PLASENTA PREVIA
65. cara diagnosis?

Anam: perdarahan pervaginam sedikit2 (flek) warna merah segar, painless, tidak ada pencetus

PF : cek tanda2 hipovolemia & anemia, malpresentasi, in spekulo: os tertutup plasenta (tidak boleh
VT)

USG: plasenta pada segmen bawah uterus


66. faktor risiko dan etiologi plasenta previa?

FR: bekas SC, maternal age, multiparitas, merokok

Etio: abnormalitas endometrium (jar parut, penurunan vaskular!SC, kuret, infeksi endometrium),
abnormalitas plasenta (multiple pregnancy, large placenta), delayed development of trophoblast

67. patofisiologi plasenta previa?

68. tatalaksana plasenta previa


69. kapan konservatif, kapan terminasi?

Preterm n mild bleeding!konservatif Severe ! terminasi

70. beda plasenta previa, solusio plasenta, vasa previa? tatalaksana-nya?


Plasenta previa : plasenta yang berimplantasi pada segmen bawah rahim sehingga menutup seluruh
atau sebagian dari oui
Vasa previa: pembuluh darah janin berada di dalam selaput ketuban dan
depan
melewati oui kemudian sampai ke dalam insersi di tali pusat.
Shg Krdapat Pla
1 sent a Utama & 1
plan lit nta sate
Solusio plasenta: terlepasnya swbagian atau seluruh permukaan maternal plasenta dari tempat
implantasi yang normal pada lapisan desidua endometrium sebelum waktunya.

71. kenapa plasenta previa bisa berdarah?
Perlengkatan pada bagian bawah uterus yang dengan
bertambahnya usia kehamilan akan terjadi pematangan bagian bawah rahim yang meregang sehingga
terjadi laserasi pada plasenta yang keluar pada ostium

72. solusio plasenta berat 3 tanda-nya apa? TFU lebih tinggi Defance muscular Djj tidak terdengar

73. vasa previa tatalaksananya apa? Ugh…. SC(?)



74. kalo pasiennya datang lagi dengan perdarahan, mau diapain? à manajemen ekspektan

CA SERVIKS
75. tipe-tipe ca serviks?

76. staging ca serviks dan tatalaksananya?

KETUBAN PECAH DINI KEHA MILAN EKTOPIK

77. etiologic [dari buku merah ada 5, jangan lupa infeksi clamidia dan GO]
- Faktor tuba : radang/infeksi à tuba sempit ato buntu. Ato pascaoperasi tuba, endometriosis tuba,
divertikel tubal (kongenital), miom ato tumor ovarium yang bikin perubahan dan patensi tuba

- Faktor abnormalitas zigot à tumbuh terlalu cepat ato terlalu besar à terhenti di tuba

- Faktor ovarium : ovum yang ditangkep tuba kontralateral

- Faktor hormonal : kontrasepsi progesterone à gerakan tuba jadi melambat

- Faktor lain : IUD à radang endometrium dan endosalping, faktor umur ibu yang lanjut, rokok

- Infeksi klamidia, GO

78. faktor resiko?


s

79. patofisiologi?
Dari faktor-faktor resiko diatas à bikin abnormalitas di tuba falopii à implementasi ovum di
tempat yang tidak seharusnya à erosi dan disrupsi dinding tuba (bikin nyeri) à lalu rupture à
bleeding and pain

80. diagnosis KET? (anam, pf, pp)
Trias : amenorhhea, vaginal bleeding or spotting, abdominal pain
Dari PF bisa ketemu : tenderness of the abdomen, nyeri goyang serviks, kehrs sign (refered pain;
dari phrenic nerve; nyeri yang menjalar hingga ke bahu karna iritasi diafragma kalo ga salah),
presence of blood or other irritants in the peritoneal cavity when a
hematokel retrouterina (darah di cul-de-sac) person is lying down and the legs are elevated
β-hCG à serum > 1500 mIU/ml untuk pemeriksaan TVS, > 6000 mIU/ml untuk pemeriksaan
transabdominal, progesterone < 5ng/ml
TVS à pseudogestational sac, free fluid di retrouterine cul-de-sac (kalo perdarahannya lebih dari
50 cc), empty cavum uteri, decidual cyst, ring-of-fire
Culdocentesis à tusuk cul-de-sac dengan spuit, trus tarik. Kalo positif ada blood clot
Fonslani test à cek Hb serial (kalo dr arie bilang ga Cuma Hb.. kayanya ditambah Ht deh)
6
apakah adat drastis dari Hb ibu


81. pasien KET nya mau diapain? [laparotomi salphingectomy]
bisa expectant, bisa medical, bisa surgical.
Syarat kalo mau expectant : β-hCG stabil dan cenderung turun, massa ektopik nya ≤ 3,5 cm, tidak
ada abdominal bleeding, gaada tanda rupture di TVS, gaada DJJ
Syarat kalo mau medical : asimptomatik pasiennya, trus musti compliant. Dan ada tanda” kalo
emang blom gawat KE nya
Medical : methotrexate (MTX) à antagonis asam folat. Dosis 50mg/m2 body surface area (single
dose) atau1 mg/kgBB (multidose, hari ke 1,3,5,7)
: Leucovorin à asam folat. Dosis 0,1 mg/kgBB, hari ke 2,4,6,8 (multidose)
Kalo surgical, pilihan besarnya bisa laparotomy ato laparoskopi. Laparoskopi kalo pasiennya
secara hemodinamik stabil. Abis itu bisa dilanjutin either salphingostomy (kalo massa blom
rupture, ukuran < 2 cm, posisi di 1/3 distal tuba) ato salphingectomy

82. kenapa laparotomy bukan laparoskopi? [laparoskopi gold standard untuk KET]

83. bedanya ruptur KET sama torsi kista?
Cavum douglasi nya(?) à kalo torsi kista harusnya kosong, kalo KET kan harusnya keisi since dia
udah rupture

KALA II MEMANJANG, FORSEP VAKUM
84. kala II memanjang kira-kira kenapa?
Faktor ibu kelelahan
Malposisi
His inadekuat
Rigid perineum

85. indikasi dan syarat forsep?
Indikasi forceps :
• Relative
è Indikasi de Lee
- Kepala telah berada di dasar panggul
- Putaran paksi dalam sudah sempurna
- M. levator ani sudah teregang
è Indikasi Pinard
- Kala II memanjang (sudah mengejan selama 2 jam)
• Absolut
è Janin : gawat janin
è Ibu :
- Penyakit jantung kompensata, penyakit paru fibrotic
- Kelelahan
- Infeksi intrapartum
- Kala II memanjang
- Eclampsia, preeklampsia

Syarat forceps :
• Gaada kontraindikasi PSP (ex CPD)
• Pembukaan lengkap
• Sudah engagement
• Kepala janin bisa dipegang dengan cunam
• Janin hidup
• Ketuban sudah pecah
86. indikasi dan syarat vakum?
Indikasi vakum :
• Ibu
è penyakit jantung kompensata, penyakit paru fibrotic
è kelelahan
è infeksi intrapartum
è kala II memanjang
minimal hodge 3,
• Janin
pembukaan lengkap,
è Gawat janin, non-reassuring amniotomi
è Prolapse umbilical cord
è Premature separataion of placenta nsolusioplasenta

Syarat vakum :
- Pembukaan lebih dari 7 cm ([ada multigravida)
- Penurunan kepala janin pada hodge II
- Harus terdapat kontraksi rahim dan tenaga mengejan dari ibu


BUCUR dr. YUMA
1. Apa saja yang menyangga uterus?

Otot : m.sphinter ani internus &externus, m.bulbokavernosus, m.transversus perinei


superfisialis, m. iliokoksigeus, m. transversus perinei profunda, m. iskiokoksigeus, m.
koksigeus, m. levator ani, m.konstriktor uretra
Ligament :
- Lig. Cardinal (mackendrot) sinistra et dextra : terpenting , untuk mencegah uterus tidak
turun, mengikat serviks dan vagina ke lateral dinding pelvis
- Lig. Sakrouterina sin et dext : menahan uterus tidak banyak bergerak. Dari belakang
serviks kiri dan kanan melalui dinding rectum ke os sacrum kiri kanan
- Lig. Rotundum sin et dex (round ligament) : menahan uterus dalam posisi antefleksi.
Mengikat dari sudut fundus uteri kiri kanan ke inguinal kiri kanan
- Lig latum sin et dext (broad ligament) : ligamentum yang meliputi tuba. Berjalan dari
uterus ke arah lateral
- Lig. Infundibulopelvikum/
lig ovarii proprium :
menahan tuba fallopi, dari
infundibulum ke dinding
pelvis
- ligamentum ovarii
proprium sin et dext:
menahan ovarium. Pada
sudut kiri dan kanan
belakang fundus uteri

2. Tanda-tanda kehamilan à
Tanda-tanda presumtif =
perubahan fisiologik pada ibu
- Amenorea
- Mual&muntah
- Mengidam
- Payudara membesar
- Anoreksia
- Lelah
- Pigmentasi pada kulit
- Sering miksi
Tanda tidak pasti atau terduga hamil = perubahan anatomik & fisiologik pada ibu
- Perut membesar
- Uterus membesar
- Tanda hegar
- Tanda chadwick
- Tanda pisaseck
- Kontraksi Braxton hicks
- Teraba ballottement
- Pemeriksaan tes biologis kehamilan (+)

Tanda pasti kehamilan


- Gerakan janin dalam Rahim
- Terlihat/teraba gerakan janin & bagian2 janin
- Terdengar DJJ
- Pemeriksaan USG
3. Bishop score bisa 0 atau tidak? (tidak, minimal 2 karna konsistensi serviks hamil lunak, jadi skor
minimalnya 2)
4. Cara tubektomi KB ? à buku ijo hal 239

5. Jenis tubektomi à

6. Tanda inpartu apa saja ?


Inpartu ; kontraksi uterus yang efektif u/ menghasilkan pendataran dan dilatasi servix

Tanda : Bloody show, cairan bening tak tertahankan , kontraksi dominan dari fundus

7. His adekuat bagaimana ?


- Frekuensi 3-5x/ 10 menit
- Relaksasi 1-2 menit
- Tekanan 60-90 mmHg
- Lama durasi 40-60 detik
- Simetris
- Kontraksi dominan dari fundus

8. Apa yang dinilai saat pd? Kenapa nilai sacrum, sacrum yang baik gimana , linea inominta yang
baik tu bagaimana ?

PAP :
o konjugata diagonalis à dari bawah simfisis ke promontorium 11 +- 1,5 cm
o konjugata vera à dari atas simfisis ke promontorium (kon.diagonalis – 1,5) biasa 11 cm
o konjugata obstetric à dari tengah simfisis ke promontorium (jarang dipake)
o linea inominata (terminalis ) à yang bagus teraba <1/3
o ada juga diameter anteroposterior, diameter transversa (12,5-13), diameter oblikua (13
cm) à tapi kayaknya gabisa diukur pas PD
o ruang panggul : spina ischiadica runcing / tumpul à bagusan runcing supaya bidang
gesekan sama kepala bayi lebih sedikit
o Distansia interspinarum : 10,5 cm atau lebih bagusnya
o Konkavitas sacrum à bagusnya konkav , dan sudut lumosakral (inklinasi) lebih besar
supaya kepala gampang masuk

PBP :
o arkus pubis à bagusnya >90 derajat
o Distansia tuberum à jarak atara kedua tuber os.ischium , bagusnya >10cm

9. Molase ada berapa


a. 0 : tulang kepala terpisah, sutura teraba dengan mudah
b. 1 : tulang kepala bersentuhan
c. 2: tulang kepala tumpang tindih namun dapat dipisahkan
d. 3: tumpang tindih dan tidak dapat dipisahkan

konsepsi shg endometrium menebal d. luruh ( breakthrough bleeding


)
Pt tp ada has it
y progesterone

10. arias stella reaction à perubahan jinak dari endometrium. Dapat ditemukan pada kehamilan
normal. Di bawah perngaruh hormon estrogen dan progesterone dari korpus luteum gravidarum
dan trofoblas à uterus jadi besar dan lembek, endometrium berubah menjadi desidua.
Endometrium mengalami perubahan: sel epitel membesar, nukleus hipertrofi, hiperkromasi,
lobuler, bentuk ireguler. Sitoplasma mengalami vakuolisasi seperti buih dan dapat dijumpai
mitosis.
Untuk diagnosis KET à (+) jaringan yang dikeluarkan bersama dengan perdarahan terdiri atas
desidua tanpa vili korialis
11. tanda perubahan anatomi fisiologi ibu hamil à
1) Pembesaran Uterus
2) Tanda chadwick: perubahan warna vulva, vagina, serviks menjadi kebiruan/keunguan
3) Tanda goodell: perubahan konsistensi serviks dari kenyal menjadi lunak
4) Tanda hegar: perlunakan dan kompresibilitas isthmus serviks sehingga ujung-ujung jari
seakan dapat ditemukan jika isthmus ditekan dari arah yang berlawanan (pemeriksaan
bimanual: jari2 pada forniks anterior sama abdominal bisa saling keraba). Terjadi pada
kehamilan 6-8 minggu.
5) Tanda piskacek: pembesaran uterus yang tidak simetris pada awal kehamilan karena tempat
implantasi plasenta yg lebih tebal.
6) Kontraksi Braxton hicks: kontraksi non ritmik, sporadik, tanpa disertai rasa nyeri. Mulai
timbul pada kehamilan 6 minggu akibat pembesaran uterus menyebabkan peregangan
myometrium
7) DJJ: mulai berdenyut sejak awal minggu ke 4 fertilisasi. Dapat didengarkan pada minggu ke
12-20mgg dengan doppler
8) Gerak janin: mulai terasa pada usia kehamilan 16-20mgg
9) Palpasi tubuh janin pada kehamilan 20mgg
10) Ballottement (+)
11) hCG (+) pada 26 hari setelah konsepsi, puncak 60-70 hari dan menurun bertahap dan
menetap hingga akhir kehamilan
12. bagaimana menjahit rupture perineum berdasarkan grade ?

etiologi rupture : kepala lahir terlalu cepat, cara memimpin salah, banyak jaringan parut di
perineum , distosia bahu
teknik menjahit :
Aku lit } mukosa vagina
a. grade I : menggunakan catgut dengan teknik continuous / figure of 8
→ otot perineum

b. grade 2 : pinggir robekan di klem lalu digunting agar rapih à otot perineum jahit dengan
catgut à mukosa vagina dijahit dengan catgut dengan terputus atau continuous (dimulai
→ Klem pean
turns dari puncak robekan) à kulit perineum dengan benang sutura secara terputus
sphincter > 50-1 sphincter ext ; 3C
Mf
~ 3A
:<50% ext ; 313 : : sphincter int

c. grade 3 : otot sphinter ani diklem dengan klem PEAN LURUS à jahit 2-3 jahitan dengan
.

catgut kromik à lanjut ke langkah grade 2


n rectum

d. grade 4 : mukosa depan rectum dijahit à fasia perirectal dan rectovaginal dijait dengan
catgut kromik à lanjut ke langkah grade 3 (kalo end to end teknik liat yang 50 soal )

13. kala 2 memanjang pada nuli dan multi berapa lama à nulli > 1 jam, multi > 2 jam
multi para nullipara
Kala I fase later
me mahjong → > 20 jam pd multi & > 14 jam pd multi

fase aktif → 1cm /


jam nambah pembukaannya

KETUBAN PECAH DINI


14. definisi, patof, predisposisi

definisi : pecahnya selaput ketuban sebelum persalinan, kalo <37 minggu PPROM (preterm
premature ruprture of membrane) kalo 37 keatas PROM

patof : infeksi, peregangan selaput ketuban, kontraksi berulang à mengeluarkan mediator


prostaglandin, sitokinin, MMPs (matrix metalloproteinase) “Matrix degrading enzyme “ à
selaput ketuban inferior rapuh bukan seluruh selaput rapuh

factor : - berkurangnya asam askorbat dan tembaga karena merokok à padahal as. Askorbat itu
komponen kolagen (penyusun selaput ketuban)
- periodontitis à meningkatkan MMPs
- kehamilan trimester akhir à MMPs menigkat tapi FISIOLOGIS
- ISK à ke vagina -> asenden
- sering pada polihidramnion , inkompetensi serviks, solusio plasenta

15. penyebab KPD pada pasien mu ?


16. korioamnionitis gejalanya gimana? Tatalaksana ?

Ibu : takikardi, demam >38, berkeringat, leukosit >15 ribu, crp naik, uterus menjadi lembek pada
perabaan dengan/ tanpa nyeri, cairan amnion berbau keluar dari vagina
Janin : takikardi

Tatalaksana :
1. Terminasi kehamilan à seviks belom matang: prostaglandin dulu baru oksitosin ATAU SC
(tapi bisa sepsis) , kalo sudah matang : oksitosin
2. ANTIBIOTIK SEGERA: ampisilin 3x1 g , metronidazole 3 x 500 mg, gentamisin 5mg/kgBB/hari
SEMUA IV
kalo pervaginam à hentikan Ab abis lahiran, kalo SC à sampe 48 jam bebas demam
uterotonik pasca salin à supaya kontraksi baik à hambat invasi mikroorganisme melalui sinus
pemb.darah pada dinding uterus

17. surfaktan dihasilkan minggu ke berapa ?

minggu 24 dan puncaknya minggu ke 32


surfaktan terdiri dari sfingomielin dan lesitin serta fosfatidil gliserol
tanda paru matang à dari amniosintesis diukur L/S >2 à matang
hterbentukcincinungu
'

Kala U Lls 12 bahaya RDS

18. Tatalaksana KPD sesuai kehamilan dan kenapa ? (William)


a. 34 minggu atau lebih : aktif (kalo di buku merah >37 minggu)à dengan induksi
persalinan
i. kalo bishop skor <5 à misoprostol 25 mcg intravaginal tiap 6 jam maksimal 4
kalià kemudian oksitosin à gagal à SC
ii. kalo bishop >5 à oksitosin
b. 24-33 à konservatif , biasa 2 hari karna sesuai dexa
i. kortikosteroid u/ pematangan paru (betametason 12 mg sehari dosis tunggal
selama 2 hari atau dexametason 2x 6 mg selama 2 hari IM
ii. tokolitik kalau ada his --> nifedipin 3x20
1- mg

iii. antibiotik à ceftriaxone 2x1 g IV ATAU (sefadroxil 2x 500, eritromisin 4x500


selama 3-5 hari dan dapat diberikan metronidazole 2x500 selama 7 hari) PO
iv. kalau sudah tidak keluar cairan à pulang à cek tiap minggu sampai usia
kehamilan 34 à terminasi
v. kalau ternyata ada infeksi à terminasi Nifedipin merupakan antihipertensi golongan
c. <24 à aktif (terminasi) atau konservatif ? calcium channel blocker. Dosis penggunaan
nifedipine sebagai tokolitik adalah 30-mg
i. kortikosteroid tidak disarankan loading dose peroral, selanjutnya 10 - 20 mg
setiap 4 - 6 jam (dosis maksimal  180 mg
perhari).
19. kalo 37 minggu mau diapain ? à di no 18
20. kalo 32-34 mau diapain à di no 18
21. kenapa kalo <24 minggu harus diterminasi ?
a. mencegah terjadinya korioamnionitis à lebih sering pada premature daripada aterm
b. oligohidramnion à menekan tali pusat à asfiksi dan hipoksia à gawat janin
c. IUGR, deformitas Karena kompresi muka, anggota badan
1
d. Hipoplasi pulmonary (karna surfaktan baru dibentuk usia 24 minggu)
22. Kenapa KPD aterm harus diinduksi / diaugmentasi ?
Karena usia janin sudah aterm, Takut terjadi komplikasi seperti korioamnionitis,
oligohidramnion yang dapat membuat jawat janin
23. Kenapa dikasi AB? Apa yang diperiksa u/ tau tanda infeksi? à sudah ada, Kalau tidak ada
masalah kenapa tidak dipertahankan
(gatau di usia kehamilan berapa ini
kasusnya)
24. Komplikasi KPD à persalinan
premature, infeksi, hipoksia karena
kompresi tali pusat, deformitas janin
25. Bishop score pasien kamu à
tergantung kasus

Bishop
lffacement

station
Hard , medium , soft

Opening
Position
26. VBAC SCORE à sudah ada
27. Definisi ancaman partus prematurus?
Ancaman Partus Prematurus (APP) adalah adanya suatu ancaman pada kehamilan yaitu
timbulnya tanda-tanda persalinan pada usia kehamilan yang belum aterm (20- 37 minggu)
28. Kenapa tidak di dx ancaman partus prematurus? à sesuai kasus
29. Kalo ripening cervix gagal harus diapain? Kalau sudah di induksi gagal à SC
30. VT apa saja yang dinilai

VT OBSTETRI
a. Portio = letak (ant, media, post)
b. Konsistensi
c. Pendataran
d. Dilatasi
e. Presentasi
f. Denominator
g. Ketuban intak / tidak
h. Hodge
31. Etiologi dan patofisiologi OLIGOHIDRAMNION?
Etiologi :
- Kehamilan >40 minggu à terjadi apoptosis plasentaà insufisiensi àseminggu turun 8%
- Rupture membrane (KPD)
- Solusio plasenta
- Insufisiensi plasenta berat yang kronik (Hipertensi, Preeklampsia, DM gestational)
- Konsumsi ACE inhibitor
- Anomaly janin (15-25%) c/ renal agenesis, multicystic dysplastic kidney
- Dehidrasi

Patof :
- insufisiensi plasenta à cairan ke sirkulasi bayi berkurang à filtrasi berkurang à urin
berkurang à amnion berkurang
- Anomali janin à gangguan produksi urinà amnion berkurang
- ACE inh à hypotensi à perfusi utero plasenta berkurang

32. Gambaran CTG pada oligohidramnion?


Deselerasi variable, juga pada lilitan tali pusat , tapi selama variabilitas masih baik biasa janin
tidak mengalami hipoksia yang berarti

Kriteria deselerasi variable :


- bervariasi saat timbulnya, lamanya, amplitudonya
-biasa ada akselerasi sebelum atau sesudah terjadinya deselerasi
- dianggap berat kalau RULE OF SIXTY yaitu desel mencapai 60 dpm di bawah baseline dan
lamanya >60 detik

33. Gawat janin dapat dilihat dari mana? CTG nya gmn?
-Disebut gawat janin bila djj >160 / <120 / menit, denyut tidak teratur, keluarnya meconium
yang kental pada awal persalinan
- dilihat dari djj, ctg, fetal blood sampling pH <7,2 karena asidosis TAPI ga dipakai lg
-CTG : hasil NST Disebut abnormal kalau ada à bradikardi, deselerasi 40dpm atau lebih dari
baseline, denyut jantung janin mencapai 90 dpm selama minimal 60 detik.
Pada keadaan ini sebaiknya terminasi bila janin sudah viable

HIPERTENSI DALAM KEHAMILAN


34. Kenapa diagnosis PEB à tergantung kasus
35. Bedanya PEB sama HT gestasional à dijelasin di nomor 36
36. Jenis Hipertensi dalam kehamilan
a) Hipertensi gestasional = tekanan darah > 140/90 mmHg yang ditemukan pertama kali
setelah midpregnancy (20 minggu) (Williams) dan menghilang sebelum 12 minggu pasca
persalinan.
b) Preeklampsia dan eclampsia. Preeklampsia = pregnancy specific syndrome that can affect
virtually every organ system. Eklampsia = kejang pada wanita dengan preeklampsia yang
tidak disebabkan oleh penyebab lain
c) Hipertensi kronik: ht yang muncul sebelum umur kehamilan 20 minggu atau ht yang
pertama muncul setelah usia 20 minggu DAN menetap sampai 12 minggu pasca persalian
d) Hipertensi kronik superimposed preeclampsia = wanita dengan hipertensi kronik yang
mengalami peningkatan tekanan darah (biasanya setelah 24 minggu) + new onset
proteinuria
37. Patofisiologi PEB
a. Kelainan vaskularisasi plasenta
Remodeling A. spiralis Abnormal.
Tidak terjadi invasi trofoblas ke dalam lapisan otot A. spiralis dan ke jaringan matriks sekitar
A. spiralis à lapisan otot tsb tetap kaku dan keras à vasokontriksi relative A. spiralis
(d=200mikron) à penurunan aliran darah uteroplasenta à hipoksia dan iskemia plasenta

b. Iskemik plasenta, radikal bebas, disfungsi endotel

Iskemik plasenta akan menghasilkan radikal bebas/oksidan yang merupakan senyawa penerima
elektron. Salah satu radikal bebas penting, yakni hidroksil. Hidroksil sangat toksik, dan
menyerang secara khusus terhadap membrane sel endotel sehingga menyebabkan “toxaemia”.
Selain itu, hidroksil juga merusak membrane sel yang banyak asam lemak tidak jenuh sehingga
menjadi peroksida lemak yang nantinya akan merusak membrane sel itu sendiri, merusak
nucleus dan protein sel endotel sehingga menyebabkan disfungsi endotel. Disfungsi endotel
dapat menyebabkan :
ü gangguan metabolism prostaglandin/PG (prostasiklin / PGE2 à vasodilator kuat)
ü agregasi trombosit pada endotel rusak à trombosit kemudian memproduksi
tromboksan à vasokonstriktor kuat
ü perubahan khas pada sel endotel kapiler glomerulus (glomerular capillary
endotheliosis)
ü peningkatan permeabilitas kapiler
ü peningkatan produksi endotelin (vasokonstriktor/vasopressor)
ü penurunan produksi NO (vasodilator)
ü peningkatan faktor koagulasi
c. teori imunologi

Ekspresi HLA-G mengalami penurunanà(Immune-maladaptation)à HLA-G berkurang di


desidua daerah plasenta à hambat invasi sel trofoblas ke dalam jaringan desidua ibu à
Remodeling A. spiralis Abnormal.
Fungsi HLA G:
ü modulasi respon imun sehingga si ibu tidak menolak hasil konsepsi (plasenta) yang
bersifat asing.
ü melindungi trofoblas janin dari lisis oleh NK cell ibu sehingga mempermudah invasi sel
trofoblas ke dalam jaringan desidua ibu. Jadi, HLA-G adalah faktor prakondisi invasi
trofoblas.

d. Adaptasi Kardiovskular
Terjadi gangguan metabolism PG-à Sehingga pembuluh darah kehilangan daya refrakter
terhadap vasopressor sehingga menjadi sangat peka.

e. Genetic
Ada faktor keturunan dan familial dengan model gen tunggal. Telah terbukti bahwa ibu dengan
pre-eklampsia 26% anak perempuannya akan mengalami pre-eklampsia pula, sedangkan hanya
8% anak menantu yang mengalami pre-eklampsia.

f. Diet defisiensi gizi


Konsumsi minyak ikan, minyak hati ikan halibut dapat mengurangi risiko à minyak ikan
mengandung banyak asam lemak tidak jenuh à hambat aktivasi trombosit à hambat produksi
tromboksan (vasokonstriktor kuat) à hambat vasokonstriksi
Defisiensi kalsium meningkatkan risiko à suplementasi kalsium perlu

g. Inflamasi
Faktor risiko dari proses inflamasi berlebihan adalah banyaknya sel trofoblas plasenta, misal
pada plasenta besar dam hamil ganda

38. Faktor risiko PE


• Nulipara (POGI: 3 kali lipat), primipaternitas
• Hiperplasentosis, misalnya mola hidatidosa, multifetal, diabetes mellitus, hidrops fetalis, bayi
besar
• Usia muda, nulipara. (Usia tua à hipertensi kronik superimposed preeklampsia)
• Jarak dengan kehamilan sebelumnya 10 tahun atau lebih (risiko hampir sama dengan
nulipara)
• Obesitas
• Multifetal
• Umur yang ekstrim
• Penyakit ginjal dan hipertensi yang sudah ada sebelum hamil
• Riwayat preeklampsia pada kehamilan sebelumnya (POGI: 7 kali)
• Riwayat keluarga preeklampsia (3 kali lipat)
Faktor risiko superimposed preeklampsia
• Riwayat preeklampsia
• Penyakit ginjal kronis
• Merokok
• Obesitas
• Diastol > 80 mmHg
• Sistol > 130 mmHg
39. Syarat PEB apa aja? komplikasinya?
Kriteria à minimal 1 dari tanda sbb:
a. Sistol >= 160 ATAU diastole >=110
b. Proteinuria 5g/24 jam atau 4+
pemeriksaan kualitatif
c. Kenaikan kadar kreatinin plasma
d. Gangguan visus serebral ; penurunan
kesadaran , nyeri kepala, scotoma,
pandangan kabur
e. Nyeri epigastrium atau kuadran atas
abdomen
f. Edema paru dan sianosis
g. Hemolysis mikroangiopati
h. Trombositopeni berat <100.000
sel/mm3
i. Peningkatan SGOT SGPT
j. IUGR
k. Sindroma HELLP

Komplikasi
a. Ibu : kejang, cerebral hemorrhage, DIC, trombositopeni, gagal ginjal, kerusakan hepar,
edema paru
b. Obstetric : insufisiensi uteroplasenta, solusio plasenta, premature, SC
c. Fetal : fetal distress, IUGR, oligohidramnion

40. Pada multipara denga riwayat PE, peningkatan risiko berapa kali ?
a. POGI à 7x
b. William à 40% di usia< 30 minggu, 23%< 37 minggu

41. Terapi PEB berdasarkan usia gestasi

PE: (PNPK)
Manajemen ekspektatif: pada kasus preeklampsia tanpa gejala berat dengan usia kehamilan
kurang dari 37 minggu dengan usia maternal dan janin yang lebih ketat (evaluasi gejala maternal
dan gerakan janin setiap hari oleh pasien, tekanan darah 2 kali dalam seminggu secara poliklinis,
jumlah trombosit dan fungsi liver setiap minggu, USG dan kesejahteraan janin berkala)
Manajemen aktif:
- Usia kehamilan ³ 37 minggu, atau
- Usia ³ 34 minggu dengan:
- Persalinan atau ketuban pecah
- Perburukan kondisi ibu dan janin
- Pertumbuhan janin terhambat
- Solusio plasenta
- Usg à AFI < 5cm
- BPP 6/10 atau kurang + persisten

PEB: (PNPK)
Manajemen ekspektatif: usia kehamilan 34 minggu dengan syarat ibu dan janin stabil.
Kortikosteroid + rawat inap direkomendasikan.
Manajemen aktif:
Terminasi kehamilan
Data maternal Data janin
Hipertensi berat tidak terkontrol Usia kehamilan 34 minggu
Gejala preeklampsia berat yang tidak berkurang Pertumbuhan janin terhambat
(nyeri kepala, pandangan kabur, dsb)
Penurunan fungsi ginjal progresif Oligohidramnion persisten
Trombositopenia persisten atau HELLP Profil biofisik < 4
syndrome
Edema paru Deselerasi variabel dan lambat pada NST
Eklampsia Doppler A. Umbilikalis: reversed end diastolic
flow
Solusio plasenta Kematian janin
Persalinan atau ketuban pecah Non viable fetus (< 14 minggu)

42. Terapi konservatif PEB kalo gagal apa tandanya?

Bila setelah diberi MgSO4 24 jam tidak ada perbaikan ke preeklampsia ringan à harus diterminasi
(aktif)
Komplikasi perawatan ekspektatif: (PNPK) à kortikosteroid, persalinan setelah 48 jam
- Gejala persisten
- Sindrom HELLP
- Pertumbuhan janin terhambat
- Severe oligohidramnion
- Reversed end diastolic flow
- Ketubah pecah atau inpartu
- Gangguan renal berat
43. Kalo ternyata TD meningkat terus mau tunggu sampe kapan? Tekanan darah naik atau ga
turun itu komplikasi? à jawaban di no 42, kalo kenaikan tekanan darah yang progresif hati2 itu
tanda impending eklamsia (nyeri kepala hebat / sakit kepala frontalis, gangguan visus, muntah-
muntah, nyeri epigastrium / RUQ, status mental terganggu, kenaikan PROGRESIF TD). Hipertensi
berat tidak terkontrol à salah 1 indikasi terminasi
44. Terapi konservatif kapan di stop dan diganti aktif à jawaban di no 41
45. Indikasi terminasi PEB? Batas terminasi? à jawaban di no 41 a. <= 36 6/7 à kortikosteroid à
Berat→ 34minggu
+
pematangan para
dalam 24-48 jam terminasi 37 minggu
Ringan →

46. Kalo PEB 34 minggu di terminasi ga? à tidak , tapi kalau memenuhi salah satu kriteria di no 41A
h
impending
à terminasi
47. Lasik/ furosemide boleh dikasih ke pEB ga?
Boleh pada edema paru, payah jantung kongestif, edema anasarca
48. Kalo pasien PEB udah aterm, mau diapain, pilih aktif atau konservatif? Kalo aktif mau PSP atau
SC?
a. Dapat di induksi lalu pervaginam dengan memperpendek kala II (forcep/vakum)
b. Kalo dr Yuma mikir SC kalo udah term dan bishop cuma 2 àBUCUR
49. Hipertensi gestasional kenapa diagnosis itu? à sesuai kasus dan onset. Hipertensi gestasional:
hipertensi yang timbul pada kehamilan tanpa disertai proteinuria dan hipertensi menghilang
setelah 3 bulan pascapersalinan atau kehamilan dengan tanda-tanda preeklampsia tetapi tanpa
proteinuria (buku merah)
50. Hipertensi gestasional: TXnya apa, obatnya apa? target t MAP 20.1 .

Antihipertensi diberikan pada preeklampsia dengan hipertensi berat, atau tekanan darah sistol ³
160 mmhg atau diastol ³ 110 mmHg. Targetnya sistol < 160 mmHg dan diastol < 110 mmHg.
Pilihan pertama: nifedipin oral short acting, hidralazine, labetalol parenteral. YANG GA BOLEH :
ACE-I (mis captropil), ARB (mis valsartan), Klortiazid

Metildopa
Agonis reseptor alfa kerja di SSP. Paling sering digunakan untuk hipertensi kronis
Safety margin paling luas à paling aman
Efek samping: letargi, mulut kering, mengantuk, depresi, hipertensi postural, anemia hemolitik,
drug induced hepatitis
Dosis: 2x250-500 mg PO dosis maksimum 2000 g/hari. Efek obat maksimal dalam 4-6 jam setelah
obat masuk, menetap selama 10-12 jam, ekskresi lewat ginjal
Dapat diekskresikan di ASI.

Nifedipin à GA ADA SEDIAAN IV


Menurunkan tekanan darah lebih cepat dibanding labetalol IV (1 jam setelah pemberian). Sbg
vasodilator arteriolar ginjal selektif, natriuretik, meningkatkan produksi urin
Regimen: 10 mg kapsul PO, diulang setiap 15-30 menit sampai MAP turun 20% dengan dosis
maksimum 120 mg/24 jam (PNPK: maks 30 mg).
Maintenance 3x10 mg PO (long-acting/Adalat OROS)
Penggunaan CCB berlebihan / sublingual bisa menyebabkan hipoksia janin dan asidosis
Untuk hipertensi kronis: 30-90 mg/hari

Nikardipin
CCB parenteral, menurunkan tekanan darah dengan efektif dalam 20 menit (lama kerja 4-6 jam).
Efek samping tersering: sakit kepala
Bekerja lebih selektif pada pembuluh darah miokard, efek samping takikardi lebih rendah drpd
nifedipin.
Dosis: awal 5 mg/jam, dititrasi 2.5 mg/jam tiap 5 menit hingga maksimum 10 mg/jam hingga
penurunan MAP rata-rata 25% tercapai. Dosis kemudian dikurangi dan disesuaikan dengan respon

51. Pasienmu umur berapa? Steril tidak? Kenapa tidak? Sterilisasi boleh digunakan intraop,
permintaan khusus dipertimbangkan
52. Dosis MgSO4, rate? Boleh ga 12 gram/12 jam? Sampe kapan dikasih?

IV (Williams)
Loading dose: 4-6 gram dalam 100 ml IV (RL, NaCl, D5%) diberikan dalam 15-20 menit (Williams)
Loading dose: 4 gram MgSO4 40% (berarti 10 cc) diberikan dalam 15-20 menit
Dicampur 10 cc RL à 10 cc MgSO4 + 10 cc RL = 20 cc dalam 20 menit
Berarti 1 cc dalam 1 menit à 20 tpm
Maintenance: 1-2 g/jam dalam 100 cc IV
1 gram/jam à 24 gram/24 jam
24 gram dalam 500 cc IV habis dalam 24 jam à 7 tpm (???)
atau 6 gram dalam 500 cc à 28 tpm
Stop 24 jam post partum atau post kejang
Sediaan: 25cc 40%
Syarat: 1) RR > 16x, 2) Ca glukonas tersedia, 3) UO > 0.5ml/kgBB/jam, 4) reflek patella +

Dosis menurut RCOG:


- Loading dose 4 gram selama 5-10 menit
- Maintenance 1-2 gram/jam selama 24 jam
- Diulang jika kejang berulang à 2 gram bolus perlahan dlm 15-20 menit
- Berulang lagi à diazepam 10 mg IV selama 2 menit

TTV, refleks patella, dan uo tiap jam

Mekanisme kerja mgso4: 1) vasodilatasi otot polos baik pembuluh darah perifer maupun uterus.
2) antikonvulsan à hambat reseptor NMDA di otak (teraktivasi saat asfiksia) à Ca2+ masu
neuron à kejang. 3) antihipertensi. 4) tokolitik

Pemberian mgso4 menurunkan PE, solusio plasenta, kematian, namun ES meningkat (rasa hangat,
flushing, nausea, muntah, kelemahan otot, ngantuk, iritasi lokasi injeksi à Ca glukonas 1 gram IV
(10 ml larutan 10%) bolus dalam 10 menit

53. Dosis dexamethasone?

2 x 6 mg IM dalam 2 hari à total dapet 24 mg. Betametason 1x12 mg IM 2 hari


Kortikosteroid à merangsang fibroblast paru untuk hasilin fibroblast pneumocyte factor à
stimulasi pneumosit tipe II untuk mensintesis phosphatidylcoline.
Dipalmitoyl phosphatidylcoline (lecithin) adalah major component of surfactant.

Cara melihat lung maturity: hitung L/S ratio (lecithin/sphingomyelin ratio) ß GOLD STANDARD
Lesitin dan spingomyelin merupakan komponen surfaktan. Sampai usia 32-33 minggu, kadar
lesitin dan spingomyelin kurang lebih sama, kemudian lesitin mulai naik sedangan spingomyelin
tetap
L/S ration < 2 à high risk of respiratory distress syndrome

Cara ambil sampel: amniocentesis and the sample is spun down in a centrifuge at 1000 rpm for
3–5 minutes. Thin layer chromatography (TLC) is performed on the supernatant, which separates
out the components. Lecithin and sphingomyelin are relatively easy to identify on TLC and the
predictive value of the test is good.

Cara lain: shake test (Clement’s)


Menilai kemampuan fosfolipid (lesitin & spingomielin) untuk support ring bubbles di permukaan
cairan amnion dan stabil selama 15 menit
Cairan amnion + NaCl + etanol à shake 15 detik à lihat setelah 15 menit
Hasil +: complete ring of bubbles at the meniscus à paru sudah matang à exclude kemungkinan
neonatal RDS
Kelebihan: mudah, on the spot result
54. Definisi eclampsia? Eclampsia adalah komplikasi dari hipertensi dalam kehamilan berupa kejang
menyeluruh tonik klonik, dan/atau koma. Ada tanda & gejala PE. Tidak ada kemungkinan
penyebab lain: epilepsy, meningitis, perdarahan subaraknoid)
55. Apa itu impending eclampsia? Kenapa ada pandangan kabur?
Tanda2 terjadinya eclampsia yaitu sakit kepala, gangguan penglihatan, nyeri ulu hati.
Gangguan penglihatan berupa: scotomata, pandangan kabur, diplopia, buta.
Patof:
• Iskemi, infark, ablasio retina (Purtscher retinopathy)
• Lesi di nucleus genikulatum lateral
• Perdarahan otak di oksipital à occipital blindness (amaurosis)
• Vasogenic edema di otak menekan N.II à pandangan kabur
56. Kalo pasien impending eclampsia masuk, kamu ngapain dulu?

Semua eklamsi/impending eklamsi harus diterminasi tanpa lihat usia kehamilan dan keadaan
janin.
• Antihipertensi kerja cepat: (ACOG)
- Nifedipine: 10-20 mg PO, diulang dalam 30 menit, maintenance 3x10 mg, dosis maks 120
mg/24 jam
- Hydralazine: loading dose 5 mg IV dilanjutkan 5-10 mg dengan interval 15-20 menit. Dosis
maks
- Labetalol: loading dose 10-20 mg IV dilanjutkan 2-80 mg setiap 20-30 menit maks 300 mg
• Pemberian MgSO4
• Dexamethasone jika <34 minggu
• Bila akan SC, transfusi jika trombosit <50.000/mm3.
57. Gimana bedain nyeri kepala karena impending eclampsia atau karena hipertensinya aja atau
karena mgso4?
- Jurnal OJOG: impending eklampsi paling banyak di frontal (71%) dan severe (50%), tidak
membaik dengan obat.
- Karena hipertensi: biasanya suborbital pressure, ada additional signs (sinus pressure,
aura, light sensitivity, irregular heartbeat)
- Karena mgso4: disertai early sign à nausea, feeling of warmth, flushing, somnolence,
double vision, slurred speech and weakness
58. Mau cek apa aja setelah SC-in impending eclampsia? Perlu cek asam urat? Buat apa?

PF: TTV, involusi uteri, perdarahan. Observasi karena eklampsi bisa terjadi sampai 24 jam
postpartus

PP:
Darah lengkap (Hb, Ht, trombosit, dll)
Fungsi hati (SGOT SGPT)
LDH à produk degradasi eritrosit
Fungsi ginjal (ureum, kreatinin, asam urat)
Asam urat meningkat karena penurunan GFR & peningkatan reabsorpsi di tubulus proksimal à
ekskresi asam urat berkurang

59. HELLP dari mana? Kriteria? Klasifikasi Tenessee dan Mississippi?

Klasifikasi Tennesse :
1. Komplit : trombosit < 100.000/µL
: LDH ≥ 600 IU/L
: SGOT ≥ 70 IU/L
2. Inkomplit : hanya terdapat 1 atau 2 tanda pada komplit
Klasifikasi Mississippi :
1. Kelas I : trombosit ≤ 50.000 mL
50.000 100 000
2. Kelas II : trombosit > 50.000 mL tapi ≤ 100.000 mL
-
→ .

3. Kelas III : tombosit >100.000 mL tapi ≤ 150.000 mL → 100 000


.
150 000
-
.

Disetai dengan hemolisis (LDH ≥600IU/L) dan disfungsi hepar (SGOT dan atau SGPT ≥40 IU/L)

60. Kenapa dikasi dexa untuk HELLP?


• Jika ada HELLP à segera rencanakan terminasi kehamilan. Jika kehamilan <34 minggu maka
diperlukan dexamethasone untuk pematangan paru
• Pemberian kortikosteroid pada sindrom HELLP dapat memperbaiki kadar trombosit, SGOT,
SGPT, LDH, tekanan darah arteri rata –rata dan produksi urin. (PNPK)
• Blunt the release of both antiangiogenic and inflammatory factors suggested to play role in
the pathophysiology of HELLP syndrome
61. Diperiksa LDH ga kalo HELLP?
• Periksa, untuk lihat hemolisis.
• LDH (Lactate Dehydrogenase) adalah enzim yg banyak terdapat di eritrosit. Jika hemolisis à
LDH naik
• LDH = enzim intrasel yang convert asam piruvat jadi asam laktat pada proses glikolisis.
Glikolisis adalah major energy pathway pada plasenta. Hipoksia yang terjadi pada
preeklampsia à tingkatkan glikolisis à LDH meningkat
• Peningkatan LDH :
lambangkan kerusakan dan
disfungsi seluler
62. Gimana cara ngecek hipertensi
kronis?

Anamnesis: hipertensi terjadi


sebelum kehamilan atau
sebelum usia kehamilan 20
minggu.
Terdapat tanda-tanda end organ
damage akibat hipertensi kronis

63. Kriteria eden pada PEB?

Eklampsia dikatakan berat apabila terdapat satu atau lebih kriteria dibawah ini:
• Koma ³ 6 jam
• Suhu ³ 39 oC
• Nadi > 120 x/menit
• Tekanan darah sistolik > 200 mmHg
• Anuria/oligouria
• RR > 40 x/menit
• Konvulsi > 10x
64. Syarat mgso4? Mgso4 sampai kapan? Tanda intoksikasi mgso4? Antidote (apa, dosis, cara
pemberian)

Syarat:
Refleks patella (+)
RR > 16 (tidak depresi napas)
UO > 0,5 cc/kg/jam atau ³ 25 cc/jam
Tersedia antidot Ca glukonas

Tanda intoksikasi:
Konsentrasi serum Manifestasi
(mg/dL)
1.5 – 3 Normal
4–6 Level terapi
5 – 10 Perubahan EKG
8 – 12 Refleks patella (-)
9 – 12 Flushing, merasa panas
10 – 12 Somnolen, bicara kacau
15 – 17 Paralisis otot, sulit bernafas
30 Cardiac arrest
Antidot:
Ca glukonas 10% 1 gram (10 cc) bolus dalam 15-20 menit

65. Kalo di tulis SC CITO, kenapa di SC CITO? SC cito jika ada gawat janin, ibu sudah eclampsia
66. Kenapa pro PSP? Karena eklamsi bukan indikasi absolut SC. SC bisa dilakukan jika ibu sudah
eklamsia, serviks tidak matang, ada tanda gawat janin.
67. Dosis nife dan metildopa
• Nifedipine (tidak boleh SL): Initial 10 mg PO, boleh diulang tiap 30 menit sampai MAP turun
20% (max 120 mg/24 jam). Maintenance 3x10 mg
• Metildopa: 250-500 mg PO 2-3x sehari dosis maksimum 3 g / hari
68. Terminasi dengan induksi oksitoksin untuk PEB apakah adekuat?

Adekuat.
Drip oxytocin:
0,5 ml oxy (5 IU) dilarutkan dalam 500 ml RL à 10 mIU/ml
dosis: 4 mIU/menit maka 4/10 x 20tpm = 8 tpm Start dose

Tiap 15 menit, tambah 4 mIU (8 tpm)


maksimal 20 mIU/ml (40 tpm) (buku merah) = atau 39 mIU/ml (72 tpm) (williams)

69. Pilihan obat antihipertensi? Dosis? Kenapa?


• Nifedipine (tidak boleh sublingual) à short acting. Initial 10 mg PO, boleh diulang tiap 30
menit sampai MAP turun 20% (max 120 mg/24 jam). Maintenance 3 x 10 mg
• Metildopa à long acting. Maintenance 3 x 500 mg PO (max 3 gram)
• Nikardipine Jika TD ≥180/110 à 1 ampul 10 mg dalam 50 cc pelarut per jam (tidak boleh RL
UHT
emergency
dan Na bikarbonat. MIMS: 5 mg/jam via slow infusion maks 15 mg/jam, dikurangi 3 mg/jam
setelah sampe target
CEPHALOPELVIC DISPROPORTION
70. Trial of labor dan test of labor untuk CPD?
• Trial of labor pada nullipara yaitu dengan adanya kontraksi uterus yang kuat dan augmentasi
oksitosin jika diperlukan akan menyebabkan penurunan dan kelahiran kepala bayi. Jika gagal
à CPD.
• CPD lebih jarang pada wanita multipara yang sebelumnya PSP normal. CPD pada wanita
multipara dapat terjadi jika bayi pada kehamilan sekarang jauh lebih besar atau ada malposisi.
• Trial of labor pada multipara masih meragukan karena risiko tuptur uteri jika memang ada
CPD dan diinduksi dengan oxytocin. CPD dapat didiangosis jika molase terus bertambah
sementara tidak ada penurunan kepala bayi ke pelvis.
• Kondisi yang dapat meningkatkan risiko ruptur uteri/TOL gagal: insisi SC sebelumnya
klasik/vertikal, riwayat SC > 1, SC sebelumnya < 18 bulan, BMI > 40, berat fetus > 4 kg
71. Pelvimetri untuk CPD

Normalnya:
• Sudut arcus pubis >90°
*
• Linea inominata teraba 2/3
• Dinding pelvis
• Spina ischiadica runcing (lebih baik daripada tumpul karena kalau tumpul bidang geseran yang
harus dilewati kepala janin lebih luas daripada yang runcing sechingga perlu tenaga dan waktu
yang lebih)
• Distansia spinarum >10 cm
• Sacrum konkaf
• Konjugata diagonalis >11,5 cm
• Konjugata vera (pinggir atas simfisis ke promontorium) > 11 cm
• Distansia intertuberosum ≥10 cm
72. Penilaian panggul

PINTU ATAS PANGGUL (INLET)


• PAP kecil jika diameter
anteroposterior <10 cm (BPD bayi
9,5-9,8 cm) atau diameter
transverse <12 cm.
• Diameter anteroposterior bisa
diukur dengan konjugata diagonal
(yang lebih besar 1,5 cm).
• Maka diameter anteroposterior
PAP kecil jika konjugata diagonal
<11,5 cm.

BAGIAN TENGAH PANGGUL (MIDPELVIS)


Normalnya:
• Transverse (distansia spinarum) 10,5 cm
• Anteroposterior 11,5 cm
• Posterior sagittal (midpoint distansia spinarum ke sacrum) 5 cm

Suspek bag tengah panggul kecil jika:


• Distansia spinarum + posterior sagittal ≤ 13,5 cm (normalnya 15,5 cm)
• Distansia spinarum < 10 cm à suspek
Distansia spinarum <8 cm à kecil
• Spina ischiadica menonjol, dinding pelvis bertemu
• Distansia intertuberosum kecil (normalnya ≥10 cm)

PINTU BAWAH PANGGUL (OUTLET)


• Distansia intertuberosum ≤8 cm
• Arkus pubis sempit <90° à kepala bayi tidak turun langsung ke bawah simfisis pubis tapi ke rami
ischiopubic à perineum terdistensi, laserasi

73. Klasifikasi CPD (Derajat/kategori)?

Absolute CPD – true mechanical obstruction


Permanent (maternal):
• Kelainan bentuk tulang panggul
• Pelvis sempit (paling sering)
• Pelvic exostoses (exostoses: pembentukan tulang baru)
• Spondylolisthesis (vertebra bergeser maju ke tulang di
bawahnya)
• Anterior sacrococcygeal tumors

Temporary (fetal):
• Hydrocephalus
• Large infant (postmature, DM)

Relative CPD
• Brow presentation
• Face presentation – mentoposterior
• Occipitoposterior positions
• Deflexed head

Grading molase
1 tulang terpisah, sutura teraba
2 sutura menutup, tidak overlap
3 tulang overlap, bisa dipisahkan
4 tulang overlap, tidak bisa dipisahkan
Derajat
Derajat 1 kepala bayi di bagian bawah simfisis pubis
Derajat 2 kepala bayi di bagian atas simfisis pubis
Derajat 3 kepala bayi ga nyentuh bagian atas simfisis pubis

74. Faktor risiko CPD (ibu dan bayi)

Ibu:
• Nullipara
• tubuh kecil à panggul sempit
tinggi badan <145 cm
• riwayat fraktur pelvis, malalignment
• diabetes mellitus à bayi makrosomia
Bayi:
• kepala besar
• malposisi (asinklitismus, occiput posterior, presentasi muka, alis)

75. Dasar diagnosis CPD? (tanda klinis: inspeksi abdomen, leopold, Osborn & Mueller-monrokerr,
usg, pelvimetri)
• Leopold: Kepala melayang di atas PAP atau di lateral salah satu fossa iliaca.
• TFU à TBJ besar
• Osborn: Pemeriksa mendorong kepala bayi ke pelvis dengan tangan kiri. Tangan kanan
diletakkan di simfisis pubis. Jika CPD, tidak terjadi penurunan à kepala overlap dengan
simfisis à tangan kanan terangkat.
• Mueller munro kerr: Pemeriksa mendorong kepala bayi ke pelvis dengan tangan kiri.
Tangan kanan melakukan VT dan menentukan seberapa jauh kepala mengikuti tekanan
tersebut.
• Terjadi molase dari kepala bayi dan gagal penurunan janin
• Pelvimetri
• USG, Xray, MRI à pelvis kecil, bayi lebih besar

76. Kenapa CPD di-SC, emang ga bisa spontan?

Mencoba induksi melalui PSP terlebih dahulu dengan persiapan untuk SC bila gagal

77. Ukuran conjugate vera berapa?


• Konjugata vera = konjugata diagonalis – 1,5 cm
• Menurutku harusnya jawabannya 10 cm. karena BPD bayi biasanya 9,5 – 9,8. Konjugata vera
= pintu PAP
• Tapi kisi2 nya jawabannya 9,5 cm. terserah iman masing2 ya

78. CPD pada inpartu tanda-tandanya apa?


• KPD karena kepala bayi ketahan di PAP, gaya tekan dari kepala bayi memecah selaput ketuban
tapi bayi tidak turun.
• Molase terus bertambah tetapi Kepala bayi tidak turun.
• Dilatasi serviks sangat pelan atau tidak ada, statis selama >2 jam.
• Pemanjangan kala I

MIOMA UTERI

79. Apa itu mioma?


• Mioma uteri adalah tumor jinak dari otot polos rahim (myometrium).
• Pertumbuhan myometrium yang abnormal oleh adanya hiperresponsifitas reseptor progesterone
dan kadar progesterone yang tinggi.
• Mioma berwarna pucat, relatif bulat, kenyal, berdinding licin, bila dibelah bagian dalamnya
menonjol sehingga mengesankan permukaan luarnya adalah kapsul.

80. Jenis dan sifat?

81. Mioma yang mana yang bisa bikin perdarahan? Kenapa bisa berdarah? [submucosa, 3 alasan]
Submucosa, karena:
• Permukaan uterus meningkat à vaskularisasi meningkat
• Disregulasi dari angiogenic growth factor dan reseptornya (fibroblast growth factor, vascular
endothelial growth factor, heparin-binding epidermal growth factor, platelet-derived growth
factor, transforming growth factor-beta, parathyroid hormone-related protein and prolactin) à
pembuluh darah meningkat, blood flow ke myoma meningkat
• Kontraktilitas uterus menurun
• Ulserasi endometrium
• Kompresi plexus vena di myometrium à kongesti di myometrium & endometrium.

82. Etiologi? Gejala?


Hormone estrogen dan progesteron menjadi precursor pertumbuhan mioma. konsentrasi reseptor
estrogen dalam jaringan mioma lebih tinggi dibanding myometrium sekitarnya, namun masih lebih rendah
dibandingkan dengan endometrium. Kedua hormone tersebut membuat growth hormone tinggi sehingga
terjadi pertumbuhan myoma
Mioma jarang ditemukan sebelum usia reproduktif, dan hanya bermanifestasi selama usia reproduktif.
Mioma tumbuh cepat saat pasien hamil atau terpapar estrogen, mengecil atau hilang setelah menopause.

Gejala:
a. Heavy menstrual bleeding (Menorrhagia)
akibat adanya disregulasi local vasoactive growth factor à vasodilatasi endometrial venules
à sehingga pada saat mens darah yang keluar lebih banyak
b. Pressure
Adanya pembesaran uterus à menimbulkan rasa tertekan, inkontinensia, konstipasi dan
urinary frekuensi à obstruksi berlanjut à hidronefrosis
c. Nyeri
Dapat terjadi dysmenorrhea, dyspareunia, acute pain à akibat adanya degenerasi (merah)
dan tissue nekrosis ataupun karena torsi pada myoma yang pedunculated

d. Infertilitas
- Adanya oklusi pada ostium tuba dan gangguan pada kontraksi uterus à mengganggu
implantasi dan transpor sperma
- Berhubungan dengan inflamasi endometrial dan perubahan vaskular à ganggu implantasi
e. Myomatous erythrocytosis syndrome
Ditandai dengan produksi eritropoetin yang berlebihan oleh ginjal dan leiomyoma sendiri
f. Pseudo-Meigs syndrome (asites & efusi pleura)
Asites disebabkan:
• Iritasi peritoneum oleh mioma à produksi cairan peritoneum berlebih
• Penekanan mioma terhadap limfa à menghambat drainase
• Mioma memproduksi mediator inflamasi à permeabilitas kapiler meningkat
Efusi pleura disebabkan cairan asites ditransfer via kanal lima transdiafragmatik.

83. Wandering mioma atau parasitic mioma?


Mioma subserosa bertangkai yang lepas dan menempel ke jaringan atau organ sekitarnya dan menjadi
myoma di tempat tersebut

84. Mioma geburt? Bedanya sama polip? Mioma geburt tatalaksana?


Mioma geburt merupakan tumor jinak pada myometrium yang bertangkai dan kemudian tumor tesebut
melewati atau dilahirkan melalui saluran serviks
Langkah :
1. Pasien diberikan anestesi yang sesuai dan diposisikan dalam posisi litotomi
2. Kandung kemih dikosongkan
3. Vagina dibuka dengan menggunakan spekulum
4. Leiomyoma yang prolaps dijepit dengan tenakulum dan tangkainya diligasi
5. Kemudian bagian tangkai diinsisi dengan electrosurgical atau dapat dipelintir dan kemudian ditarik
6. Myoma dikeluarkan

Perbedaan polip dan mioma


Polip Mioma
Asal Endometrium Miometrium
Ukuran Kecil Kecil – sangat besar
Potensi keganasan + -
Post menopause Bisa ada pre/postmenopause Mengecil/hilang

85. spie….. ligasi potong tangkainya dengan cauter


86. efek mioma pada kehamilan?
Early pregnancy
• Abortus spontan
Terutama terjadi jika myoma terletak di korpus uterus dibandingkan jika terletak di segmen
bawah rahim. Myoma jenis submucosa dan intramural juga meningkatkan risiko abortus spontan.
Mekanisme: kontraktilitas uterus meningkat, efek kompresi dari myoma, gangguan vaskularisasi
terhadap plasenta.
• Perdarahan ante partum
Terjadi jika implantasi plasenta berdekatan dengan mioma.

Late pregnancy
• Persalinan premature
Faktor risiko tinggi: myoma multiple dan myoma yang berkontak dengan plasenta.
PPROM tidak terjadi
• Abruptio placenta
Faktor risiko: myoma submucosa, myoma retroplasenta, volume myoma > 200 cm3.
Mekanisme: aliran darah ke myoma dan jaringan sekitar menurun à iskemia dan nekrosis
desidua di plasenta yang menyelimuti myoma
• Placenta previa
• Fetal anomaly
(jarang terjadi) Myoma yang sangat besar dapat menekan bayi à deformitas pada fetus seperti
dolicocephaly (kompresi lateral pada tengkorak bayi), torticollis (abnormal twisting of the neck).
IUGR tidak terjadi.

Intrapartum
• Malpresentasi, distosia, SC
Faktor risiko malpresentasi: myoma besar, multiple myoma, myoma di SBR.
Faktor risiko SC: malpresentasi, myoma besar, multiple myoma, myoma submucosa, myoma di
SBR.
• Post partum haemorrhage
Myoma mengganggu arsitektur dan kontraksi uterus à atonia uteri à PPH à risiko histerektomi
puerperal.
• Retained placenta

87. tatalaksana mioma? medikasi dan operatif


• Bed rest
• Hidrasi
• Analgesic
Penggunaan NSAID > 48 jam pada trimester 3 dapat meningkatkan terjadinya penutupan
premature dari fetal ductus arteriosus, hipertensi pulmonal, enterocolitis, perdarahan
intrakranial, oligohidramnion.
Jika nyeri tak tertahan à analgesic narkotik, analgesic epidural, miomektomi.

Operatif:
• Miomektomi
Jika mioma sangat sakit (terutama subserosa atau bertangkai), sangat besar atau bertumbuh
cepat, atau > 5 cm pada SBR à miomektomi dapat dilakukan di trimester 1 atau 2. Risiko SC akan
meningkat karena ditakutkan terjadi rupture uteri jika pernah dilakukan miomektomi.
Hindari melakukan miomektomi bersamaan SC untuk menghindari perdarahan hebat.
Miomektomi saat SC boleh dilakukan jika untuk kepentingan kelahiran bayi.
• Embolisasi arteri uterine
EAU bersamaan dengan SC efektif untu mengurangi PPH dan mengurangi risiko
miomektomi/histerektomi karena dapat mengecilkan ukuran myoma.
• Histerektomi
• Myolisis

88. apa itu myolisis?


Sebuah cara untuk menghancurkan myoma dengan listrik radiofrekuensi, supercooled cryoprobes,
focused ultrasound yang dimonitor dengan MRI. Pada thermomyolisis dan cryomyolisis, akses ke jaringan
melalui laparoskopi atau histeroskopi.

89. lesi pra kanker, menurut Bethesda? Singkatan ASCUS?


SQUAMOUS CELL
• Atypical squamous cells
- of undetermined significance (ASC-US)
- cannot exclude HSIL (ASC-H)
• Low grade squamous intraepithelial lesion (LSIL)
- encompassing HPV/mild dysplasia?CIN 1
• High grade squamous intraepithelial lesion (HSIL)
- encompassing : moderate and severe dysplasia/CIN 2/CIN 3/ CIS
- with features suspicious for invasion (if invasion suspected)
• Squamous cell carcinoma
GLANDULAR CELL
• Atypical glandular cells (AGC)
- specify endocervical, endometrial, or not otherwise specified
• Atypical glandular cells, favor neoplastic
- specify endocervical or not otherwise specified
• Endocervial adenocarcinoma in situ (AIS)
• Adenocarcinoma

90. Kamu tau hyperplasia pra-kanker


untuk? Ada berapa macam kanker
untuk hyperplasia?

Mengetahui staging dari CIN untuk


menentukan terapinya.

91. Cara miomektomi? Kapan


dilakukan miomektomi? Indikasi
miomektomi?
Cara miomektomi
1) Pasien diberikan anestesi yang sesuai
2) Pasien dalam posisi terlentang, pasang kateter, setelah dilakukan tindakan aseptik-antiseptik di
daerah abdomen. Lapangan operasi dipersempit dengan memasang doek steril
3) Buat insisi pfannenstiel jika ukuran uterus < 14 minggu gestasi atau insisi vertikal jika uterus lebih
besar, dimulai dari kulit lapis demi lapis hingga peritoneum
4) Tampak uterus dan identifikasi leiomyoma, palpasi dapat membantu mengetahui jenis myoma
(intramural atau submukosa)
5) Dapat digunakan uterine tourniquet ataupun injeksi vasopresin untuk mengungari perdarahan
6) Insisi serosa di bagian anterior (mengurangi resiko adhesi)
7) Leiomyoma dijepit dengan tenakulum, kemudian ditarik keluar menjauhi insisi, dilakukan diseksi
tajam dan tumpul pada pseudokapsul disekitar leiomyoma untuk membebaskan tumor dari
myometrium disekitarnya
8) Setelah leiomyoma dikeluarkan dapat terjadi perdarahan yang banyak sehingga dilakukan ligasi
sebelum dilakukan eksisi
9) Jika insisi yang dilakukan mencapai cavum endometrium, maka dilakukan penjahitan menggunakan
benang delayed absorbable 4.0 atau 5.0
10) Myometrium ditutup dengan beberapa layer untuk mengurangi perdarahan dan hematoma,
digunakan benang delayed absorable 2.0 – 0
11) Serosa ditutup dengan menggunakan running baseball stitch atau subserosal running closure (mirip
subkutikuler) menggunakan benang monofilamen delayed absorbable 4.0-5.0
12) Dilakukan pembilasan pada rongga abdomen dengan normal saline
13) Setelah diyakini tidak ada perdarahan, luka operasi dijahit lapis demi lapis

Indikasi myomectomy :
- massa pelvis yang membesar dengan cepat
- perdarahan yang persisten
- nyeri atau rasa tertekan
- myoma uteri yang asimptomatis dgn ukuran > 8 cm
- wanita ingin hamil
Indikasi histerektomi :
- myoma uteri yang asimptomatis terpalpasi per abdominal
- perdarahan yang hebat à perdarahan yang disertai dengan gumpalan darah > 8 hari, ada anemia
- adanya pelvic discomfort yang akut dan berat, nyeri kronik pada low back, penekanan pada bladder
sehingga terdapat urinary frekuensi dan dapat menyebabkan UTI

ANTEPARTUM HEMORRHAGE
92. Ada apa aja?
Early pregnancy
• Abortus
• KET
• Mola hidatidosa

Late pregnancy
• Plasenta previa
• Solusio plasenta
• Vasa previa

+ gyn = carcinoma (serviks), trauma


PLASENTA PREVIA
93. dasar diagnosis plasenta previa? etiologi, faktor risiko, komplikasi?
Dasar diagnosis
• Anam: perdarahan tanpa disertai nyeri
• PF: pemeriksaan forniks teraba bantalan. VT hanya boleh dilakukan pada double set up
exam/PDMO.
• PP: USG

Etiologi
• False drop à jatuh dan nempel di segmen bawah rahim
• Late migration /tidak terjadi migrasi (trofoblasnism)
o Pembentukkan SBR. Minggu ke-20 memanjang 0.5 cm, aterm jadi >5cm
o Placenta-free uterine wall bertumbuh lebih cepat dibanding dinding uterus yang tertutupi
plasenta
o Trophotropism à pertumbuhan trofoblas menjauh dari OUI menuju fundus yang punya
aliran darah yg lebih banyak
• Kerusakan endometrium (mis. Riwayat SC, kuretase, multiparitas)
• Hiperplasentosis (mis. Gemelli, merokok)

Faktor risiko
• Usia ibu >30 tahun
• Multiparitas
• Riwayat operasi rahim/kuret
• Gemelli
• Merokok

Komplikasi
• PPH (karena SBR rapuh dan banyak pembuluh darah, juga kurangnya elemen otot di SBR sehingga
tidak bisa kontrakasi dengan adekuat)
• plasenta adhesive (karena SBR tipis jadi trofoblas bisa invasi dalam à plasenta akreta, inkreta,
perkreta)
• Solusio plasenta
• Kelainan letak janin & prematuritas

94. persentase plasenta previa di indonesia?


• Insiden di RS pemerintah 1,7 – 2,9%.
• Insiden di negara maju <1%.

95. kenapa bisa berdarah?


Pada kehamilan lanjut, ismus uteri melebar menjadi SBR, cervix mendatar & dilatasi (biasa lebih dari
4cm)à plasenta yang berimplantasi mulai terlepas à di SBR banyak pembuluh darah, dan SBR tidak bisa
kontraksi dengan adekuat à painless bleeding keluar rahim, tidak membentuk hematoma retroplasenta.

96. tatalaksana plasenta previa?


• Jika >37 minggu, ada/tidaknya tanda gawat janin, ibu perdarahan massif, tanda in partu à
terminasi
• Perawatan konservatif à resusitasi, tokolitik
Jika <34 minggu, beri dexamethasone
97. patofisiologi plasenta previa?
Adanya faktor-faktor False drop, Late migration, Kerusakan endometrium (mis. Riwayat SC, kuretase,
multiparitas), Hiperplasentosis (mis. Gemelli, merokok) menyebabkan plasenta berimplantasi di segmen
bawah rahim yang banyak pembuluh darah & tidak banyak berotot. Menjelang kehamilan aterm, terjadi
mekanisme perdarahan (no.95). kompllikasi yang sering terjadi yaitu plasenta adhesive.

98. kenapa dikasi asam tranexamat?


As tranexamat adalah agen antifibrinolitik, mencegah breakdown fibrin clots dengan cara block binding
plasminogen & plasmin ke fibrin. Maka diberikan as tranxamat agar tidak terjadi perdarahan massif.

99. jenis plasenta previa, yang mana yang boleh PSP?


Klasifikasi NIH 2013:
1. true placenta previa: OUI tertutup plasenta
2. Low-lying placenta: <= 2 cm dari OUI

• Type IV : PP totalis/komplit à menutupi seluruh OUE


• Type III : PP parsialis à menutupi sebagian OUE (menutupi OUE yg blm dilatasi, tapi partial bila
sudah berdilatasi)
• Type II : PP marginalis à berada di pinggir OUE (bisa PSP khusus yang anterior)
• Type I : PP low lying à berjarak ≤2 cm dari OUE (bisa PSP)

Mild degree : type I & II


Major degree : type III & IV

100. apa itu double set up examination (PDMO)? PDMO = pemeriksaan dalam meja operasi. Dilakukan
jika USG tidak tersedia. Pemeriksa melakukan VT di meja operasi. Jika PP marginalis/low lying à kirim ke
kamar bersalin untuk PSP. Jika PP totalis/parsial à segera SC.
Untuk apa PDMO? [liat jalan lahir]
Why? kalau berdarah saat di VT bisa langsung SC.
Kontraindikasi PDMO? [perdarahan hebat]

101. plasenta previa jenis mana yang bisa di VT? kapan boleh VT?
Kalau tidak ada perdarahan dan bukan yang tipe totalis
102. kalo plasenta previa totalis mau diapain? Pro SC
103. faktor risiko plasenta previa
• Usia ibu >30 tahun
• Multiparitas
• Riwayat operasi rahim/kuret
• Gemelli
• Merokok

104. komplikasi plasenta previa


• PPH (karena SBR rapuh dan banyak pembuluh darah, juga kurangnya elemen otot di SBR sehingga
tidak bisa kontrakasi dengan adekuat)
• plasenta adhesive (karena SBR tipis jadi trofoblas bisa invasi dalam à plasenta akreta, inkreta,
perkreta)
• Solusio plasenta
• Kelainan letak janin & prematuritas

105. mana yang lebih bahaya, PP anterior atau posterior?


anterior lebih tipis jadi risiko akreta lebih besar, posterior bisa kompresi plasenta dan umbilical cord, jadi
asfiksi. Anterior lebih nyebabin early bleeding, premature dan bbl
Dangerous placenta previa: type II posterior PP
1. Terletak pada curved birth canal dan promontorium à memperkecil diameter AP pelvic inlet à
tidak terjadi engagement à mengganggu kompresi separasi plasenta yg efektif u/ menghentikan
perdarahan
2. Plasenta tertekan bila persalinan pervaginam

Risiko kompresi tali pusat atau prolapse tali pusat

106. solusio plasenta: gejala, berdasarkan derajat, penyebab?


Gejala: perdarahan pervaginam disertai rasa nyeri pada perut, uterus yang keras, abnormalitas FHR
Penyebab : terdapat perdarahan retroplasenta
Grading

Class 1 characteristics include the following:

• No vaginal bleeding to mild vaginal bleeding


• Slightly tender uterus
• Normal maternal BP and heart rate
• No coagulopathy
• No fetal distress

Class 2 characteristics include the following:

• No vaginal bleeding to moderate vaginal bleeding


• Moderate to severe uterine tenderness with possible tetanic contractions
• Maternal tachycardia with orthostatic changes in BP and heart rate
• Fetal distress
• Hypofibrinogenemia (ie, 50-250 mg/dL)

Class 3 characteristics include the following:

• No vaginal bleeding to heavy vaginal bleeding


• Very painful tetanic uterus
• Maternal shock
• Hypofibrinogenemia (ie, < 150 mg/dL)
• Coagulopathy
• Fetal death

FR: merokok, penggunaan kokain, usia lebih dari 35, hipertensi, multigestasi, trauma
107. 28-29 minggu kenapa ballottement (+)?
Ballottement merupakan maneuver untuk mendeteksi objek yang floating pada tubuh. The use of a
finger to push sharply against the uterus and detect the presence or position of a fetus by its return
impact.
108. Apa kepentingan tau presentasi kepala pada plasenta previa?
Pada pemeriksaan dalam dilakuin buat melihat tipe apakah plasenta previanya. Apabila teraba seperti
bantalan maka itu merupakan plasenta. Lalu apabila sudah menemukan tipe plasenta, maka keluarkan
tangan untuk melihat apakah ada perdarahan yang keluar dari cerviks. Kalo ada keluar darah bisa
disuction. Kalo udah dilatasi lengkap dan merasakan presentasi kepala bisa di amniotomy dan keluarin
bayinya. Yang bisa dikeluarin hanya mild type (tipe low lying dan antiror marginalis)

EARLY APH
109. Perdarahan early: Mola, Abortus, KET à definisi, klasifikasi, etiologi, faktor risiko
MOLA
• Definisi: kehamilan abnormal
dimana terjadi abnormalitas
proliferasi trofoblas, disertai
degenarasi kistik vili dan
gagalnya pembentukan janin.
• Klasifikasi:
• Penyebab: abnormalitas
fertilisasi à abnormalitas
kromosom
Mola komplit disebabkan oleh
fertilisasi antara sperma
haploid (23 X) dan telur
haploid yang kosong (gennya
mengalami inaktivasi) à hasil
fertilisasi 46 XX akibat duplikasi kromosom ayah
saja.
Mola parsial disebabkan oleh fertilisasi antara dua
sperma haploid (23 X atau 23 Y) atau dispermi dan
telur haploid (23 X) yang gennya tidak mengalami
inaktivasi à hasil fertilisasi triploid 69 XXX atau 69
XXY dengan 2 set kromosom berasal dari ayah.
• Faktor Risiko:
• Kehamilan di usia terlalu muda (<16
tahun) atau tua (>35 tahun)
• Riwayat kehamilan mola sebelumnya
• Beberapa penelitian menunjukkan penggunaan kontrasepsi oral
• Merokok
• Golongan darah A atau AB
• Defisiensi kadar beta karoten
ABORTUS
• Definisi: terminasi kehamilan sebelum usia kehamilan mencapai 20 minggu atau dengan
berat janin <500 gram
• Klasifikasi abortus:
o Abortus Iminens: abortus pada tingkat permulaan, merupakan ancaman terjadinya
abortus, ditandai perdarahan pervaginam, ostium uteri masih tertutup, dan hasil
konsepsi masih baik dalam kandungan
o Abortus Insipiens: abortus yang sedang mengancam yang ditandai dengan serviks
telah mendatar dan ostium uteri telah membuka, akan tetapi hasil konsepsi masih
dalam kavum uteri dan dalam proses pengeluaran
o Abortus Inkomplit: sebagian hasil konsepsi telah keluar dari kavum uteri dan masih
ada yang tertinggal
o Abortus Komplit: seluruh hasil konsepsi telah keluar dari kavum uteri pada
kehamilan kurang dari 20 minggu atau berat janin kurang dari 500 gram
o Missed Abortion: abortus yang ditandai dengan embrio atau fetus yang telah
meninggal dalam kandungan sebelum kehamilan 20 minggu dan hasil konsepsi
seluruhnya masih tertahan dalam kandungan
o Abortus Septik: abortus yang ditandai infeksi genital
• Penyebab dan faktor risiko:
• Janin (Fetal): Kelainan genetik (kromosom) à paling sering
• Ibu (Maternal):
- Infeksi
- Kelainan hormonal: hipotiroidisme, diabetes mellitus, insufisiensi
progesteron
- Malnutrisi
- Penggunaan obat-obatan, merokok, konsumsi alkohol
- Faktor imunologis: antiphospolipid syndrome, perbedaan rhesus
- Defek anatomis seperti uterus didelfis, inkompetensia serviks, dan
sinekhiae uteri karena sindom Asherman
- Kelainan fungsi koagulasi darah
• Ayah (Paternal): Kelainan sperma
KET
• Definisi: kehamilan yang pertumbuhan sel telur yang telah dibuahi tidak menempel pada
dinding endometrium kavum uteri
• Klasifikasi:
o Kehamilan tuba, meliputi > 95% yang terdiri atas kehamilan pada pars ampularis
(55%), pars ismika (25%), pars fimbriae (17%), dan pars interstitialis (2%)
o Kehamilan ektopik lain (< 5%) antara lain terjadi di serviks uterus, ovarium, atau
abdominal. Untuk kehamilan abdominal lebih sering merupakan kehamilan
abdominal sekunder di mana semula merupakan kehamilan tuba yang kemudian
abortus dan meluncur ke abdomen dari ostium tuba pars abdominalis (abortus
tubaria) yang kemudian embrio/buah kehamilan mengalami reimplantasi di kavum
abdomen, misalnya di mesenterium/mesovarium atau di omentum.
o Kehamilan intraligamenter, jumlahnya sangat sedikit
o Kehamilan heterotopik, merupakan kehamilan ganda di mana satu janin berada di
kavum uteri sedangkan yang lain merupakan kehamilan ektopik. Kejadian sekitar
satu per 15.000 – 40.000 kehamilan.
o Kehamilan ektopik bilateral

• Penyebab: implantasi janin di luar kavum uteri


• Faktor Risiko:
o Faktor Tuba
§ Adanya peradangan atau infeksi pada tuba menyebabkan lumen tuba
menyempit atau buntu
§ Keadaan uterus yang mengalami hypoplasia dan saluran tuba yang
berkelokkelok panjang dapat menyebabkan fungsi silia tuba tidak berfungsi
dengan baik. Pada keadaan pascaoperasi rekanalisasi tuba dapat juga
merupakan predisposisi terjadinya kehamilan ektopik
§ Endometriosis tuba atau divertikel saluran tuba yang bersifat kongenital
§ Tumor di sekitar tuba yang menyebabkan perubahan bentuk dan patensi
tuba
o Fator abnormalitas dari zigot
§ Zigot yang tumbuh terlalu cepat atau tumbuh dengan ukuran besar akan
tersumbat dan berhent di saluran tuba
o Faktor Ovarium
§ Produksi ovum yang ditangkap oleh tuba kontralateral membutuhkan
proses khusus dan waktu yang lebih panjang sehingga kemungkinan
terjadinya kehamilan ektopik lebih besar
o Faktor hormonal
§ Pil KB yang mengandung progesterone dapat mengakibatkan gerakan tuba
melambat
o Faktor lain
§ Pemakaian IUD, peradangan yang timbul pada endometrium dan
endosalfing, dan perokok
110. Perdarahan late: Plasenta previa, solutio plasenta, vasa previa

VASA PREVIA
• Passage blood vessel melewati os internum di bawah presentasi janin. Perdarahan terjadi
apabila selaput ketuban melewati pembukaan serviks robek atau pecah dan vascular janin
ikut terputus. Blood vessel berada diantara amnion dan korion dan tidak terlindungi
Wharton jelly.
• Lakukan APT test (Alkali Denaturation Test): campur specimen darah janin dengan NaOH
(Sodium hidroksida). Darah janin berwarna pink, darah ibu berwarna kuning kecoklatan.
• Faktor risiko: plasenta bilobate, plasenta suksenturia, plasenta letak rendah, kehamilan
fertilisasi dengan in vitro, kehamilan ganda
111. Beda plasenta previa dan solutio plasenta
Plasenta Previa Solutio Plasenta

Painless Nyeri

Darah merah segar Darah merah kehitaman

Uterus rileks Uterus teraba keras

Biasanya adhesive Plasenta copot

112. Grading solutio plasenta


• Solusio plasenta ringan
o Luas plasenta yang terlepas <25%
o Jumlah darah yang keluar <250 ml
o Komplikasi terhadap ibu dan janin belum ada
• Solusio plasenta sedang
o Luas plasenta yang terlepas 25-50%
o Jumlah darah yang keluar 250-1000 ml
o Gejala: nyeri pada perut yang terus menerus, denyut jantung janin menjadi cepat,
hipotensi dan takikardi.
• Solusio plasenta berat
o Luas plasenta yang terlepas >50%
o Jumlah darah yang keluar >1000 ml
o Gejala makin jelas, keadaan umum penderita buruk disertai syok, dan hampir
semua janinnya telah meninggal.
o Komplikasi koagulopati dan gagal ginjal yang ditandai dengan oliguria biasanya
telah ada.
113. Plasenta inkreta, perkreta, arkreta?
• Plasenta akreta: villi melekat ke myometrium
• Plasenta inkreta: villi menginvasi myometrium
• Plasenta perkreta: villi menembus myometrium
114. HBES
Kriteria HBES dari Acosta Sison berarti
H: having expelled product conception
B: bleeding
ES: enlargement and softening of uterus
Artinya adalah wanita yang pernah mengeluarkan hasil konsepsi, apa pun jenisnya, kemudian
mengalami perdarahan pervaginam, disertai subinvolusi uterus, maka wanita tersebut perlu
dicurigai koriokarsinoma.
Diagnosis tumor trofoblastik gestasional: riwayat mola, bleed, enlarged and soft uterus,
peningkatan HCG. Test pack positif: >25 mIU/mL.

MOLA
115. Mola Hidatidosa di nomor 109
116. Diagnosis Mola Hidatidosa (Anamnesis, PF, dan PP)
TRIAS mola
a. Amenorea (1-2 bulan) terus diikuti perdarahan pervaginam dari spotting-profuse
b. Mual-muntah hebat
c. TFU lebih tinggi dari usia gestasi, konsistensi lunak
Anam: keluhan mual-muntah hebat, berdebar-debar, amenorea, diikuti perdarahan per vaginam
PF:
• Inspeksi: muka (dan kadang-kadang badan juga) berwarna kekuningan à mola face
• Palpasi: ukuran uterus membesar tidak sesuai usia hamil, teraba lunak, tidak teraba bagian
janin, ballottement, dan gerak janin
• Auskultasi: tidak ada DJJ
• Pemeriksaan dalam: Memastikan besar uterus, uterus lunak, terdapat perdarahan di
kanalis servikalis
Hasil penemuan fisik:
- Mola sempurna
• Ukuran uterus tidak sesuai usia hamil: disebabkan karena pertumbuhan trofoblastik
berlebih dan darah yang tertampung
• Preeklamsia: sekitar 27% pasien mola sempurna datang dengan hipertensi, proteinuria,
dan edema dengan hiperrefleksia. Kejang jarang terjadi.
• Kista techa lutein: kista ovarium dengan ukuran >6 cm, diikuti dengan pembesaran
ovarium. Kista ini biasanya tidak dapat dipalpasi pada pemeriksaan bimanual namun
dapat teridentifikasi dengan USG. Pasien biasanya mengeluh nyeri pelvis. Risiko torsi
dari kista ini >>. Kista ini terbentuk karena tingginya kadar beta-HCG pada pasien
dengan mola.
- Mola parsial
• Jarang memperlihatkan tanda fisik, biasanya dengan penemuan USG
• Gejala pada mola sempurna jarang terjadi
- Mola kembar
• Gestasi kembar dengan mola sempurna dan janin dengan plasenta normal telah
dilaporkan. Bayi lahir sehat (dengan kembar mola) juga pernah ada.
• Wanita dengan gestasi normal dan mola berisiko menjadi persisten dan ganas. Akhiri
kehamilan.
• Kehamilan boleh dilanjutkan jika kondisi ibu stabil, tanpa perdarahan, tirotoksikosis,
atau hipertensi berat.
• Diagnosis genetic melalui sampling chorionic villus atau amniosentesis
direkomendasikan untuk evaluasi karyotype fetus
PP:
• Serum beta-HCG
o Komplit: >100.000 mIU/mL
o Parsial: <100.000 mIU/mL
• USG
o Komplit: snowstorm appearance
o Parsial: terdapat penebalan uterus, plasenta multikistik dengan janin/jaringan janin
• Histopatologi
o Stroma villi korialis edematous, tidak mengandung pembuluh darah, disertai
hyperplasia sel sitotrofoblas dan sinsitiotrofoblas
• Lab
o Immunochemical stain: p57 expression
§ Komplit: p57 KIP2 negatif
§ Parsial: p57 KIP2 positif

117. Diagnosis pasti mola: PA, hasilnya? Di nomor 116


118. Teori terjadinya mola? Di nomor 109
119. Perbedaan high risk mola dan low risk mola? Gaada high risk mola atau low risk mola, adanya
high risk GTN dan low risk GTN
Score 0 1 2 4
Usia <40 tahun ≥40 tahun
Kehamilan sebelumnya Mola Abortus Hamil (aterm)
Jarak hamil sebelumnya <4 bulan 4-7 bulan 7-12 bulan >12 bulan
HCG serum sebelum treatment <1 ribu <10 ribu <100 ribu >100 ribu
Largest tumor size <3 cm <5 cm ≥5 cm
Lokasi metastase (jika ada) paru-paru ginjal, limpa GIT otak, liver
Jumlah metastase 0 1 s.d. 4 5 s.d. 8 >8
Gagal kemoterapi none single drug multidrug
Jika 0-6: low risk
Jika ≥7: high risk
120. Kenapa pada mola dapat terbentuk kista techa luthein?
Overstimulasi gonadotropin oleh tingginya kadar beta-HCG (analog dari FSH dan LH) à
meningkatkan estrogen dan progesteron à progesteron tetap tinggi à luteinisasi berlebihan à
kista theca lutein.
121. Apakah mola bisa bikin takikardi? Kenapa bisa?
Bisa. Mola à beta-HCG sangat tinggi, dimana beta-HCG menyerupai TSH à Peningkatan produksi
FT4 à Takikardi
122. Kurva regresi beta-HCG Mochizuki?
Setelah jaringan mola dievakuasi à kadar beta-HCG akan menurun perlahan sampai tidak
terdeteksi lagi. Menurut Mochizuki, waktu yang dibutuhkan untuk menjadi normal (<5 mIU/mL)
yaitu 12 minggu. Sampai dengan minggu ke-12, follow up akan dilakukan secara klinis. Apabila pada
minggu ke-12, test pack masih positif berarti terdapat distorsi kurva regresi. Cek beta-HCG secara
berkala.
Minggu ke-4: beta HCG <1000 mIU/mL
Minggu ke-6: beta HCG <100 mIU/mL
Minggu ke-8: beta HCG <20-30 mIU/mL
Minggu ke-12: beta HCG <5 mIU/mL

123. Saat pasien mola kontrol post-kuretase, apa yang dipantau?


• Cegah kehamilan minimal 6 bulan dengan kontrasepsi hormonal
• Cek kadar beta HCG setelah 48 jam pasca kuretase
• Pantau kadar beta HCG tiap minggu pada 3 bulan pertama
• Pantau kadar beta HCG tiap 2 minggu pada 3 bulan kedua
• Pantau kadar beta HCG tiap bulan pada 6 bulan berikutnya
124. Dasar diagnosis dari mola? Di nomor 116
ABORTUS
125. Dasar diagnosis abortus inkomplit?
Anam: riwayat ekspulsi hasil konsepsi dan telah keluar dari rahim diikuti dengan
• Nyeri pada lower abdomen
• Perdarahan pervaginam masih berlanjut à bisa anemia dan syok
PF: pemeriksaan dalam
• Uterus lebih kecil dari periode amenorea
• Kanalis servikalis masih terbuka dan teraba jaringan di kavum uteri/menonjol pada OUE
Pemeriksaan penunjang:
• USG: besar uterus lebih kecil dari usia gestasi, kantong gestasi sulit dikenali, kavum uteri
berupa massa hipoekoik bentuk tidak beraturan (ada hasil konsepsi)
126. Bedanya abortus inkomplit dan missed abortion?
Missed abortion: abortus dengan embrio atau fetus meninggal dalam kandungan sebelum
kehamilan 20 minggu dan hasil konsepsi masih tertahan di dalam kandungan.
Anam: tidak ada keluhan, pada usia kehamilan 14-19 minggu à rahim dirasa mengecil, tanda
sekunder kehamilan di payudara menghilang (di usia minggu ke-12 abortus à maserasi dan
mumikasi), bisa ada persistent brownish vaginal discharge
PP:
• USG: uterus mengecil, kantong gestasi mengecil, bentuk tidak beraturan, DJJ (-), gerak janin
(-)
• Tes urin: negative satu minggu dari berhentinya pertumbuhan kehamilan
127. TFU sesuai usia kehamilan 17-18 minggu berapa? Kira-kira ukuran janin berapa? Panjang dan
berat berapa?
TFU 2-3 jari di bawah pusat, PB 13-14 cm, BB 140-190 gram
Usia gestasi Panjang Berat
8 mgg 1.6 cm 1 gram
Usia gestasi Panjang Berat
9 mgg 2.3 cm 2 gram
26 mgg 35.6 cm 760 gram
10 mgg 3.1 cm 4 gram
27 mgg 36.6 cm 875 gram
11 mgg 4.1 cm 7 gram
28 mgg 37.6 cm 1005 gram
12 mgg 5.4 cm 14 gram
29 mgg 38.6 cm 1153 gram
13 mgg 7.4 cm 23 gram
30 mgg 39.9 cm 1319 gram
14 mgg 8.7 cm 43 gram CRL
31 mgg 41.1 cm 1502 gram
15 mgg 10.1 cm 70 gram
32 mgg 42.4 cm 1702 gram
16 mgg 11.6 cm 100 gram
33 mgg 43.7 cm 1918 gram
17 mgg 13 cm 140 gram
34 mgg 45 cm 2146 gram CHL
18 mgg 14.2 cm 190 gram
35 mgg 46.2 cm 2383 gram
19 mgg 15.3 cm 240 gram
36 mgg 47.4 cm 2622 gram
20 mgg 16.4 cm 300 gram
37 mgg 48.6 cm 2859 gram
20 mgg 25.6 cm 300 gram
38 mgg 49.8 cm 3083 gram
21 mgg 26.7 cm 360 gram
39 mgg 50.7 cm 3288 gram
22 mgg 27.8 cm 430 gram
CHL 40 mgg 51.2 cm 3462 gram
23 mgg 28.9 cm 501 gram
41 mgg 51.5 cm 3597 gram
24 mgg 30 cm 600 gram
42 mgg 51.7 cm 3685 gram
25 mgg 34.6 cm 660 gram
128. Kenapa dilakukan dilatasi dan kuretase?
Jika sudah inkomplit, tidak perlu dilakukan dilatasi.
129. Definisi abortus? Di nomor 109
130. Faktor risiko abortus? Di nomor 109
131. Jenis abortus ada apa aja? Persenannya? Gejala dari masing-masing..? Di nomor 109
132. Missed abortion: apa itu? Mau diapain? Kuret langsung atau tunggu?
Definisi nomor 126
Terapi evakuasi. Jika kehamilan <12 minggu: lakukan dilatasi dan kuretase. Jika kehamilan >12
minggu dan <20 minggu, lakukan induksi dengan oksitosin 1 ampul per 500 cc RL atau D5%. Jika
gagal, ulangi esok harinya. Boleh diulang hingga tiga kali. Setelah pengeluaran hasil konsepsi,
lakukan kuretase.
133. Terapi abortus?
Trimester I:
• Medis: mifepristone dan misoprostol/MTX dan misoprostol/Tamoxifen dan misoprostol
(dosis pada table 18-9)
• Surgical: induksi menstruasi, aspirasi vakum, suction evacuation dan atau kuret, dilatasi dan
evakuasi
Trimester II
• PGE1 (misoprostol), carboprost, dinopristone, intrauterine instillation of hyperosmotic
solution, oksitosin infusion

134. Hubungan progesteron untuk pertahankan kehamilan


Progesteron à mempertahankan endometrium tidak luruh dan membantu pembentukan dinding
rahim untuk implantasi janin
135. Faktor penyebab aborsi? Di nomor 109
136. Penyebab tersering abortus? Di nomor 109
137. Infeksi apa yang paling sering menyebabkan abortus? TORCH
a. Apa itu TORCH?
TORCH à toxoplasma, rubella, cytomegalovirus, dan herpes simplex virus-II
b. SIklus hidup toxoplasma, kista jenis apa?
Parasit ini merupakan jenis intraseluler obligat (hanya bisa bereproduksi ketika di dalam
sel inang) dengan hospes definitive kucing dan hospes intermediate manusia. Siklus
kehidupannya digambarkan terjadi di dalam tubuh manusia dan kucing.
Siklus kucing: kucing terinfeksi jika memakan tikus/burung yang mengandung kista Toxo
atau makanan terkontaminasi ookista à ookista pecah à bradizoit di dalamnya keluar
dan menembus epitel usus kucing à menjadi takhizoit à mikrogamet dan makrogamet
à zigot à ookista à keluar bersama feses kucing dan hidup di dunia luar hingga ditelan
hospes
Siklus manusia: ookista (berisi takhizoit) yang tertelan akan sampai ke usus dan pecah à
takhizoit à ke usus, darah, dan lain-lain à multiplikasi intraseluler à membentuk
pseudokista (berisi bradizoit) pada fase kronik. Bradizoit biasanya tidak multiplikasi.
Pertumbuhan terhenti di fase pseudokista à pecah, siklus berulang.
Penularan terjadi melalui:
o Peroral (misalkan menelan daging mentah/kurang matang, sayur yang tidak dicuci
bersih)
o Perinhalasi (menghirup ookista yang terbang bersama debu)
o Tranfusi darah yang mengandung takhizoit
o Menerima organ donor yang terinfeksi dan mengandung bradizhoit
o Transmisi transplasenta
c. Toxo vs. Rubella pada TM I, mana yang lebih bahaya?
Rubella. Karena angka kelainan kongenital akibat Rubella pada infeksi trimester pertama
lebih tinggi (25% rubella, 1/10.000-30.000 toxo). Rubella juga memiliki angka lebih tinggi
dalam mengakibatkan kematian janin (stillbirth).
138. Kelainan darah yang bikin abortus? Apakah yang paling sering?
Antiphospolipid syndrome (APS), Systemic Lupus Erithematous, Trombophilia, Rhesus
incompatibility.
Definisi APS: ditemukannya autoantibodi phospolipid yang berikatan dengan protein plasma dan
kejadian abortus serta meningkatnya risiko tromboemboli.
Patofisiologi dari APS:
1. Aktivasi prokoagulan
2. Inaktivasi antikoagulan
3. Aktivasi sistem komplemen
4. Gangguan diferensiasi sinsitiotrofoblas karena adanya antibodi phospholipid terhadap
beta 2-glikoprotein 1 yang ada di permukaan sinsitiotrofoblas à uncontrolled placental
complement activation by antiphospholipid antibodies à (1) fetal loss and growth
restriction, (2) exposure of basement membrane à adherensi platelet à trombus
Diagnosis
1. Ditemukan satu dari dua gejala klinis (vascular thrombosis atau pregnancy morbidity)
2. Ditemukan minimal satu dari kriteria pemeriksaan laboratorium: aktivitas LAC (lupus
anticoagulant), IgG-ACA atau IgM-ACAs (Anticardiolipin antibody) terkonfirmasi dengan 2
kali pemeriksaan dimana jarak pemeriksaan 12 minggu.

APS dan kehamilan:


• decidual vasculopathy
• placental infarction
• fetal-growth restriction
• early-onset preeclampsia
• recurrent fetal death

Tatalaksana APS:
Aspirin dan heparin merupakan regimen paling efektif. Low-dose unfractionated heparin (7500-
10,000 U) diberikan secara subkutan sebanyak 2 kali. Aspirin diberikan 75-100 mg/hari per oral.
139. Tatalaksana abortus dan gejala/keluhan di nomor 133
140. Kenapa dikasih misoprostol? Selain misoprostol bisa diberikan apa supaya serviks lunak dan
bisa dikuret?
Misoprostol diberikan untuk pematangan kanalis servikalis ibu à mau kuret, mau diinduksi tapi
Bishop’s score masih di bawah 5. Selain misoprostol bisa diberikan disoptostone atau dilakukan
pematangan serviks dengan mekanik (foley catheter)
141. Abortus pre-embrionik penyebabnya apa?
Faktor kromosom à aneuploidy à gagal gametogenesis à bisa trisomy autosomal, monosomi
142. Tokolitik yang direkomendasikan?
Nifedipine.
Nifedipin adalah obat golongan calcium channel blocker yang mengakitbatkan relaksasi otot polos
dan digunakan untuk terapi dari hipertensi kronis. Obat ini telah digunakan sejak tahun 1980
sebagai tokolitik yang dikatakan lebih efektif dibandingkan golongan betamimetik. Nifedipin
diberikan per oral, dengan loading dose 30 mg dan maintenance dose 20 mg per 6 jam. Dengan
regimen ini, sebagian besar wanita berhenti mengalami kontraksi. Efek samping utama dalah sakit
kepala, namun efek pada janin tidak ada.
143. Abortus tuba? Di nomor 109

KEHAMILAN EKTOPIK
144. Kehamilan abdominal?
Kehamilan abdominal adalah implantasi janin pada rongga peritoneum setelah kehamilan tuba,
ovarium, dan interligamen dieksklusi.
Insidens: 1 dari 10000-25000 kelahiran hidup. Sebanyak 20% janin dari kehamilan abdominal
mengalami malformasi dan deformitas pada wajah dan atau ekstremitas, sendi, dan sistem saraf
pusat.
Dapat terjadi secara primer: menempel langsung di dinding abdomen sejak awal dan sekunder: di
nomor 109
Diagnosis: asimtomatik, pada leopold teraba posisi janin yang abnormal, dan serviks berubah
tempat. Pada gambaran USG sulit terdeteksi: oligohidramnion, janin terpisah dari uterus atau
berada di pelvik, sedikitnya myometrium diantara janin dan dinding abdomen ibu atau pun kandung
kemih, dan ditemukannya jaringan plasenta di luar rahim. Sering ditemukan peningkatan
alfafetoprotein.
Kehamilan abdominal bisa mengancam nyawa, dan tata laksana tergantung pada usia gestasi saat
kehamilan ditemukan. Prinsip utama tata laksana adalah melahirkan bayi dengan pengawasan ketat
pada plasenta agar tidak terjadi perdarahan.
1. Melahirkan janin:
a. Terminasi pada saat diagnosis ditegakkan
b. Menunggu janin viabel à kehamilan dilanjutkan hingga 24 minggu dengan tatalaksana
konservatif. Tata laksana konservatif sendiri berbahaya karena adanya risiko
perdarahan tiba-tiba.
2. Melahirkan plasenta
Jika plasenta sudah terbentuk, dapat dilakukan preoperative angiographic embolization atau
pemasangan kateter di dekat arteri uterus untuk mengurangi jumlah hilangnya darah selama
terminasi janin dilakukan. Pengangkatan plasenta dapat menyebabkan perdarahan besar
karena gangguan kontraksi myometrium pada kehamilan abdominal.
a. Jika plasenta bisa diangkat, lakukan setelah ligasi arteri plasenta.
b. Jika tidak bisa, plasenta dibiarkan di dalam rongga perut untuk resorpsi alami dengan
risiko terjadinya infeksi, adhesi, dan obstruksi. Resorpsi dapat terjadi selama bertahun-
tahun. Selama menunggu resorpsi, perlu dilakukan pemantauan ketat dari plasenta
dengan USH dan pengukuran beta HCG. Penggunaan metotreksat masih kontroversial.

145. Anti fosfolipid syndrome atau Hughes syndrome?


APS atau dikenal dengan Hughes syndrome mrupakan kelainan pada sistem imun – autoimun
yang menyebabkan peningkatan risiko pembekuan darah. Pada APS sistem imun membentuk antibodi
abnormal yang dikenal dengan antifosfolipid antibodi dengan tujuan menyerang protein yang menempel
pada molekul lipid pada fosfolipid sehingga menyebabkan darah lebih mudah membeku.

146. KE ovarium itu apa? Kriteria nya apa?


KE ovarium termasuk dalam kelompok KE non-tuba, dimana kejadian ektopik pada ovarium jarang
(3%). Walaupun KE ovarium dapat mengakomodasi kehamilan dan membesar lebih mudah dibandingkan
dengan tuba fallopi tetapi pada stadium awal biasanya sering terjadi ruptur. Diagnosis dapat dilakukan
dengan 4 kriteria dari Spiegelberg (1878) dan USG, dimana ditemukan kista dengan lingkaran echogenik
yang lebar pada atau di sekitar ovarium.

147. Kriteria Spiegelberg apa aja? Buat apa aja? Isinya ada apa aja?
Kriteria Spiegelberg untuk mendiagnosis KE ovarium. Kriteria Spiegelberg :
• Tuba ipsilateral normal dan terpisah dari ovarium
• Kehamilan ektopik berlokasi pada ovarium
• Kehamilan ektopik dihubungkan pada uterus melalui ligamentum uteroovarian (ligamentum ovari
proprium)
• Jaringan ovarium yang ditemukan secara histologis pada jaringan plasenta (dalam dinding
kantong janin)

148. Perdarahan paling banyak pada KET di lokasi apa? Kenapa?


Perdarahan paling banyak dapat terjadi pada KET tuba pars interstisialis karena dekat dengan
arteri uterine dan arteri ovarika. KET interstitsialis yang tidak terdiagnosis biasanya ruptur setelah 8-16
minggu amenorrhea karena kemampuan distensi dari miometrium yang melapisi segmen interstisial.
Tatalaksana KET pars interstisialis adalah wedge resection atau cornual resection atau cornuostomy pada
kornu uteri atau histerektomi.

149. Etiologi KE
Dari Williams adanya risiko :
• Anatomi abnormal dari tuba fallopi (bisa kongenital akibat paparan diethylstilbestrol)
• Pembedahan KE tuba sebelumnya
• Riwayat KE sebelumnya
• Riwayat penyakit menular seksual atau infeksi tuba lainnya (salpingitis/isthmica nodosa, infeksi
chlamydia)
• Perlengketan perituba akibat salpingitis, apendisitis, endometriosis
• Pengunaan IVF (konsepsi natural dengan IVF memiliki perbedaan % lokasi)
• Implantasi atipikal (kornual, abdominal, servikal, ovarium, dan heterotopik)
• Merokok
• Kegagalan metode kontrasepsi (sterilisasi tuba, IUD-Cu dan LNG, pil progestin)

150. Terapi KE
Terapi KE bervariasi tergantung letak dari kehamilan ektopik. Namun secara umum terapi untuk
kehamilan ektopik dapat diberikan melalui obat-obatan atau pembedahan. Secara tradisional, obat-
obatan terdiri dari antimetabolit seperti metotreksat. Metotreksat dapat diberikan dosis tunggal, dosis
multipel atau dua dosis (dosis hybrid, pada hari ke 0 dan ke 4 tanpa disertai leucovorin). Pembedahan
dapat dilakukan salpingostomi atau salpingektomi.
• KE interstisial (riwayat ipsilateral salpingektomi) à bedah (cornual resection atau cornuostomy
via laparotomy atau laparascopy) dan terapi konservatif (MTX 50 mg/m2 BSA atau direct MTX ke
gestasional sac.
• KE abdominal (bergantung pada usia gestasi) à waiting, angiographic embolization, catheter
urine, tujuannya melahirkan janin dan menilai implantasi plasenta.
• KE intraligamen à laparotomy atau eksisi laparaskopik
• KE ovarium à pembedahan (kalau kecil di wedge resection atau cystectomy) kalau besar
dilakukan oophorectomy. Dan MTX sistemik atau lokal dapat mengatasi kehamilan tuba
• KE servikal à MTX merupakan 1st line therapy mengikuti tabel di atas. Beberapa studi dapat
dikombinasikan dengan emboli arteri uteri. Pada beberapa kasus dapat dilakukan suction
curettage atau histerektomi.
• KE bekas sesar à histerektomi pada pasien yang berharap sterilisasi. Apabila masih berharap
adanya kehamilan dapat diberikan MTX yang diinjeksi secara lokal sendiri ataupun digabung
dengan pembedahan konservatif (visually guided suction curettage atau aspirasi transvaginal,
pembedahan histeroskopik, eksisi ismus. Emboli arteri uterine untuk mengutangi risiko
perdarahan).
• Kehamilan ektopik di luar (omentum, spleen, liver, dan retroperitoneum, intramular uterine
implantations
151. Patofisiologi KE
Sel telur yang sudah dibuahi dalam perjalanannya menuju endometrium tersendat sehingga
embrio sudah berkembang sebelum mencapai kavum uteri dan akibatnya akan tumbuh di luar rongga
rahim. Inflamasi pada tuba fallopi juga dapat menghambat progresi embrio dan mencetuskan sinyal
proinflamasi dini. Merokok meningkatkan endocannabinoid sehingga mengganggu fungsi tuba fallopi.

KEHAMILAN EKTOPIK TERGANGGU


152. Dasar diagnosis KET
Nyeri pelvis unilateral/nyeri abdomen bawah, vaginal bleeding, massa adneksa, uterus kecil,
perdarahan serviks, hipotensi, takikardi, gejala iritasi peritoneum (Kehr Sign), beta HCG rendah,
hematokrit dan Hb menurun, usg ditemukan massa pada adneksa atau kehamilan di luar uterus.

153. Perbedaan KE dan KET


Disebut KET apabila tempat implantasi hasil konsepsi tidak dapat menyesuaikan ukurannya
dengan ukuran hasil konsepsi yang terus berkembang sehingga terjadi ruptur pada tempat tersebut
menjadikah kehamilan ektopik ini terganggu. KE ditemukan rasa tidak nyaman di salah satu sisi abdomen,
amenorrhea, bisa ditemukan flek darah; Pada KET ditemukan nyeri abdomen yang berat, perdarahan, dan
tanda-tanda syok.

154. Trias KET


• Amenorrhea
• Flek darah dari vagina (vaginal spotting)
• Nyeri perut

155. Definisi KET?


Kegawatdaruratan obstetri pada awal kehamilan dimana terdapat implantasi abnormal dari hasil
konsepsi di luar dinding endometrium kavum uteri dan tempat nidasi hasil konsepsi tersebut tidak dapat
menyesuaikan dengan besarnya buah kehamilan sehingga terjadi ruptur.

156. KET yang sering terjadi dimana?


Kehamilan ektopik tersering terjadi di tuba (95%) pars ampularis (70%), isthmus 12%, interstisial
2-3%, fimbrial 11%, ovarium 3%, cervical <1%, skar sesar <1%, intraligament atau abdominal 1%.

157. Etiologi KET


• Faktor tuba
o Infeksi à salpingitis à lumen tuba menyempit
o Endometriosis
o Pasca operasi rekanalisasi tuba
o Kongenital à divertikel saluran tuba
o Massa à mioma uteri, tumor ovarium
• Faktor ovarium à ovum ditangkap oleh tuba kontralateral
• Faktor zygot à zygot tumbuh terlalu cepat atau tumbuh dengan ukuran besar
• Faktor hormonal à pil KB dengan progesteron
• Faktor lainnya à pemakaian IUD, perokok
158. Kenapa KB progesterone bisa bikin KET?
KB progesterone dapat menyebabkan perlambatan gerakan silia pada tuba sehingga dapat
mencetuskan kehamilan ektopik.
159. Cara penanganan KET di kornu uterus / interstisial?
Pembedahan secara Corneal resection (>4 cm) atau cornuostomy (<3,5cm). Dapat pula diberikan
MTX 50 mg/m2 jika syarat-syarat penggunaan MTX terpenuhi.

160. Kalo gak ada USG bisa nentuin KET dari mana?
• Anamnesis à trias KET (amenorrhea, nyeri abdominal, vaginal bleeding/spotting)
• Pemeriksaan fisik à nyeri tekan abdomen, Kehr Sign (nyeri di punggung), nyeri goyang serviks,
uterus membesar di bawah usia kehamilan, KU memburuk, tanda syok.
• Pemeriksaan pelvis à mukosa vagina putih pucat, uterus normal atau bulky, extreme tenderness
pada palpasi forniks dan nyeri goyang serviks (Sliding pijn).
• Pemeriksaan penunjang à tes kehamilan, beta HCG <1500mIU/mL; progesterone < 5ng/mL;
kuldosentesis; fonslani test; leukosit (CBC).

161. Tatalaksana KET?


• Salpingektomi dengan kondisi (buku hijau kebidanan)
o Kondisi penderita buruk (dalam keadaan syok)
o Kondisi tuba buruk, terdapat jaringan parut yang tinggi risiko akan KE berulang
o Penderita menyadari kondisi fertilitasnya dan mengingini fertilisasi invitro
o Tidak ingin punya anak lagi
• Salfingo-ooforektomi atau cornuostomi pada interstisial (buku hijau kebidanan)
• Konservasi tuba (buku hijau)
o Salpingostomi (ampula dan infundibulum)
162. Syarat MTX? Dosis MTX?
• Dosis 1 mg/kgBB IV
• Pars ampularis tuba yang belum pecah
• Diameter kantong gestasi ≤ 3,5-4 cm
• Beta HCG < 1500 mIU/mL
• Perdarahan dalam rongga perut ≤ 100 mL
• Tanda vital baik dan stabil
• DJJ (-)

163. Kalo KET pas minggu 8, apa yang diperiksa? Berapa cut off Beta HCG?
• Serum beta-HCG (> 1500 dengan TVUS dan >6000 dengan transabdominal)
• TVS
o Pemeriksaan TVS kehamilan normal ditemukan :
§ Gestational sac usia kehamilan 4½ hinga 5 minggu.
§ Yolk sac ditemukan pada 5-6 minggu,
§ Fetal pole dengan cardiac activity 5½ - 6 minggu
o Pemeriksaan TVS KET :
§ Trilaminar endometrial pattern
§ Ring of fire

164. Selain beta-HCG temuan apa yang khas?


• Penemuan USG ditemukan gestasional sac di luar cavum uteri
• Kuldosentesis ditemukan darah kecokelatan
• Pemeriksaan Hb serial (fonslani test)

165. Yang didapat dari TVUS apa?


• Pseudogestational sac
• Free fluid
• Doppler à Ring of fire
• Perdarahan cul de sac
• Decidual cyst
• Trilaminar pattern

POST-PARTUM HEMORRHAGE
166. Definisi HPP?
Perdarahan pasca persalinan dengan kehilangan ≥500 mL darah setelah selesai kala III (setelah
bayi dan plasenta lahir) pada persalinan pervaginam. Jika dilakukan persalinan melalui seksio sesarea
ditemukan perdarahan ≥1000 mL.
Perdarahan masif yang dapat menimbulkan gejala pada pasien disertai atau tanpa disertai dengan
tanda hypovolemia.

167. Penyebab HPP 4T? Infeksi apakah bisa bikin HPP?


• Primer (<24 jam)
o Tonus (90%)
o Trauma (6%)
o Tissue (3-4%)
o Thrombin (sisanya)
• Sekunder (>24 jam hingga 6-12 minggu)
o Uterine infection (paling sering)
o Retained placenta
o Fragment à keluar jaringan seperti selaput
o Abnormal involution of placental site (subinvolution à sikatrik akibat bekas lepasnya
plasenta)
168. Jenis HPP?
Early (6-12 minggu) dan late (24 jam hingga 12 minggu) post partum hemorrhage?

169. Tanda-tanda inversio uteri? PF yang didapatkan? Patofisiologi?


Faktor risiko : implantasi plasenta di fundus, atonia uteri, plasenta akreta, dan traksi berlebihan
pada tali pusat saat kala III.
Tanda : fundus uteri menempel pada plasenta saat persalinan plasenta, respon vasovagal berat,
syok karena kesakitan (neurogenik), perdarahan banyak bergumpal, di vulva tampak endometrium
terbalik dengan atau tanpa plasenta yang masih melekat. Penyebab atonia uteri, serviks masih terbuka
lebar, adanya kekuatan yang menarik fundus ke bawah (plasenta akreta, inkreta, perkreta, tali pusat
ditarik keras), atau ada tekanan pada fundus dari atas (manuver Crede), tekanan intraabdominal keras
dan tiba-tiba (batuk keras/bersin)

170. Tujuan PTT (peregangan tali pusat terkendali) apa aja?


PTT untuk pelahiran plasenta dengan meningkatkan kontraksi uterus, mencegah HPP dengan
averting atonia uteri

171. HPP late dan early onset?


Late HPP kalau perdarahan setelah 24 jam pertama hingga 12 minggu setelah persalinan.

172. Apa itu inversio uteri? Penyebabnya? Tanda-tandanya? Tatalaksana?


Keadaan dimana lapisan endometrium turun dan keluar lewat OUE dan dapat dikategorikan
menjadi inversion uteri komplit dan inkomplit.

ENDOMETRIOSIS
173. Definisi endometriosis? Patogenesis endometriosis?
Pertumbuhan kelenjar dan stroma dinding uterus pada lokasi abnormal atau heterotopik.
Perempuan dengan endometriosis bisa tanpa gejala, subfertil, atau menderita rasa sakit pada daerah
pelvis terutama pada menstruasi (dismenorea). Tanda klinis endometriosis :
• Dismenorhea – 50% (nyeri haid disebabkan oleh reaksi peradangan akibat sekresi sitokin dalam
rongga peritoneum, akibat perdarahan lokal pada sarang endometriosis dan oleh adanya infiltrasi
endometriosis ke dalam syaraf rongga panggul)
• Nyeri pelvis - 85% (akibat perlengketan, lama-lama dapat mengakibatkan nyeri pelvik yang kronis,
rasa nyeri dapat menyebar ke dalam panggul, punggung, paha, dan bahkan menjalar sampai ke
rectum dan menyebabkan diare. Nyeri kebanyakan dialami pada saat intermenstrual).
• Dispareunia – 64% (hubungan seksual memberikan penekanan langsung pada organ pelvis yang
immobile apabila endometriosis tumbuh di sekitar kavum Douglasi dan ligamentum sakrouterina
dan terjadi perlengketan sehingga dalam posisi retrofleksi).
• Subfertilitas (perlengketan pada ruang pelvis menyebabkan endometriosis menganggu pelepasan
oosit dari ovarium dan menghambat perjalanan ovum untuk bertemu dengan sperma).
Endometriosis meningkatkan volume cairan peritoneal, konsentrasi makrofag yang teraktivasi, IL-
1, TNF dan protease. Cairan peritoneum mengandung inhibitor penangkap ovum yang
menghambat interaksi normal fimbrial cumulus à efek buruk bagi oosit, sperma, embrio, dan
fungsi tuba. Kadar tinggi NO à memperburuk motilitas sperma, implantasi dan fungsi tuba.
Antibodi IgA dan IgG dan limfosit dapat meningkat à mengubah reseptivitas endometrium dan
implantasi embrio.
• Diskezia – keluhan sakit buang air besar (bila endometriosis sudah tumbuh dalam dinding
rektosigmoid dan terjadi hematokezia pada saat siklus haid).
Patogenesis endometriosis eksterna (COMPRE GYN) :
• Retrograde menstruation (John Sampson) : sel endometrial dan mesenkim dikeluarkan pada saat
menstruasi à terdapat obstruksi traktus genitalia à mengalami refluks à sel endometrium
sampai di peritoneum à adhesi dan invasi à aktivasi dan infiltrasi sel inflamasi dan sel imun à
tumbuh dipengaruhi oleh hormon (prostaglandin dan estradiol) dan mengalami angiogenesis
serta antiapoptosis à fibrosis, scarring pain.
• Metaplasia koelomik : metaplasia epitel coelomic yang berasal dari Mullerian duct atau
proliferasi sisa embrionik akibat stimulus hormonal menjadi sel endometrium ektopik
• Metastasis limfo dan hematogenik : endometrial berpindah melalui jalur limfogenik dan
hematogenik
• Diseminasi iatrogenic (post sc) : endometriosis pada dinding abdomen anterior post sc
memungkinkan kelenjar dan stroma untuk berimplantasi saat prosedur berlangsung.
• Patoimunologi : reaksi abnormal imunologi yang tidak berusaha membersihkan refluks haid dalam
rongga peritoneum justru memfasilitasi terjadinya endometriosis dengan cara apoptosis sel
endometrium ektopik menurun. Peningkatan jumlah makrofag dan monosit dalam cairan
peritoneum yang teraktivasi mengjasilkan faktor pertumbuhan dan sitokin yang merangsang
tumbuhnya endometrium ektopik. Aktivitas aromatase intrinsic meningkat pada sel endometrium
à menghasilkan estrogen lokal à estrogen lokal berlebihan à respon sel endometrium ektopik
terhadap progesteron menurun. Peningkatan sekresi molekul neurogenic NGF (nerve growth
factor) dan reseptornya à tumbuhnya saraf sensoris pada endometrium. IL-1 dapat
meningkatkan perkembangan endometriosis à merangsang faktor angiogenik (VEGF), IL-6, IL-8
dan ICAM-1 (intercellular adhesion molecule -1) à membantu sel endometrium refluks ke dalam
rongga peritoneum terlepas dari pengawasan imunologis.IL-8 merupakan sitokin angiogenik yang
kuat dengan fungsi merangsang perlengketan sel stroma endometrium ke protein matrix
ekstraselular à meningkatkan aktivitas MMP à membantu implantasi dan pertumbuhan
endometrium ektopik.

174. Endometriosis ada beberapa jenis? (gambar next page)

175. Gambaran endometriosis? GnRH agonis cara kerjanya gimana? Kalo kasih GnRH antagonis boleh
gak? Bisa dengan COC?
Pemeriksaan endometriosis dapat dilakukan :
• USG à dapat digunakan untuk mendiagnosis endometriosis > 1 cm, tidak dapat digunakan untuk
melihat bintik-bintik endometriosis ataupun perlengketan. Melalui TVUS ditemukan kista
endometriosis berbentuk kistik dan interval echo di dalam kista.
• MRI à lebih superior dibandingkan USG. MRI dapat melihat kista, massa ekstraperitoneal, invasi
ke usus dan septum rektovagina.
• CA-125 à terjadi peningkatan kadar CA-125 namun pemeriksaan ini mempunyai nilai sensitifitas
yang rendah. Ca-125 dapat digunakan sebagai monitor prognostic pascaoperatif endometriosis
bila nilainya tinggi (hasil serum > 65 mIU/mL preoperatif menunjukan derajat beratnya
endometriosis)
• Laparaskopi (GOLD STANDARD) à ditemukan lesi berwarna merah terang menandakan lesi aktif
yang baru, sednagkan jika lesi aktif sudah lama akan berwarna merah kehitaman. Lesi nonaktif
berwarna putih dengan jaringan parut. Pada endometriosis yang tumbuh di ovatium dapat
membentuk kista yang disebut dengan endometrioma biasanya isinya berwarna cokelat
kehitaman (chocolate cyst).
• Patologi anatomi à didapatkan adanya kelenjar dan stroma endometrium pada kista.
GnRHa (gonadotropin releasing hormone agonist) diberikan selama 6-12 bulan bekerja menyebabkan
sekresi terus menerus dari FSH dan LH sehingga hipofisis mengalami desensitisasi à sekresi FSH dan RH
menurun mencapai keadaan hipogonadotropik hipogonadisme, dimana ovarium tidak aktif sehingga tidak
terjadi siklus haid. Biasanaya diberikan terapi add black dengan estrogen dan progesterone alamiah untuk
mengurangi efek samping (hot flushing, vagina kering, kelelahan, sakit kepala, penurunan libido, depresi
atau penurunan densitas tulang).
Kombinasi kontrasepsi oral (COC) dapat diberikan dengan dosis rendah (30-35 mcg etinilestradiol
terus menerus). Pilihan pertama merupakan kombinasi monofasik (sekali sehari selama 6-12 bulan)
diberikan untuk menimbulkan kondisi kehamilan palsu yang ditandai dengan timbulnya amenorea dan
desidualisasi jaringan endometrium

176. Beda adenomiosis dengan mioma uteri?


Adenomiosis à jaringan endometrial di miometrium.
Mioma uteri à tumor jinak yang struktur utamanya adalah otot polos uterus. Gejala yang
ditimbulkan antara lain metroragia, nyeri, menoragia, hingga infertilitas. Klasifikasi :
• Mioma submukosa à menempati lapisan di bawah endometrium dan menonjol ke dalam kavum
uteri. Mioma jenis ini dapat bertangkai panjang sehingga dapat keluar melalui ostium serviks.
Yang harus diperhatikan dalam menangani mioma bertangkai adalah kemungkinan terjadinya
torsi dan nekrosis sehingga risiko infeksi sangat tinggi.
• Mioma intramural à mioma yang berkembang di antara miometrium
• Mioma subserosa à mioma yang bertumbuh di bawah lapisan serosa uterus dan dapat
bertumbuh kea rah luar dan juga bertangkai. Mioma subserosa juga dapat menjadi parasite
omentum atau usus untuk vaskularisasi tambahan bagi pertumbuhannya.

177. Beda endometriosis dan adenomiosis?


Endometriosis à adanya functioning endometrium (kelenjar dan stroma) di tempat selain
mukosa uterus, biasanya gejala muncul secara progresif meningkat. Gejala : nyeri pelvis dismenorea,
subfertilitas, dyspareunia, abnormalitas menstruasi.
Adenomiosis à kondisi ingrowth endometrium ke dalam miometrium. Gejala : menoragia,
dismenorea, subfertilitas, Rahim membesar secara merata, biasanya dijumpai nyeri tekan dan sedikit
lunak bila dilakukan pemeriksaan bimanual sebelum haid (Halban sign), jika endometriosis besar dapat
ditemukan massa pada adneksa (biasanya kalau besar tidak nyeri dibandingkan dengan yang fokus kecil).

178. Beda adenomiosis dan leiomyoma?


Adenomiosis à endometriosis interna yang berimplantasi pada miometrium
Leiomioma à tumor jinak otot polos dari uterus. Kebanyakan leiomyoma mengandung jumlah
jaringan fibrosa yang bervariasi yang dipercaya akibat dari degenerasi sel otot polos.

179. Apa itu miolisis?


Miolisis (destruksi fibroid uteri atau suplai darah melalui ultrasound, laser, cryoteraphy atau
metode lainnya) merupakan terapi konservatif alternative bagi wanita yang tidak ingin punya anak namun
tetap ingin memiliki uterus. Syarat miolisis : ukuran lesi ≤ 5 cm atau ≤ 10 cm.

EKSTRAKSI VAKUM
180. Indikasi vakum?
• Faktor janin
o FHR nonreassuring
o Pemisahan plasenta dini
• Faktor ibu
o Penyakit jantung
o Cidera paru
o Infeksi intrapartum
o Kondisi neurologis
o Ibu kelelahan
• Faktor waktu à mempercepat kala II

• Sungsang

181. syarat vakum?


presentasi kepala
hodge III
TBJ 2500-4500 g
pembukaan lengkap
ada kontraksi uterus dan tenaga mengejan
182. cara vakum
- posisi ibu litotomi
- cup dimasukkan ke dalam vagina dan dipasang pada bagian terendah kepala
- pompa penghisap dinyalakan hingga tenaga 0,2 kg/cm2 hingga 0,8 kg/cm2 pada menit ke 6 hingga
kepala terbentuk kaput chignon
- pastikan tidak ada bagian jalan lahir yang ikut terjepit.
- saat timbul his pada ibu, ibu mengejan, cup ditarik searah dengan sumbu panggul.
- kepala janin dilahirkan dengan menarik cup ke atas hingga kepala bergerak defleksi dan kepala lahir
sebagaimana mestinya, tangan kiri penolong menahan perineum. Setelah kepala lahir, mangkuk
dilepas.
- Bila perlu dilakukan episiotomi sebelum cup dipasang atau saat kepala membuka vulva.
183. dilakukan episitomi gak pada pasien yang mau di vakum? jenisnya?
masih menjadi kontradiksi, saat ini tidak direkomendasikan karena sulit sembuh dan prolonged
discomfort. kalo memang perlu dilakukan jenis mediolateral untuk mencegah ruptur sphincter anal.
184. berapa lama ditaruh cup-nya?
kurang lebih 8 menit, dari 0,2 kg/cm2 ditingkatkan setiap 2 menit hingga 0,8kg/cm2.
185. kenapa vakum janin harus hidup?
186. Kelebihan vakum dibandingkan forceps? Forcep (hapal semua)
EKSTRAKSI FORCEPS
187. Indikasi forcep?

188. Syarat forcep?

189. Macam-macam forcep?


• cunam tinggi: kepala masih di atas PAP. sudah tidak dipakai, diganti SC
• cunam tengah: tidak memenuhi kriteria cunam tinggi atau rendah, kepala sudah bagus (sudah
engaged). cunam tengah untuk ekstraksi dan rotasi mengikuti gerakan putaran paksi dalam.
jarang dipakai, diganti vacum atau SC
• cunam rendah: kepala mencapai pintu bawah panggul dan sutura sagitalis dalam anterior
posterior.
• tipe simpson: tangkai cunam terbuka, cephalic curve lebih datar dan lebih besar. untuk kepala
sudah molase
• tipe elliot: tangkai tertutup, lengkungan kepala lebih bundar dan sempit. untuk kepala bundar
dan belum molase
• tipe khusus: cunam piper untuk melahirkan kepala janin letak sungsang.

190. Cara pasang dan cara pakai?


191. beda cunam piper dengan cunam yang lain?
daun cunam berfenestra, mempunyai lengkung panggul yang agak mendatar.
tangkai panjang, melengkung ke atas dan terbuka. untuk menghindari kompresi berlebih
pada kepala janin.
192. kapan forcep dianggap gagal / syarat gagalnya forceps?
sendok cunam tidak dapat dikunci walau pemasangan cunam sudah betul.
3 kali traksi dengan tenaga cukup janin tidak lahir.
penyebab: kesalahan menentukan denominator, adanya lingkaran konstriksi, CPD

193. Langkah forcep


HISTEREKTOMI
194. Langkah-langkah histerektomi
195. Cara histerektomi yang bukan HTSOB?
196. Beda histerektomi total dengan BSOT?
197. Apa yang dipotong kalo histerektomi?
198. Ligament mana dulu yang dipotong? Ligamen uterus apa aja? Insersio dan origo
ligament?
199. Urutan lapisan pas histerektomi?

200. Gimana cara histerektomi vaginal dan abdominal? Lapisan abdomennya apa aja?
Tujuan pemotongan ligamentum rotundum buat apa?
201. urutan histerektomi abdominal?
202. langkah histerektomi transvaginal?
203. histerektomi radikal?
PERSALINAN SPONTAN PERVAGINAM
204. Nyanyian PSP (kala II dari mau mulai sampe selesai)
• KALA 2 - tunggu crowning 4-5 cm. minta ibu memposisikan diri memeluk kaki kearah
dada, letakan tangan penolong yang dilapis duk steril di perineum, dan satu lagi di pubis.
Saat kepala keluar, lakukan suction dan lakukan pemeriksaan leher apakah ada tali pusat
yang melilit atau tidak, bila ada yang melilit lepaskan atau potong. kepala kemudian
akan dengan sendirinya melakukan putar paksi luar, kemudian bantu lahiran pundak
dengan traksi ke posterior dan ke anterior dengan memegang kepala bayi di biparietal.
Sehingga beringan keluar pundak anterior dan posterior kemudian klem tali pusat 5 cm
dari insersinya ke bayi, kemudian letakan klem yang ke 2 2-3 cm dari klem pertama,
potong tali pusat.
• KALA 3 - Tunggu HIS adekuat kembali muncul baru lakukan maneuver peregangan tali
pusat terkendali. Sambil melakukan tahanan pada fundus kearah dorsokranial, sampai
plasenta keluar
• KALA 4 - Pantau TTV ibu sampai 1 jam post partum dengan pemeriksaan terhadap nadi
dan tensi setiap 15 menit

SECTIO CAESAREA
205. Langkah SC (Indonesia raya)!
• Pasien ditidurkan diatas meja operasi dengan sebelumnya diberikan premedikasi di
Ruang Persiapan oleh bagian anestesi.
• Dilakukan anestesi oleh Dokter Anestesi.
• Dilakukan toilet pada daerah operasi dengan Alkohol 70%, kemudian dengan Betadine.
• Pasien ditutup dengan duk steril kecuali daerah operasi.
• Dilakukan irisan pada daerah perut 1 cm diatas SOP ke arah pusat sepanjang 10 cm atau
irisan melintang (pfanen style), kemudian irisan diperdalam lapis demi lapis (subkutis,
fasia, otot, peritoneum parietale).
• Setelah peritoneum dibuka, pasang tampon usus, dilakukan pembukaan pada plika
vesikouterina, kemudian kandung kencing disisihkan sejauh mungkin ke kaudal.
• Dilakukan irisan pada segmen bawah rahim kemudian dilebarkan secara tumpul.
• Tangan kiri operator memegang kepala janin (presentasi kepala), mencari kaki janin,
kemudian melakukan ekstraksi (pada presentasi bokong dan letak lintang), setelah janin
lahir dilakukan pemotongan tali pusat (diantara dua klem), muka bayi diusap untuk
membersihkan lender, kemudian janin diserahkan kepada perawat / dokter perinatologi
untuk Resusitasi.
• Plasenta secara manual, kemudian disuntikkan 10 unit Oksitosin intra mural.
• Sudut perdarahan kanan dan kiri diklem, kemudian diikat dengan benang kromik.
• Segmen bawah rahim dijahit dua lapis secara satu-satu atau kros, kemudian plika
vesikouterina dijahit secara jelujur.
• Tampon usus diangkat, kavum abdominal dibersihkan, control perdarahan.

206. Urutan lapisan pas SC?


camper
kutis, subkutis, fascia carpem, fascia scarpae, apponeurosis, m rektus abd, m rektus externus, m
rektus internus, m rektus transversus, fascia transversus, peritoneum
207. kalo SC insisinya apa? vertikal/klasik? tergantung kasus insisi scnya, bagusnya low transverse
trans peritoneal is

pro fund a
208. Fascia pas SC ada apa aja? carpem, scarpae, transversus
camper
209. Bekas SC boleh lahir normal? dipimpin berapa lama? bekas sc boleh lahir normal dengan
syarat dulu low transverse, dan sudah minimal 18 bulan

PCOS
210. kriteria PCOS? oligo/anovulation, hyperandrogenism, polycystic ovaries on ultrasound
US6 → to likeI spt Mutiara 712 buah Ukuran < 1 mm

Mutiara volume ovarian 10cL


di perier spt Kahng ,

minimal 20 buah { haru5 Sdh > 8thn post menarche


ESHRE 2018 → to like/

PRESENTASI BAHU
211. Cara melahirkan bahu yang paling bagus menurut kamu apa? Emang ada berapa macem?

ada 3, klasik, lovset, muller, paling bagus lovset, karena paling tinggi angka keberhasilannya
meskipun bayinya besar

212. Kalo di-SC insisinya apa?

Kalo dr. Yuma, insisi low transverse sambil versi luar supaya lebih gampang ngeluarin
bayinya tanpa pakai insisi klasik

213. Konselingnya apa? Konseling ditambah KB à 3 tujuan KB apa (contoh masing2 tujuan?)

I. Menunda kehamilan à paling bagus pil karena kalau ingin hamil lagi bisa cepat baliknya,
kalau suntik buat balik hamilnya 6 bulan
II. Menjarakkan kehamilan à IUD. Ada 2 : levonogestrel à efek progesterone 3 tahun,
copper T bisa sampai 10 tahun. tipe IUD apa aja?
III. Meniadakan kehamilan à steril. Syarat: ada anak yang umurnya > 5 tahun, jumlah anak
>3

SUNGSANG
214. Macam-macam letak sungsang? komplit, inkomplit termasuk frank dan footling

215. Cara paling bagus untuk lahirin sungsang? cara lovset, karena minimal resiko infeksinya

216. Maneuver sungsang secara garis besar dan detail? maneuver spontan dimana semua
menggunakan tenaga ibu, maneuver partial dimana penolong membantu dalam melahirkan bahu
dan kepala, serta maneuver total dimana peran penolong 100% dalam melahirkan bayi
217. Mana macam sungsang yang boleh PSP? frank saja menurut William, frank dan footling
menurut dr andrie

218. Cara melahirkan kepala apa aja?


Cara yang paling bagus? Alasan?
Tekniknya gimance?

- Mauriceau manuver : Resiko cedera


bayi lebih minim, resiko infeksi
paling rendah.
Caranya : tangan penolong yang
sesuai muka janin masuk ke jalan
lahir à jari tengah masuk ke mulut,
telunjuk dan jari keempat
mencengkam fosa kanina, jari lain
mencengkam leher à badan anak
diletakkan di atas lengan bawah
penolong à kedua tangan penolong
menarik kepala janin curam ke
bawah, asisten melakukan ekspresi
Kristeller à bila suboksiput tampak
di bawah simfisis, kepala janin di
elevasi ke atas
- Naujoks manuver à dilakukan bila
kepala masih tinggi, resiko : trauma
berat di sumsum tulang leher
Caranya : kedua tangan penolong
mencengkam leher janin dari depan
dan belakang à kedua tangan
penolong menarik bahu curam ke
bawah + asisten mendorong kepala
janin kea rah bawah.
- Prague terbalik : kalo oksiput dengan
UUK di belakang sakrum, dan muka
janin menghadap simfisis
Caranya : satu tangan penolong
mencengkam leher dari arah bawah
dan punggung janin diletakkan pada
telapak tangan penolong à tangan
penolong yang lain memegang
kedua pergelangan kaki à kaki janin
ditarik ke atas bersamaan dengan
tarikan pada bahu janin à perut
janin mendekati perut ibu
- Cunam piper
Caranya : asisten megang badan janin pada
kedua kaki, kedua lengan janin di punggung
janin à badan janin elevasi ke atas sehingga
punggung janin deket dengan punggung ibu à
pasang cunam piper dari arah bawah (sejajar
dengan pelipatan paha belakang) à setelah
suboksiput tampak di bawah simfisis, cunam
dielevasi ke atas

219. Perlu kasi oksitosin? perlu, 2-5 IU IM, saat


bokong sudah crowning untuk merangsang
kontraksi uterus agar fase cepatnya selesai dalam
2 his berikutnya.

220. Bisa dikerjain sendiri gak pimpinan persalinannya? Kalo pake asisten ngapain asistennya?
bisa, tapi kalau ada asisten bisa bantu untuk mendorong fundus untuk mempercepat persalinan,
jaga kepala tetep fleksi dan cegah tangan tertinggal
221. Fase persalinan sungsang? [fase lambat, fase cepat] Gimana tuh ceritanya?

- Tahap pertama/fase lambat à mulai bokong hingga pusat (skapula depan). disebut fase lambat
karena melahirkan bokong yang tidak berbahaya
- Tahap kedua/fase cepat à mulai dari pusat hingga mulut. Perlu persalinan cepat pada fase ini
karena kepala janin sudah mulai masuk PAP, mungkin terjadi tali pusat terjepit. Kalo mulut udah
lahir, janin bisa napas lewat mulut
- Tahap ketiga/fase lambat àmulai dari mulut hingga seluruh kepala. Diperlukan persalinan
lambat pada fase ini karena transisi dari dalam uterus [tekanan tinggi] ke dunia luar [tekanan
rendah] à mencegah ICH karna rupture tentorium serebelli

222. Faktor risiko sungsang? hidramnion, anencephaly, plasenta previa, pre-term, adanya kelainan
anatomis uteus, massa di uterus

Dari Williams : Early gestational age, abnormal amniotic fluid, multifetal gestation, hydrocephaly,
anencephaly, uterine anomalies, placenta previa, fundal placental implantation, pelvic tumor, high
parity with uterine relaxation, prior breech delivery, prior caesarean section

223. Penyebab footling presentation? tumor jalan lahir, anomaly jalan lahir, anensefali janin, letak
plasenta di bawah

CA SERVIKS
224 & 225. Staging detail? CIN itu apa?

CIN 1 – 1/3 lapisan bawah cervix; CIN 2 – 2/3 lapisan bawah; CIN 3 - >2/3 lapisan bawah
226. Terapi? loop electrosurgical excision

227. Vaksin HPV?


228. Pap smear?

Dari checklist SL :
1. informed consent
2. minta pasien ke wc untuk pipis kosongin kandung kemih
3. minta pasien litotomi, idupin lampu
4. siapin alat : sabun, air, anduk, speculum, spatula Ayre, cytobrush, tempat sampah medis
dan non medis , larutan hipoklorit, object glass, sarung tangan steril, alcohol 95%, ovum
klem, baskom berisi larutan klorin 0,5%
5. cuci tangan secara aseptik, lalu pakai sarung tangan steril
6. inspeksi daerah vulvovaginal dan perineum lalu bersihkan dengan kasa steril dan larutan
antiseptic
7. dengan jari ketiga tangan kiri dan ibu jari, sisihkan labium minora kearah lateral
8. pasang speculum dengan sudut 45 derajat. Lalu kunci speculum nya. Lalu cek vulva dan
sekitarnya, dinding vagina (fluor albus, rugae vaginalis), portio vaginalis cervicis uteri
(posisi, bentuk, erosi, radang, polip, tumor, ulkus, mudah berdarah)
9. jika sekret vagina banyak à bersihkan dengan kassa steril dengan ovum klem
10. ambil sampel endoserviks dengan cytobrush à masukkan ke kanalis servikalis ± 2 cm,
putar sesuai dengan jarum jam ±4-5 kali. Keluarkan dari lumen dengan hati-hati
11. oles ke object glass secara searah
12. ambil sampel ektocerviks dengan memakai spatula Ayre yang diputar 360 derajat (lakukan
4-5 kali) , dengan mengusahakan pengambilan pertama pada portio di daerah zona
transformasi. Oles ke object glass
13. fiksasi sampel dengan etilalkohol 95% dan biarkan kering
14. lepas speculum
15. selesai

229. Misal pada pap smear didapatkan hasil dysplasia ringan, apa yang harus dilakukan? [pap
smear ulang 3-6 bulan] Kenapa gitu? [karena bisa regresi jadi gak perlu takut. 99% regresi] Dysplasia
ringan > cek ulang 6 bulan lagi untuk lihat ilang atau tambah parah

230.Tipe penyebab kanker pada HPV apa aja? HPV tipe apa yang bisa bikin kanker? jinak 6,11 dan
ganas 16,18 [paling banyak HPV 16 & 18 (70%)]

EMBRIOTOMI
231. Jenis embriotomi? Dilakukan pada kasus apa? dilakukan pada IUFD, yakni Embriotomi –
potong bagian tubuh, dekapitasi – patahin kepala, kleidotomi – patahin klavikula, eviserasi – rusak
dinding abdomen atau toraks, spndilotomi – potong tulang belakang, pungsi – kluarin cairan tubuh

Persalinan buatan dengan cara merusak atau memotong bagian tubuh janin agar dapat lahir spontan
a. Kraniotomi : suatu tindakan yang memperkecil ukuran kepala janin dengan melubangi tengkorak
janin dan mengeluarkan isi tengkorak
- Perforasi : perforatoar Niegel, perforatoar Siebold
- Kranioklasi : cunam Muzeaux, kranioklas Braun
b. Dekapitasi : tindakan untuk memisahkan kepala janin dari tubuhnya dengan memotong leher
janin
- Pengait Braun + gunting Siebold, gergaji Gigli
c. Kleidotomi : tindakan untuk memotong/mematahkan 1-2 klavikula, guna memperkecil diameter
biakromial
- Gunting Siebold
d. Eviserasi atau eksenterasi : tindakan merusak dinding abdomen/thorax dan untuk mengeluarkan
organ-organ viscera
e. Spondilotomi : tindakan memotong ruas-ruas tulang belakang à gunting Siebold
f. Pungsi : tindakan mengeluarkan cairan dari tubuh janin

232. Gunting seabold? gunting seabold untuk embriotomi, dekapitasi, kleidotomi, bisa potong
tulang dan jaringan lunak

233. alat untuk dekapitasi?

pengait Braun
gunting Siebold
gergaji Giggli

234. Alat2 embriotomi? sama kayak 233 + Pisau bedah (scalpel), Perforator SIMPSON, Kranioklast,
Cunam BOER, Cunam Mouzeaux, syringe pungsi

TIROID KEHAMILAN

HYPEREMESIS GRAVIDARUM – grading, PP


Sumber dari Buku merah :
Grading dibagi 3 :
- Grade I à muntah terus menerus, intoleransi makanan-minuman, BB turun, nyeri epigastrium,
muntah (makanan à lendir + cairan empedu à darah), Nadi > 100 bpm, TD sistolik turun, mata
cekung, lidah kering, turgor kulit kurang, urin sedikit tapi masih normal
- Grade II à semua yang dimakan ato diminum dimuntahin, haus hebat, subfebril, nadi > 100 –
140 bpm, TD sistolik dibawah 80 mmHg, apatis, kulit pucat, lidah kotor, kadang icterus, aseton
dan bilirubin urin (+), BB turun cepat
- Grade III à delirium-koma, muntah kurang atau berhenti, tapi icterus, sianosis, nystagmus, ggn
jantung, bilirubin dan protein urin (+)

PP yang dapat dilakukan : darah rutin (ht dan hb bisa naik karena hemokonsentrasi), elektrolit
(hipoNa, hipoK). Serum β-HCG kuantitatif. Urinalisis à badan keton, proteinuria. USG : untuk DD
dengan multifetal gestation atau mola

CORNUOSTOMY
KISTA OVARIUM
235. Kista dan tumor ovarium? Kista ovarium adalah kantong berisi cairan yang terbentuk di dalam
ovarium. Sedangkan tumor ovarium adalah kanker yang tumbuh pada indung telur atau ovarium.
Jadi sau padat satu cair

236. Klasifikasi/jenis kista ovarium?


Pembagian kista ovarium berdasarkan non neoplastik dan neoplastik yaitu:
a. Non Neoplastik
1) Kista folikel: Kista ini berasal dari folikel de graaf yang tidak sampai berovulasi,
namun tumbuh terus menjadi kista folikel, atau dari beberapa folikel primer
yang setelah bertumbuh di bawah pengaruh estrogen tidak mengalami proses
atresia yang lazim, melainkan membesar menjadi kista. Biasanya di dapati
beberapa kista dengan diameter kista 1-1,5 cm. Kista yang berdiri sendiri sebesar
jeruk nipis. Cairan di dalam kista jernih dan mengandung estrogen, oleh sebab itu
jenis kista ini sering mengganggu siklus menstruasi. Kista folikel ini lambat laun
mengecil dan menghilang spontan. Gambar 1. Kista folikel (Wiknjosastro, 1999)
2) Kista korpus luteum: Dalam keadaan normal korpus luteum lambat laun mengecil
dan menjadi korpus albikans, kadang-kadang korpus luteum mempertahankan diri
(korpus luteum persisten), perdarahan yang sering terjadi di dalamnya
menyebabkan terjadinya kista, berisi cairan yang berwarna merah coklat karena
darah tua. Frekuensi kista luteum lebih jarang dari pada kista folikel, dan yang
pertama bisa lebih besar dari yang kedua.
3) Kista teka lutein: Biasanya terjadi pada mola hidrosa, koriokarsinoma, dan kadang
– kadang tanpa adanya kelainan tertentu, ovarium dapat membesar menjadi
kistik. Kista biasanya bilateral dan bisa menjadi sebesar tinju. Pada pemeriksaan
mikroskopik terlihat luteinisasi sel-sel teka. Sel-sel granulosa dapat pula
menunjukkan luteinisasi, akan tetapi sering kali sel-sel menghilang karena atresia.
Tumbuhnya kista ini adalah pengaruh hormon koriogonadotropin yang
berlebihan, dan dengan hilangnya mola atau koriokarsinoma, ovarium mengecil
spontan.
4) Kista inklusi germinal: Biasanya terjadi karena invaginasi dan isolasi bagian-bagian
kecil dari epitel germinativum pada permukaan ovarium, besarnya jarang
melebihi diameter 1 cm. Kista ini biasanya kebetulan di temukan pada
pemeriksaan histologi ovarium yang diangkat sewaktu operasi.
5) Kista endometrium: Kista ini endometriosis yang berlokasi di ovarium.
6) Kista stein-levental: Biasanya kedua ovarium membesar dan bersifat polikistik,
permukaan licin, kapsul ovarium menebal dan tampak tunika yang tebal dan fibrotik
pada pemeriksaan mikroskopis.

b. Neoplastik
1) Kistoma ovarii simpleks: Kista ini memiliki permukaan rata dan halus, biasanya
bertangkai, seringkali bilateral, dan dapat menjadi besar, dinding kista tipis dan
cairan dalam kista jernih, terus berwarna kuning.
2) Kistadenoma ovarii musinosum: Kemungkinan berasal dari suatu teratoma
dimana didalam pertumbuhannya satu elemen mengalahkan elemen lain. Tumor ini
mempunyai bentuk bulat, ovoid tidak teratur, dengan permukaaan rata berwarna
putih kebiru-biruan. Gambar 2. Kistadenoma ovarii musinosum (Wiknjosastro, 1999)
3) Kistadenoma ovarii serosum: Berasal dari epitel permukaan ovarium, dinding
luarnya dapat menyerupai kista musinosum. Dinding dalam kista sangat licin,
sehingga pada kista yang kecil sukar dibedakan dengan kista folikel biasa.
4) Kista endometrioid: Kista ini biasanya unilateral dengan permukaan licin, pada
dinding dalam terdapat satu lapisan sel yang menyerupai lapisan epitel
endometrium.
5) Kista dermoid: Suatu teratoma kistik yang jinak dimana struktur ektodermal
dengan diferensiasi sempurna, seperti epitel kulit, rambut, gigi, dan produk
glandula sebacea berwarna putih kekuningan menyerupai lemak.

237. Ciri kista ovarium ganas ada di 236

238. Kista ovarium terpuntir


I. Klasifikasi kista ovarium fungsional, benign neoplastic dan non-neoplastic,
malignant, gambaran klinis, gambaran USG, terapi
II. Gambaran, sebutkan, jelaskan, semua jenis-jenis dari nyeri abdomen
III. DD/? [intususepsi, bedanya nyerinya gimana]

239. Yang cenderung dari benign ke ganas kista ovarium yang mana?

240. Perbedaan tumor ovarium jinak dan ganas?

241. Staging Ca ovarium dan tatalaksananya?

OPERATIVE OBSTETRIK
242. Kamu tau gunting seabold untuk apa? Caranya gimana? seabold untuk potong bagian keras
dan lunak janin seperti tulang dan otot (EMBRIOTOMI)

243. Pengait braun untuk apa? pengait braun untuk menarik leher janin keluar

244. 5 macam op steril? Yang paling bagus apa dan kenapa?

245. Cara steriliasi? bisa dengan UCHIDA, IRWING, MADLENER, KROENERD, PARKLAND > dasarnya
semua tuba dipotong jadi bikin tubanya diskontinuitas
- UCHIDA :
• Tuba ditarik ke luar abdomen melalui insisi kecil di atas simfisis pubis à suntikkan di daerah
ampula tuba dengan larutan adrenalin di bawah serosa tuba à mesosalphinx
menggembung à pada bagian menggembung disayat à tuba dibebaskan 4-5 cm, dijepit,
diikat lalu digunting à ujung proksimal tertanam dengan sendirinya di bawah serosa à
ujung distal dibiarkan di luar serosa
- IRWING :
• Tuba dipotong diantara 2 ikatan benang à bagian proksimal tuba ditanam ke dalam
myometrium dan bagian distal ditanam ke dalam ligamentum latum
- MADLENER
- KROENERD :
• Bagian fimbrae diikat dengan benang à seluruh fimbrae dipotong
- PARKLAND

PROLAPS UTERI GRADE II


246. Kriteria prolapse uteri?

• feeling like you're


sitting on a ball.
• vaginal bleeding.
• increased discharge.
• problems with sexual
intercourse.
• seeing the uterus or
cervix coming out of
the vagina.
• a pulling or heavy
feeling in the pelvis.
• constipation.
• recurrent bladder
infections.

247. di papsmear ga?

248. derajat prolapse uteri


249. curiga ca corpus uteri bisa di D&C atau nggak? Tergantung kasusnya

250. Beda prolapse uteri dan elongasi coli? ada jawabannya kalo elongasi coli otot dan ligament
kalo prolapse otot aja

251. tatalaksana? HT transvaginal? Gak corporaphy?

Sumber:

• Ingrid Julia (Ponti) – G1P0A0, 31 tahun, gravid 38-39 minggu menurut HPHT, belum in partu, dengan
impending pre-eklampsia, JTHIU, presentasi kepala
• Joey (Sukabumi) – G2P0A0, 17 tahun, gravid 37-38 minggu menurut HPHT, inpartu kala 1 fase laten, riwayat
KPD >12 jam, JTHIU, presentasi kepala
• Jessica Ariesta (Sukabumi) – G3P2A0, 29 tahun, 38-39 minggu menurut HPHT, inpartu, kala 1 fase aktif,
JTHIU, presentasi bahu
• Riyanti Teresa Arifin (Sukabumi) – G1P0A0, 20 tahun, 38-39 minggu menurut USG TM1, dengan hepatitis B
kronis, JTHIU, presetansi kepala [KASUS JARANG skip baca bucur]
• Jessica Novia – G1P0A0, 29 tahun, 28-29 minggu menurut HPHT, sudah inpartu, dengan KPD dan
korioamnionitis + oligohidramnion, JTHIU, presentasi kepala
• Rhevensa (Sukabumi) – G5P0A0, 37 tahun, 38-39 minggu, belum inpartu, dengan CHF NYHA FC II, JTHIU,
presentasi kepala [KASUS JARANG skip baca bucur]
• Angela Djuanedi (Ponti) – 37 tahun, 38-39 minggu menurut HPHT, belum inpartu, dengan KPD, JTHIU,
presentasi kepala
• Angga (Ponti) – G2P1A0, 39 tahun, gravid 36 minggu, belum inpartu, dengan HELLP syndrome, riwayat SC,
JTHIU, presentasi kepala
• Monica Aprilyanti (Bandung) – G2P1A0, 31-32 minggu, dengan KPD, JTHIU, presentasi kepala
• Vincent (Jogja) – 28 tahun, G1P0A0, gravid 37 minggu, dengan PEB, JTHIU, presentasi kepala
• Yovita Gotama (AFO) – G1P0A0, 16 tahun, gravid 42-43 minggu menurut HPHT, sudah inpartu, kala 1 fase
laten, dengan HT gestational, JTHIU, presentasi kepala, dengan gawat janin
• Stephanie Elaine (Sukabumi) – 28 tahun, G2P0A1, gravid 37-38 minggu menurut HPHT, dengan KPD, JTHIU,
presentasi kepala
• Putra Stefano (Sukabumi) – 24 tahun, G1P0A0, gravid 31-32 minggu menurut HPHT, dengan KPD, JTHIU,
presentasi kepala
• Irene Rosali (Bandung) = G2P1A0, 35 tahun, 39-40 minggu menurut HPHT, belum inpartu, dengan CPD dan
riwayat SC, JTHIU, presentasi kepala
• Koas tag-an Ponka = G2P1A0, 38-39 minggu menurut HPHT, belum inpartu, dengan overt DM dan riwayat
SC, JTHIU, presentasi kepala
• Steven Johanes = G3P1A1, 34 tahun, gravid 38-39 minggu menurut HPHT, belum inpartu, dengan riwayat
SC, JTHIU, presentasi bokong
• Dicky Adrian (Sukabumi) = G3P2A0, 21 tahun, 38-39 minggu menurut HPHT, inpartu, kala 1 fase aktif,
dengan PEB, riwayat SC 1x, JTHIU, presentasi kepala, dengan gawat janin
• Regina Michele (Pontianak) = Plasenta previa
• Alansan Julio Sutanto = KET
• Felsus = 38 tahun, G5P2A2, gravid 38-39 minggu menurut HPHT,dengan Plasenta Previa Totalis, dengan
JTHIU, presentasi kepala
• Ajeng Hana Anjani = G1P0A0, 19 tahun, gravid 38-39 minggu menurut HPHT, inpartu, kala 1 fase laten,
dengan PEB, dengan JTHIU, presentasi kepala
• ANONIM = P3A2, 65 tahun, dengan prolapse uteri grade II [jarang beud]
• Michael Lie – G3P2A0, 31 tahun, 38-39 minggu, menurut HPHT, dengan protracted phase, suspek hipertensi
gestational, JTHIU, footling presentation
• Yuri = G3P1A1, 37 tahun, 37-38 minggu, belum inpartu, PEB, JTHIU, presentasi kepala
• Sisca = G2P1A0, 29 tahun, 37-38 minggu menurut HPHT, inpartu, kala II, JTHIU, presentasi bokong
• CF =G1P0A0, 19 tahun, 17-18 minggu menurut HPHT, abortus inkomplit
• Anastasia Michelle = G2P1A0, 30 tahun, gravid 8 minggu menurut HPHT, dengan KET
• Suster (Plasenta Previa), Melina (SC a.i letak lintang & CPD + Hipertensi Kronis), Jojo (KPD), Yunita
(sungsang)
• Belinda Anabel (Bandung) = G2P1A0, 31 tahun, gravid 38039 minggu menurut HPHT, sudah inpartu, kala 1
fase laten dengan riwayat KPD & SC, JTHIU, presentasi kepala.
• Fenyta, Felsur, Davalia, Merry
• Gabrielle Glenis (AFO) = G3P2A0, 32 tahun, gravid 6-7 minggu, dengan KET
• Febriana (Jogja) = G1P0A0, 30 tahun, gravid 38-39 minggu menurut HPHT, inpartu, kala 1 fase laten dengan
CPD, JTHIU, presentasi kepala
• Indah Wati Wjaya (AFO) – G3P2A0, 38 tahun, 11-12 minggu menurut HPHT, dengan perdarahan
pervaginam e.c mola hidatidosa komplit
• Cindy Amadea (Sukbum) – G5P2A2, 38 tahun, 28-29 minggu menurut HPHT, belum inpartu, dengan
antepartum hemorrhage e.c plasenta previa totalis dengan JTHIU, presentasi kepala
• Elda Prisca – G1P0A0, 34 tahun, gravid 39-40 minggu menurut HPHT, dengan KPD, JTHIU, janin tunggal
hidup intrauterine, presentasi kepala
• Fidia Tania (Pontianak) = G1P0A0, 18 tahun, 37-38 minggu menurut HPHT, dengan kala II memanjang,
riwayat KPD, JTHIU, presentasi kepala (di tag karena gakbisa jawab pertanyaan TL)
• Silvia Kartika (Bandung) = G2P1A0, 32 tahun, 37-38 minggu menurut HPHT, belum inpartu, dengan plasenta
previa totalis, JTHIU, presentasi kepala
• Gabriel Budianto (Sukabumi) = G4P2A1, 33 tahun, 39-40 minggu menurut HPHT, dengan PEB dan riwayat
SC, JTHIU, presentasi kepala [ditag karena gakbisa jawab pertanyaan TL]
• Shannia Tritama = GPP2A1, 29 tahun, 38-39 minggu, dengan KPD dan oligohidramnion, JTHIU, presentasi
kepala
• Christinawati Angelita Pusparani (Sukabumi) = P1A0, 41 tahun, dengan kista ovarium terpuntir
• Yosephine Ivanda T (Sukabumi) = G1P0A0, 24 tahun gravid 11-12 minggu menurut HPHT, dengan
perdarahan pervaginam e.c suspek mola hidatidosa
• ANONIM = 41 tahun, gravid 34-35 minggu menurut USG, belum inpartu, dengan KPD dan PEB, JTHIU,
presentasi kepala
OBSTETRICS AND GYNECOLOGY
HIGH-YIELD NOTES

2021

Kehamilan Normal
1. Apa saja tanda pasti dan tanda tidak pasti kehamilan?
Tanda pasti kehamilan:
● Denyut jantung janin
● Gambaran USG Janin
● Gerakan Janin
Tanda tidak pasti kehamilan adalah perubahan anatomic dan fisiologik yang
dapat dikenali oleh pemeriksa:
● Pembesaran uterus dan perut
● Chadwick sign – perubahan warna kebiruan atau keunguan pada vulva,
vagina dan serviks
● Piskacek sign - asimetri uterus, karena implantasi dari blastokis. Bagian
yang membesar biasanya lebih lunak
● Ballottement (+)
● Tes biologis kehamilan (+) - hCG (+) pada 26 hari setelah konsepsi, puncak
60-70 hari dan menurun bertahap dan menetap hingga akhir kehamilan
● Goodell sign – perubahan konsistensi serviks (seperti bibir)
● Hegar sign – pelunakan ismus serviks (diperiksa dengan bimanual exam)
sehingga ujung-ujung jari seakan dapat ditemukan jika isthmus ditekan dari
arah yang berlawanan (pemeriksaan bimanual: jari2 pada forniks anterior
sama abdominal bisa saling keraba). Terjadi pada kehamilan 6-8 minggu.
● Striae gravidarum, linea nigra
● “In early pregnancy, increased vascularity within the cervix stroma beneath
the epithelium creates an ectocervical blue tint that is characteristic of
Chadwick sign. Cervical edema leads to softening-Goodell sign, whereas
isth­ mic softening is Hegar sign.”
Tanda Presumtif:
● Amenorrhea
● Mual & Muntah
● Payudara membesar
● Anorexia
● Lelah
● Pigmentasi pada kulit
● Poliuri
2. ANC itu ada 10T, apa saja?

3. Kapan DJJ bisa dideteksi?


DJJ mulai ada pada minggu ke-4 setelah fertilisasi namun, baru dapat dideteksi
oleh:
● Fetoskop – 20 minggu
● Doppler/usg – 12-20 minggu
Jantung dan sirkulasi mulai terbentuk secara embriologi pada minggu ke 3-4
setelah HPHT. Pada minggu ke-6 setelah HPHT, detak jantung sudah bisa terlihat
di USG, tapi DJJ baru bisa dinilai secara lebih pasti pada 12-20 minggu.
4. Kapan jenis kelamin bisa dinilai?
Organ genitalia sudah berkembang sejak 4 minggu dari HPHT, tapi biasanya baru
bisa dilihat pada periode tumbuh kembang janin karena pada usia gestasi 12
minggu mulai ada tanda-tanda pasti genitalia laki dan perempuan dan pada usia
gestasi 18 minggu, mulai lebih jelas pertumbuhan uterus dan kanalisasi vagina,
20 minggu mulai ada skrotum dan testis.
5. Jelaskan embriologi janin
Dimulai 2 minggu setelah mulai menstruasi = fase ovulasi diikuti dengan
konsepsi/fertilisasi. Dalam periode 2 minggu setelah fertilisasi, zigot akan
berkembang menjadi blastokista. Blastokista akan implantasi kira-kira 6-7 hari
setelah fertilisasi. Blastokista yang adalah 58 sel, 5 nya akan menjadi embryo-
producing cells (inner cell mass), 53 sisanya akan jadi trofoblas plasenta. 4
minggu dari HPHT, atau 2 minggu setelah ovulasi dan fertilisasi (jadi sudah lewat
perkembangan blastokista dan implantasi), dimulailah periode embriogenik
(organogenesis) selama 6 minggu dan diikuti periode tumbuh kembang janin pada
minggu ke 9 dari HPHT hingga akhir kehamilan.

6. Kapan gerakan janin bisa dirasakan oleh ibu?


Gerakan janin bermula pada usia kehamilan 12 minggu, namun baru dapat
dirasakan pada usia kehamilan 16-20 minggu
Multigravida 16-18 minggu
Primigravida 18-20 minggu
7. Kapan USG bisa mulai mengukur GS dan embrio
minggu ke-5, GS itu 1 cm, dan janin 3 mm, udah terdeteksi di USG.
8. USG bisa mengukur CRL hingga kapan? Sehingga kita harus pindah
mengukur BPD, FL, dll
CRL itu oke dinilai sampai minggu ke-13 (kurleb 8,4cm). 11-12 itu paling akurat
kesalahan perkiraan usia gestasi 4-5 hari. Setelah itu, baru gunakan BPD, HC,
AC, FL
9. Kapan HCG bisa dideteksi
● HCG bisa dideteksi sekitar 26 hari setelah konsepsi, meningkat di usia
kehamilan 30-60 hari.
● Puncak produksi 60-70 hari. Lalu penurunan bertahap
● Menetap hingga akhir kehamilan setelah usia kehamilan 100-130 hari
10. Berapa penambahan berat badan yang ideal pada kehamilan
● BMI normal: 0,4 kg/minggu (total = 11,5 - 16 kg)
● BMI kurang: 0,5 kg/minggu (total = 12,5 - 18 kg)
● BMI berlebih: 0,3 kg/minggu (total = 7 - 11,5 kg)
● Obesitas: total 7 kg
● GemellI: total 16 - 20,5 kg
11. Apa itu Braxton-hicks contractions
Kontraksi yang terjadi akibat peregangan miometrium yang disebabkan oleh
terjadinya pembesaran uterus. Kontraksi bersifat non-ritmik, sporadic, tanpa
disertai rasa nyeri, intensitas tidak dirasakan semakin kuat
12. Perbedaan HIS vs Braxton-Hicks contractions
His Braxton-hicks
● Rasa sakit teratur ● Rasa sakit tidak teratur
● Interval makin pendek ● Interval Panjang
● Semakin lama semakin kuat ● Kekuatan tetap
● Dirasakan paling sakit di daerah ● dirasakan terutama di daerah
punggung perut
● Intensitas semakin kuat kalau ● tak ada perubahan walaupun
penderita berjalan penderita berjalan
● Keluar show ● tidak keluar show
● Serviks membuka dan menipis ● serviks tertutup dan tidak ada
pembukaan
13. Kedudukan janin intrauterine normoposisi, malpoisisi, malpresentasi
● Normoposisi (fisiologi)
o Presentasi belakang kepala dengan UUK di segmen depan (depan,
kanan depan, kiri depan)
● Malposisi
o Presentasi belakang kepala dengan uuk tidak di segmen depan
● Malpresentasi
o Presentasi bukan belakang kepala

Anatomi Jalan Lahir


1. Dinding Abdomen Anterior
a. dermis (arranged transversely, jadi kalau di insisi vertikal → tension lateral
lebih kuat dan lukanya lebih luas)
b. subkutan (superfisial: camper fascia, dominan lemak; dalam: scarpa fascia,
dominan membran)
c. otot :
i. ext oblique
ii. int oblique dan rectus abdominis dan pyramidalis
iii. transversus abdominis
d. peritoneum
e. pendarahan:
i. arteri femoralis (dibawah ligamen inguinal) → a. epigastrika
superfisial, a. iliaka circumflex, a. pudendal eksternal → supply kulit,
subkutan, dinding abdomen anterior dan mons pubis. Yang paling
bermakna saat operasi itu a. epigastrika superfisial, dia beberapa cm
dari midline dan diatas scarpa fascia, jadi harus diidentifikasi dan
dioklusi
ii. arteri iliaka eksterna -> a. epigastrika inferior → supply otot dan
fascia. arteri ini lokasinya biasanya di sekitar batas lateral m. rectus
abdominis dan dibawah aponeurosis dari m. oblique internal dan m.
abdominis transversus.
iii. arteri thoraksika interna → a. epigastrika superior
f. Persarafan:
i. T7-T11
ii. T12
iii. L1
2. Otot Pelvic Floor → menyangga uterus
● M. Spinchter ani internus dan eksternus
● M. levator ani
● M. iskiokoksigeus, iliokoksigeus, koksigeus
● M. konstriktor uretra
● M. transversus perinei profunda dan superfisialis
● M. bulbokavernosus
3. Ligament yang berperan menahan organ genitalia internal
● Lig. Cardinal (mackenrodt) sinistra et dextra - di bawah lig. latum:
terpenting, untuk mencegah uterus tidak turun, mengikat serviks dan
vagina ke lateral dinding pelvis. Berjalan bersama a. dan v. uterina
● Lig. Sakrouterina sin et dext : menahan uterus tidak banyak bergerak. Dari
belakang serviks kiri dan kanan melalui dinding rektum ke os sacrum kiri
kanan INGAT dekat banget ini ligamen dengan ureter!
● Lig. Rotundum sin et dex (round ligament / ligamentum teres uteri) :
menahan uterus dalam posisi antefleksi. Mengikat dari sudut fundus uteri
kiri kanan ke inguinal kiri kanan → SEBENARNYA TIDAK MEMBERIKAN
SUPPORT.
● Lig latum sin et dext (broad ligament) : ligamentum yang meliputi tuba.
Berjalan dari uterus ke arah lateral → SEBENARNYA TIDAK
MEMBERIKAN SUPPORT
● Lig. Infundibulopelvicum / IP ligament / Lig. suspensorium: menahan tuba
fallopi, dari infundibulum ke dinding pelvis Berjalan bersama a. dan v.
ovarika. INGAT dekat banget ini ligamen dengan ureter!
● Lig. ovarii proprium sin et dext: menahan ovarium pada sudut kiri dan
kanan belakang fundus uteri
● Parametrium: jaringan ikat sejajar dan lateral dari uterus (dekat dengan lig.
latum / broad ligament)
● Paracervical tissues: jaringan ikat sejajar dengan serviks
● Paracolpium: jaringan ikat sejajar dengan dinding vagina
● Extras:
a. uterina berasal dari a. iliaka interna, dia jalan masuk ke uterus dari
lig. cardinale, naik ke atas ramus asenden dan ke bawah ramus
descenden.
yang naik ke atas itu berjalan di lateral uterus, masuk ke
dalam membentuk a. arkuata, masuk lagi ke dalam
miometrium sebagai a. radial, masuk lagi pecah cabang a.
basal dan a. spiralis (spiralis ini yang bikin lapisan
endometrium / lapisan fungsional). Selain masuk ke dalam
miometrium dan endometrium, a. uterina juga naik ke atas
untuk ke fundus uteri, nah dia pecah jadi 3 cabang dulu:
● cabang ovarika → anastomosis dengan a. ovarika
● cabang tubal → perdarahi tuba falopi
● cabang fundal → perdarahi fundus uteri
turun ke bawah itu lebih kecil arterinya, dia ke serviks bagian
bawah dan vagina bagian atas (a. cervicovaginalis)
a. ovarika berasal dari aorta, dia jalan masuk lewat IP ligament. terus
bercabang-cabang
jelas masuk ke ovarium
ada yang gak masuk ovarium, dia ke tuba falopi
ada yang berjalan menuju cornu uterus → membentuk
anastomosis dengan a. uterina → memperdarahi uterus yang
masuk k sebagai a. arkuata, a. radial, dll (seperti di atas) ini
yang bikin kalo ada ligasi di a. uterina atau a. iliaka interna
karena perdarahan post partum masif, uterus gak jadi iskemik
karena ada supply dari a. ovarika.
4. Apa saja yang membentuk tulang panggul?
● Os coxae (os illium, os iskium, os pubis) / tulang innominate 2 buah, kiri
dan kanan
● Os sacrum
● Os coccygeus
5. Pelvis mayor vs Pelvis minor
● Pelvis mayor – terletak diatas linea terminalis, disebut juga false pelvis
● Pelvis minor – terletak dibawah linea terminalis, disebut juga true pelvis
6. Apa saja yang membentuk pintu atas panggul dan pintu bawah panggul
Pintu atas panggul (Pelvic inlet)
● Promontorium korpus vertebra sacral 1
● Linea innominata (terminalis)
● Pinggir atas simfisis
Pintu Tengah Panggul (midpelvis)
● Spina ischiadica (distansia interspinosus)
Pintu bawah panggul (Pelvic outlet)
● 2 tuber os iskii
● Ujung os sacrum
● Tepi bawah simfisisis
(buku merah)
7. Kapan dilakukan pemeriksaan pelvimetri dalam?
● Usia 37 minggu karena paparan progesteron yang lama akan melunakkan
jalan lahir, sehingga baru bisa diukur
● Sebelum hamil tidak bisa diukur karena jaringan tidak selunak dan selentur
ketika hamil (terutama saat inpartu)
8. Komponen Pelvimeteri klinis (apa saja yang dinilai saat pelvimetri dalam)
Dinding vagina lurus, tidak “funnel shape”. Harusnya kalau lahiran lurus.
PAP/pelvic inlet:
● Konjugata vera (10-11 cm) – jarak dari pinggir atas simfisis ke
promontorium
● Konjugata Transversa (12,5 -13 cm) – jarak terjauh garis melintang pada
PAP
● Diameter oblikua (13 cm) – Bila ditarik garis dari artikulasio sakroiliaka ke
titik pertemuan antara diameter transvera dan konjugata vera diteruskan
ke linea inominata
● Konjugata diagonalis – pinggir bawah simfisis ke promontorium (konjugata
diagonalis – 1,5 cm = konjugata vera)
Midpelvis:
● Distansia interspinarum - transversal (10 cm)
● Spina ischiadica = tajam atau tumpul (tajam lebih baik karena bidang
gesekan lebih sedikit)
● Diameter sagitallis posterior – midpoint distansia spinarum ke sacrum (5
cm)
● Diameter Anteroposterior – Tepi bawah simfisis ke ujung sacrum (11,5 cm)
● Lengkungan os sakrum
PBP/Pelvic Outlet:
Terdiri dari 2 bidang datar - masing2 berbentuk segitiga
Dibentuk oleh garis antara kedua tuber os iskii dengan ujung satunya os sacrum
Dan bidang satunya lagi dengan ujung bawah simfisi
● Arkus pubis (90 derajat)
● Diameter transversa / Distansia tuberum (10,5 cm)

9. Bagaimana cara mengukur arkus pubis?

10. Anatomi Uterusu

a. endometrium → myometrium → serosa (viscera peritoneum)


b. serosa uterus bagian posterior, membentuk bagian anterior dari cul de sac /
douglas pouch
c. serosa uterus bagian anterior cuman sampe ¾, karena sisanya nempel dengan
vesica urinaria dan hanya dibatasi dengan septum/plica vesicouterina (ini yang di
insisi pas SC dan HT)
d. uterus terdiri atas bagian korpus (atas, besar) dan serviks (bawah, kecil) dan
perbatasan diantaranya adalah isthmus (membentuk SBU)
e. ukuran uterus 6-8 cm di nullipara dan 9-10 cm di multipara
f. berat uterus 60g di nullipara dan >60 gr di multipara


CTG
1. Kapan CTG bisa dilakukan?
Mulai minggu ke -26 saat autonomic nervous system sudah terbentuk.
CTG kalo kata dr astrid 34 minggu baru bisa
2. Indikasi pemeriksaan CTG
● Pada awalnya pemeriksaan CTG dikerjakan saat inpartu
● Akhir2 ini dilakukan pada pemeriksaan antenatal, terutama pada kasus-
kasus dengan faktor risiko terjadi gangguan kesejahteraan janin (hipoksia)
o Hipertensi dalam kehamilan
o Gestasional DM
o Postterm
o IUGR
o PPROM, PROM
o Gerakan janin berkuran
o Kehamilan dengan anemia
o Kehamilan ganda
o Oligohidramnion
o Polihidramnion
o Riwayat obstetri buruk
o Kehamilan dengan penyakit ibu
3. Kriteria Gawat janin

Induksi dan Augmentasi


1. Bishop Score
Digunakan untuk melihat apakah induksi memiliki outcome yang bagus
berdasarkan favorability dari serviks. >= 9 induksi kemungkinan besar berhasil,
<= 4 induksi kemungkinan besar gagal. Bishop score tidak bisa 0, karena ketika
hamil pasti serviks sudah pasti lebih soft konsistensinya jadi minimal 2.

2. Apa faktor lain selain favorability serviks?


● Multipara
● BMI <30
● Panggul tidak sempit, sulit dinilai sebenarnya. Jadi biasanya pakai presentil
hubungan tinggi badan ibu dengan kondisi panggul, biasanya panggul
tidak sempit pada ibu dengan tinggi badan > 140cm
● TBJ <3500 gr
● Usia ibu cenderung muda
3. Langkah induksi
a. Cervical Ripening (preinduksi)
Biasanya pakai analog prostaglandin. PGE2 (Dinoprostone) atau PGE1
(Misoprostol), dua-duanya meningkatkan Bishop Score tapi Misoprostol
lebih sering digunakan karena angka komplikasi SC nya lebih rendah dan
lebih sering tidak membutuhkan oksitosin, meskipun Dinoprostone
memiliki time to delivery lebih rendah. Efek samping Misoprostol seperti
angka kejadian uterine tachysystole (harus kondisi janin, bisa
dengan/tanpa perubahan HR janin)
Misoprostol (Cytotec) tab, 50-100 micro/3-6 jam PO. 25 micro/3-6 jam
vaginally. Sebenarnya Misoprostol itu untuk peptic ulcer, dan dia
digunakan untuk pre-induksi secara “off-label”.
Kalau cara mekanik bisa pakai foley catheter atau pakai dilator seperti
laminaria.
b. Induksi dan Augmentasi
Bisa pakai cara amniotomy, prostaglandin, dan oksitosin. Tapi kalau
prostaglandin itu bukan untuk klinis tapi penelitian, karena dia angka
kejadian uterine tachysystole nya sering banget. Namun memang untuk
induksi pada wanita yang near term dengan PROM atau dengan
favorable serviks, penggunaan prostaglandin oral/vaginal sama efeknya
dengan oksitosin IV.
Ada saran dari Williams: Misoprostol oral 100micro → 6 jam apakah maju
atau tidak → tidak maju, Misoprostol oral 100micro → 6 jam apakah maju
atau tidak → tidak maju atau bahkan ada tachysystole → oksitosin.
Biasanya oksitosin itu digunakan ketika cervical ripening tidak memicu
persalinan dengan adekuat. Yang wajib dicek ketika kasih oksitosin
adalah wajib CTG atau cek manual kontraksi uterus dan DJJ.
4. Dosis oksitosin dan cara pemberiannya sampe itung2an berapa tpm?
10 U dalam 1.000 cc RL!
10 U = 10.000 mU
diencerkan dalam 1.000 cc RL, sehingga diperoleh:
10.000 mU/1.000 cc,
maka 10mu/cc
Contoh:
mau start oksitosin dari 4 mU/ menit 4mU/10 = 0,4 cc/menit
1 cc = 20 tpm
maka, 0,4 cc x 20 tpm = 8 tpm!
Start di 5mU/menit itu sudah high dose, berikut adalah regimen low dan high
dose serta intervalnya.

5. Indikasi & Kontraindikasi induksi


Intinya, indikasi itu ketika benefit terhadap janin atau ibu lebih besar dibandingkan
melanjutkan kehamilan.
● Indikasi Janin
o Kehamilan lewat waktu (post-term)
o Oligohydramnios
o Ketuban pecah dini
o Janin mati
● Indikasi Ibu
o Kehamilan dengan hipertensi
o Kehamilan dengan diabetes mellitus
● Kontraindikasi Janin
o Gemelli
o Malposisi dan malpresentasi janin
o Hidrosefalus berat
o Makrosomia
o susp CPD
● Kontraindikasi Ibu
o Riwayat insisi uterus tertentu sebelumnya → cacat rahim
o Insufisiensi plasenta
o susp CPD
o Panggul sempit
o Grande multipara
o Distensi Rahim berlebih (e.g. hydramnios)
o Plasenta previa
6. Syarat induksi (dengan oksitosin)
● Kehamilan aterm
● Ukuran panggul dalam batas normal
● Taksiran berat janin / ukuran kepala janin dalam batas normal
● Tidak ada CPD
● Janin dalam presentasi kepala
● Serviks sudah matang yaitu porsio teraba lunak, mulai mendatar dan sudah
mulai membuka → Bishop score diatas sama dengan 9
7. Komplikasi pemberian oksitosin drip (komplikasi induksi in general)
● Aktivitas miometrium yang sangat meningkat ’ mempengaruhi
pengaliran O2 ke janin
● Intoksikasi air ’ jika dosis 50 mU/menit dapat bekerja sebagai
antidiuretika
● Ruptur uteri, kalau mau VBAC, tidak disarankan dikasih prostaglandin prior
to induction karena prostaglandin kan selain induksi, juga penipisan serviks
+ peningkatan kontraksi uterus
● Chorioamnionitis (e.c amniotomy)
● PPH
● Atonia uteri
● Seksio sesarea

Persalinan Normal
1. Apa saja tanda dan gejala inpartu?
Kalau kata Williams & buku merah, in labour/inpartu adalah beberapa jam terakhir
kehamilan ditandai dengan adanya kontraksi uterus yang menyebabkan
penipisan/pendataran, dilatasi serviks dan mendorong janin keluar melalui jalan
lahir.
2. Apa saja kriteria persalinan normal
Panggul ukuran normal, bayi ukuran rata-rata, aterm, presentasi verteks, tidak
ada CPD
3. Langkah persalinan normal
Kontraksi uterus → dilatasi serviks dan penipisan/pendataran serviks →
pendorongan janin keluar
4. Aktivitas uterus pada persalinan normal
Bagian fundus yang lebih tebal ototnya dan lebih banyak vaskularisasinya akan
lebih tebal dan lebih kuat kontraksinya, kontraksi juga dimulai dari fundus
kemudian menyebar.
Bagian segmen bawah uterus yang lebih tipis ototnya akan menjadi pasif dan
berubah perlahan semakin tipis hingga memberikan jalan untuk kelahiran.
Terdapat cincin perbatasan kedua segmen tersebut, yaitu cincin retraksi
fisiologis. Kalau macet, dapat berubah cincinnya menjadi patologis, yaitu Bandl’s
Ring.
5. Berapa lama kala I berlangsung
Primipara - 13 jam
Multipara - 7 jam
6. Kapan yang dimaksud dengan kala I fase laten memanjang (partus lama)?
Primipara - 20 jam
Multipara - 14 jam
7. Apa perbedaan kala I pada primipara dan multipara
● Primi – ostium uteri internum akan membuka lebih dahulu, sehingga serviks
akan mendatar dan menipis, kemudian ostium uteri eksternum membuka
● Multigravida – OUI dan OUE sudah sedikit terbuka. Penipisan dan
pendataran serviks berlangsung bersamaan dengan pembukaan
8. Sebutkan Tanda dan gejala kala II
● Dorongan untuk meneran
● Tekanan pada anus
● Perineum menonjol
● Vulva terbuka
9. Kapan harus merujuk jika bayi belum lahir pada kala II?
Primipara – 120 menit, sumber lain bilang 60 menit sudah harus lahir
Multipara – 60 menit, grande multipara (lebih dari 4) 30 menit sudah harus lahir.
kalau gak lahir, rujuk atau augmentasi.
Kala II memanjang karena ibu kelelahan, malposisi, his tidak adekuat, perineum
rigid.
10. Indikasi dan syarat ekstraksi forceps
Indikasi relatif De Lee
- Kepala sudah di dasar panggul
- Putar paksi sudah maksimal
- M. levator anii sudah teregang
Indikasi relatif Pinard
- Kala II memanjang sudah mengejan 2 jam
Indikasi absolut:
- gawat janin
- Penyakit jantung kompensata, penyakit paru fibrotic
- Kelelahan
- Infeksi intrapartum
- Kala II memanjang
- Eclampsia, preeklampsia
Syarat forceps:
- Gaada kontraindikasi PSP (ex CPD)
- Pembukaan lengkap
- Sudah engagement
- Kepala janin bisa dipegang dengan cunam
- Janin hidup
- Ketuban sudah pecah
11. Indikasi dan syarat ekstraksi vakum
Indikasi vakum ibu:
- penyakit jantung kompensata
- kelelahan
- infeksi intrapartum
- kala II memanjang
Indikasi vakum janin:
- gawat janin, non-reassuring
- prolapse tali pusat
- premature separation of plasenta
Syarat vakum:
- pembukaan lebih dari 7 cm (multigravida)
- penurunan kepala janin pada Hodge II
- harus terdapat kontraksi rahim dan tenaga mengejan pada ibu
12. Bagaimana cara melihat kemajuan persalinan
Dilatasi serviks, penurunan kepala. Dicatat pada partograf
Hal ini karena setiap kontraksi akan menyebabkan uterus memanjang menjadi
ovoid → dilatasi serviks dan penipisan/pendataran & penurunan diameter
horizontal atau melurusnya kolumna vertebralis janin
13. Apa yang dilakukan ketika persalinan tidak maju?
Kenali dulu gangguan persalinan tidak maju

Untuk kala I laten memanjang sudah jelas, tidak ada tindakan berarti yang penting
bed rest dan pemantauan kondisi ibu dan janin. Perburukan kondisi ibu dan janin
baru dimulai induksi oksitosin (cek indikasi dan kontraindikasi) atau SC.
Untuk kala I aktif ada yang protracted (atau lebih lambat dari biasanya) dan ada
yang arrested (berhenti total).
Ketika diketahui protracted atau arrested, banyak faktor yang harus
dipertimbangkan sebelum dinyatakan harus SC:
- cek kontraksi uterus, kita lihat dalam CTG 10 menit, ada berapa kali
kontraksi, kemudian tekanan kontraksinya berapa? apakah sudah melebihi
180 Montevido units. Kalau belum, sebenarnya kontraksi uterus masih bisa
ditingkatkan sampai 200 Montevido Units
- kala I laten sudah selesai, ditandai dengan dilatasi serviks 6 cm (guideline
yang terbaru yang direkomendasikan)
- Tidak ada perubahan dilatasi serviks selama 2 jam (atau 4 jam) - akselerasi
(dari 4/6 - 9 cm). Atau ada fase deselerasi kala 1 fase aktif yag memanjang
lebih dari 2 jam - deselerasi (dari 9 - 10 cm)
- Ketika diketahui sudah ada CPD, langsung SC saja. tapi kalau belum, boleh
dicoba induksi oksitosin → kontroversial
- Ketika diberikan oksitosin, dan tidak ada kemajuan (pembukaan tidak
bertambah, HIS tidak adekuat) setelah 6 jam.
14. Seven cardinal movements
1. Engagement
2. Descent
3. Flexion
4. Internal rotation
5. Extension
6. External rotation
7. Expulsion
15. Yang mempengaruhi persalinan normal:
3P + 2P
a. passage: tidak bisa diubah
b. power: his, bisa diinduksi dan diaugmentasi
c. passenger:
i. tunggal/gemelli
ii. intrauterine/lainnya
iii. kesejahteraan janin. DJJ bisa dinilai dengan CTG.
iv. letak atau kedudukan:
1. axis longitudinal fetus terhadap axis longitudinal uterus
2. terdiri atas situs longitudinal, situs transverse, situs oblique.
v. sikap: posisi kepala bayi terhadap tulang belakang fetus (derajat
kepala bayi): fleksi/defleksi/ekstensi
vi. posisi/denominator: posisi kalau dari pemeriksaan leopold =
menentukan letak punggung janin terhadap dinding perut ibu,
sedangkan pada PD untuk menentukan kedudukan salah satu
bagian janin yang terendah pada jalan lahir.
1. UUK untuk presentasi belakang kepala
a. UUK kanan anterior/posterior/transverse
b. UUK kiri anterior/posterior/transverse
c. UUK anterior/posterior

2. Dahi untuk presentasi dahi


3. Dagu/mentum untuk presentasi muka
4. Sakrum untuk presentasi bokong
5. Akromion/skapula untuk presentasi bahu
vii. Presentasi: bagian fetus yang masuk ke panggul
1. presentasi belakang kepala/cephalic/vertex:
i. presentasi belakang kepala dengan
denominator UUK di anterior
(normopresentasi, normoposisi)
ii. presentasi belakang kepala dengan
denominator UUK di posterior
(normopresentasi, malposisi)
iii. presentasi puncak kepala (kepala dalam
defleksi ringan) dengan denominator UUB
(malpresentasi, malposisi)
iv. presentasi dahi (kepala dalam defleksi sedang)
dengan denominator dahi/frontal
(malpresentasi, malposisi)
v. presentasi muka (kepala dalam defleksi
maksimal) dengan denominator dagu/mentum
(malpresentasi, malposisi)
* presentasi ini mempengaruhi besar kepala janin yang
akan masuk ke PAP. Diameter sebelum fleksi itu kalau
normopresentasi adalah diameter oksipitofrontal
(11,5cm) dan ketika fleksi diameternya jadi
suboksipitobregma (9,5cm). Kalau presentasi wajah
kan denominatornya dagu, dia diameternya diameter
supraoksipitomental/verticomental (13cm) dan ketika
fleksi diameternya jadi submentobregma (11,5cm, ada
sumber yang bilang 9,5cm)

b. presentasi bokong:
i. bokong sempurna (complete breech): hip fleksi,
kedua kaki fleksi, kedua kaki di samping
bokong. → TIDAK BISA PSP
ii. bokong murni (frank breech): hip fleksi, knees
ekstensi jadi dekat kepala. kedua tungkai lurus
ke atas → BISA PSP kalau denominator
sakrum anterior
iii. bokong tidak sempurna (incomplete breech):
satu atau kedua hip ekstensi, satu atau kedua
kaki fleksi → TIDAK BISA PSP
1. bokong kaki: tungkai terlipat pada lipat
paha dan lipat lutut
2. kaki: kaki turun ke bawah lebih rendah
dari bokong
3. lutut: lutut turun ke bawah lebih rendah
dari bokong
c. presentasi bahu
d. presentasi majemuk
viii. ukuran:
1. bisa diperkirakan dengan USG. Biasanya yang digunakan itu
TBJ. Meski memang yang lebih mempengaruhi adalah HC
nya.
2. Kalau tidak ada USG, bisa pakai rumus Johnson syaratnya
presentasi kepala:
a. sudah masuk PAP dan tidak bisa digerakkin: (TFU-
11)x155 = ...gr
b. sudah masuk PAP bisa digerakkin: (TFU-12)x155 =
...gr
c. belum masuk PAP: (TFU-13)x155 = ...gr
3. Kalau tidak ada USG, bisa pakai rumus Risanto syaratnya
presentasi sungsang: (125xTFU)-880 = ...gr
Normalnya 2.500-4.000gr (atau 3.500gr).
Bila > 4.000gr, faktor risiko distosia bahu dan CPD
< 2.500gr, jangan pakai vakum bisa prolaps otaknya, lebih baik
pakai forceps
ix. Kedudukan
1. Hodge
a. I → pinggir atas simfisis
b. II → pinggir bawah simfisis
c. III → spina ischiadika
d. IV → os. cocygeus
2. Station
a. 0 → setinggi hodge III
b. -1 s/d -5 → diatas spina ischiadika / hodge III s/d
floating
c. +1 s/d +5 → di bawah spina ischiadika / hodge III s/d
perineum
3. Perlimaan
a. 5/5 → kepala diatas PAP (floating)
b. → bagian terbesar kepala belum masuk PAP, sulit
digerakkan (Hodge I-II)
c. → bagian terbesar kepala belum masuk PAP (Hodge
II-III)
d. → bagian terbesar kepala sudah masuk PAP (Hodge
III+)
e. → bagian kepala di dasar panggul (Hodge III-IV)
f. 0/5 → bagian kepala di perineum (Hodge IV)
d. penolong
e. psikologis
16. Sebutkan tanda his adekuat
● Kontraksi simetris, dominan dari fundus
● Ada periode relaksasi (1-2 menit)
● Amplitudo 40-60 / 60-90 mmHg atau intensitas kuat
● Durasi 60-90 detik
● Menjalar sampai ke pinggang
● Frekuensi 3-5x/10 menit
17. Dengan power yang kuat maka..
dilatasi serviks akan sempurna, fetal head akan turun, caput succedaneum akan
rendah angkanya
18. Kapan harus merujuk jika plasenta belum lahir pada kala III?
Setelah penyuntikan oksitosin 10 unit IM, jika plasenta tidak lahir setelah
penegangan tali pusat terkendali selama 15 menit, ulangi pemberian oksitosin 10
unit IM, ulang lagi 15 menit. Jika tidak lahir setelah 30 menit, segera rujuk.
19. Tanda-tanda lepasnya plasenta
● Fundus uteri naik
● Tali pusat memanjang +/- 3 cm
● Bentuk uterus membulat dan keras
● Kadang-kadang disertai pengeluaran darah dengan tiba-tiba
20. Mekanisme pelepasan plasenta
● Shiny Schultze (sentral)
Hematoma retroplasenta mendorong plasenta menuju rongga uterus,
pertama bagian tengah dan kemudian sisanya
● Dirty Duncan (perifer)
Pemisahan plasenta terjadi pertama kali di perifer, dengan akibat darah
mengumpul di antara membrane dinding uterus dan keluar dari plasenta.
Pada situasi ini plasenta turun ke vagina secara menyamping dan
permukaan ibu adalah sisi yang pertama kali terlihat

21. Kapan masa nifas dimulai dan berakhir


Masa nifas/puerprium dimulai sejak 1 jam setelah lahirnya plasenta sampai
dengan 6 minggu (42 hari) setelah itu
22. Persalinan abnormal
Semua yang selain normal, misalkan malpresentasi, malposisi, gemelli.
Paling banyak ditemukan adalah breech/sungsang. Dinilai dengan Zatuchni-
Andros breech scoring

interpretasi:
<= 3 → SC
4 → evaluasi terutama BB nya
>= 5 → PSP
23. Ruptur perineum dan tatalaksananya
Etiologi ruptur: kepala lahir terlalu cepat, cara memimpin salah, banyak jaringan
parut di perineum , distosia bahu
Teknik menjahit:
a. grade I: menggunakan catgut dengan teknik continuous / figure of 8
b. grade 2: pinggir robekan di klem lalu digunting agar rapih otot perineum
jahit dengan catgut mukosa vagina dijahit dengan catgut dengan terputus
atau continuous (dimulai dari puncak robekan) kulit perineum dengan
benang sutura secara terputus
c. grade 3: otot sphincter ani diklem dengan klem PEAN LURUS jahit 2-3
jahitan dengan catgut kromik lanjut ke langkah grade 2
d. grade 4: mukosa depan rectum dijahit fasia perirectal dan rectovaginal
dijait dengan catgut kromik lanjut ke langkah grade 3 (kalo end to end
teknik liat yang 50 soal)

Persalinan Sungsang
1. Manuver persalinan sungsang? Bagaimana pemilihannya?
2. Syarat PSP presentasi bokong?
Dilatasi/pembukaan serviks harus lengkap
3. Kontraindikasi PSP presentasi bokong?
● Kontraindikasi persalinan pervaginam
● Presentasi kaki dan variannya
● Hiperekstensi kepala janin (stargazing fetus)
● Berat bayi >3600 gram
● Tidak adanya informed consent
● Tidak adanya petugas yang berpengalaman
4. Mengapa cut off TBJ di ZAS 2.500 – 3.500?
karena kontraindikasi tbj >3600 g ???
5. Apakah syarat versi luar?

6. Komplikasi versi luar


● Bradikardia janin sementara
● Solusio plasenta
● Kompresi tali pusat
● Perdarahan feto-maternal
● Ketuban pecah dini
7. Faktor risiko sungsang?
● Poli/oligohydramnion
● Multifetal gestation
● Hydrocephalus
● Anencephaly
● Structural uterine abnormality
● Plasenta previa
● Riwayat sungsang

Sectio Cesaria
1. Mengapa angka SC meningkat?
a. KB → semakin sering ditemukan wanita yang nullipara
b. rata-rata usia wanita yang hamil semakin tua
c. penggunaan monitoring fetal heart rate elektronik meningkat, jadi makin
mudah mendeteksi distress dari gambaran yang abnormal atau meragukan
d. sungsang → disepakati SC
e. preeklamsia → disepakati SC
f. rate keberhasilan VBAC makin lama makin menurun
g. angka obesitas meningkat
h. atas permintaan sendiri, ibu-ibu makin paham akan risiko persalinan
spontan dan makin takut
2. Indikasi SC
a. sisi ibu
i. BSC, terutama yang <18 bulan jarak antar SC karena meningkatkan
angka kegagalan VBAC atau bahkan ruptur uteri hingga 3x lipat
ii. letak plasenta abnormal
iii. riwayat histerotomi, terutama dengan insisi klasik
iv. dehiscence
v. atas permintaan sendiri
b. hubungan ibu - janin
i. CPD
ii. plasenta previa
iii. solusio plasenta
c. sisi janin
i. kesejahteraan janin meragukan
ii. malpresentasi
iii. makrosomia
iv. kelainan kongenital → hidrosefalus, anensefali, dll
3. Timing SC elektif
Jangan sebelum 39 minggu kalau tidak ada indikasi lain.
4. Apa saja yang dilakukan preoperatif?
- sedatif (bahkan may be given 1 malam sebelum)
- puasa 6-8 jam sebelum operasi, boleh minum hingga 2 jam sebelum
operasi
- antasida sebelum anestesi, untuk meminimalisir lung injury karena aspirasi
asam lambung
- cek DJJ di OK sebelum operasi
- cukur, tapi indikasi cukur itu bukan untuk menurunkan infeksi, tapi untuk
menyingkirkan yang mengganggu aja. Cukurnya di clipping, bukan di
shaving.
- kateter → (1) untuk kolaps in vesika urinaria, cegah kena insisi, (2), hindari
retensi urin karena anestesi regional, (3) untuk ukur urine output post op.
Meskipun demikian, risiko ISK jelas meningkat. Ada rekomendasi juga
untuk tidak dikateter kalau wanitanya stabil secara hemodinamika
- jangan lupa tatalaksana untuk kondisi spesifik
- antibiotik profilaksis, direkomendasikan single dose IV b-lactamase, seperti
cephalosporin → 1 gr cefazolin. Antibiotik dapat diberikan lebih banyak
kalau ada perdarahan >1500mL atau operasi >3 jam. Kalau ada riwayat
MRSA, tambahin vancomycin. Kalau alergi
cefazolin/cephalosporin/penicillin, kasihnya clindamycin 600mg +
aminoglikosida dosis sesuai berat badan. idealnya, antibiotik diberikan
60 menit sebelum SC mulai.
- asepsis dan antisepsis dengan clorhexidine atau povidone-iodine di kulit
dinding abdomen
5. Urutan lapisan yang dibuka pas SC + langkah SC
a. Pasien ditidurkan diatas meja operasi dengan sebelumnya diberikan
premedikasi di ruang Persiapan oleh bagian anestesi.
b. Dilakukan anestesi oleh Dokter Anestesi.
c. Dilakukan toilet / tindak asepsis dan antisepsis pada daerah operasi
dengan alkohol 70%, kemudian dengan povidone iodine / clorhexidine
d. Pasien ditutup dengan duk steril kecuali daerah operasi.
e. Dilakukan irisan pada kutis dan subkutis transversal/melintang pada
daerah perut pada level hairline pubis atau biasanya 3 cm diatas batas
superior dari simfisis pubis. Irisan sepanjang 12-15 cm. Pfannenstiel
incision.
f. Irisan secara tajam dilanjutkan hingga subkutis dan fascia. Identifikasi
fascia camper (dominan lemak). Satu irisan dilanjutkan ke fascia scarpa,
akan ada a. epigastrika superfisial, lokasinya di atas fascia scarpa dan
beberapa cm dari midline. bila terlaserasi → cauter.
g. fascia di insisi secara tajam di midline. Biasanya diinsisi itu ada 2 layer:
i. aponeurosis dari m. oblik eksternal
ii. aponeurosis dari m. oblik internal dan m. transversus abdominis
(fused layer)
iii. di daerah ini ada a. epigastrika inferior, bila terlaserasi → cauter
h. setelah itu, batas fascia inferior dijepit dengan Kocher, dan operator
memisahkan lapisan fascia dengan m. rectus abdominis hingga simfisis
pubis
i. batas fascia superior kemudian dijepit dengan Kocher, dan operator
memisahkan lapisan fascia dengan m. rectus abdominis hingga lengkap.
j. bila ada pembuluh darah terlaserasi → cauter
k. separasi fascia ini dilanjutkan ke arah cephalad dan lateral hingga
ditemukan area semisirkuler dengan diameter kurang lebih 8 cm (bisa
diperbesar bila TBJ berat, atau diduga makrosomia.
l. inisisi tajam atau tumpul pada m. rectus abdomnis dan m. pyramidalis dari
superior dan kemudian inferior pada midline.
m. insisi tumpul pada fascia transversalis dan lemak-lemak preperitoneum
n. identifikasi peritoneum. Pada ujung bagian atas dari insisi, identifikasi
peritoneum nya dan insisi tumpul. Ini untuk menurunkan risiko cystotomy.
o. insisi peritoneum (serosa vesikouterina) secara lateral tepat diatas batas
vesika uterina. Insisi kemudian diperpanjang ke superior hingga ke batas
insisi fascia, ke inferior sampai ke batas vesika urinaria.
p. Setelah peritoneum dibuka, pasang tampon usus.
q. Kemudian kandung kencing disisihkan sejauh mungkin ke kaudal. Ini dapat
dilakukan secara tumpul ataupun tajam pada septum vesikouterina
(biasanya tidak lebih dari 5 cm)
r. Identifikasi SBU, dengan palpasi, atau biasanya dibagian yang semula ada
di septum vesikouterina. Dilakukan irisan pada SBU kemudian dilebarkan
secara tumpul, menggunakan tarikan lateral kedua jari telunjuk. jangan
sampai berlebihan ke lateral margin uterus karena bisa kena a. uterina.
Biasanya low transverse hysterotomy ini tidak cukup, jadi bisa sampai ada
extended incision ke vertikal “J”, “U” atau bahkan “T”.
s. Nilai air ketuban: warna, jumlah.
t. Presentasi kepala: tangan kiri operator memegang kepala janin (presentasi
kepala), mencari kaki janin. kepala kemudian diarahkan sedikit ke insisi dan
dapat dilakukan pendorongan fundus transabdominal (dari luar).
u. CPD: kalo coba-coba melepas kepala janin yang impaksi, bisa bikin risiko
extended incision, perdarahan dan fraktur kepala janin meningkat. Jadi ada
3 pendekatan:
i. dorong kepala janin dari vagina ke arah atas, sehingga kepala janin
lepas dari CPD nya
ii. tarik kaki janin ke atas, meski agak susah karena ruangnya sempit,
ini juga meningkatkan resiko extended incision ke arah vertikal.
iii. “fetal pillow”, jadi dimasukin balon intravaginal sehingga kepala bayi
terdorong ke atas.
v. belum inpartu: biasanya sulit karena kepala janin masih “unmolded” jadi
masih bundar, dan tidak punya “ujung” cephalic, jadi susah untuk
diarahkan ke insisi. apalagi kalau belum inpartu, SBU masih tebal.
Direkomendasikan pakai ekstraksi forceps atau vakum.
w. lahirkan kepala kemudian jari-jari operator letakkan di leher janin, cari
apakah ada lilitan tali pusat, dan lepaskan melalui kepala.
x. kepala janin kemudian diputar ke posisi transversal oksiput. genggam
dengan lembut kepala janin. traksi ke bawah untuk melahirkan pundak
anterior, kemudian traksi ke atas untuk melahirkan pundak posterior.
JANGAN KERAS karena bisa cedera pleksus brakhialis. Kemudian lahirkan
badan janin.
y. Klem dan potong tali pusat, bersamaan dengan muka bayi diusap untuk
membersihkan lender, kemudian janin diserahkan kepada perawat / dokter
perinatologi untuk Resusitasi.
z. Suntik oksitosin 20U/L kristaloid, rate 10mL/menit. Atau 10U IM
aa. Nilai kontraksi uterus, bila cukup maka traksi tali plasenta dan lahirkan
plasenta
bb. Bersihkan kavitas uterus dengan suction, sponge, kasa.
cc. Sudut perdarahan kanan dan kiri diklem, kemudian diikat dengan benang
kromik (ligasi terpisah), atau dapat juga dijahit bersamaan dengan insisi
SBU nya secara kontinu
dd. salah satu sudut insisi dijahit kemudian dilanjutkan dengan menjahit
miometrium secara continuous lockring hingga melewati sudut insisi
seberangnya.
ee. perhatikan apakah diperlukan jahitan tambahan pada area yang masih
terdapat perdarahan, terutama sudut insisi.
ff. Tampon usus diangkat
gg. kavum abdominal dibersihkan, bisa di irigasi
hh. jahit peritoneum, jahit m. rectus abdominis, jahit fascia, jahit subkutis, dan
jahit kutis secara subkutikuler
6. Jenis insisi SC
7. perawatan post-op SC
a. evaluasi hemodinamika
b. capai euvolemia
c. palpasi uterus dan nilai kontraksi → nilai apakah terdapat perdarahan / lokia
d. nilai urine output, bila <30mL/jam hati-hati apakah terdapat perdarahan
atau efek antidiuretik dari oksitosin
e. nilai fungsi pencernaan: apakah terdapat distensi? sudah bisa kentut?
sudah bisa BAB?
f. edukasi untuk mobilisasi dini misalkan berjalan ke kamar mandi →
menurunkan risiko tromboemboli vena
g. dressing luka bekas SC 24 jam post SC dan inspeksi insisinya setiap hari.


Bekas SC
1. Apa beda VBAC dan TOLAC?
VBAC itu berhasil, karena dia kata benda. TOLAC itu proses atau rencana
tatalaksana, karena dia kata kerja. Jadi TOLAC itu bertujuan untuk mencapai goal
yaitu VBAC, ketika gagal maka dinyatakan gagal induksi atau “failed trial of labor”,
sehingga opsi berikutnya adalah “elective repeat cesarean delivery” atau ECRD
2. Apa risiko dilakukannya TOLAC?
Risiko ibu:
a. ruptur uteri komplit, dan inkomplit (dehiscence)
b. mortalitas ibu
Risiko janin dan neonatus
a. mortalitas perinatal
b. Hypoxic ischemic encephalopathy (HIE)
c. transient tachypnea
3. Siapa saja kandidat TOLAC?
a. tipe insisi SC sebelumnya. Jelas yang terbaik adalah low-transverse
tunggal (0,2-0,9%). Kalau vertical, yang rendah ya low-vertical (1-9%),
sedangkan kalau classical (2-9%) dan T-shaped (4-9%) itu tinggi banget
risiko rupturnya, bahkan dianggap sebagai kontraindikasi TOLAC. Ada juga
penelitian yang bilang kalau sebelum aterm saja, sudah bisa ruptur.
b. tipe jahitan SC sebelumnya. Sebenarnya ini faktor yang kurang
menentukan karena beberapa studi menunjukkan tidak adanya perbedaan
pada kejadian dehiscence atau ruptur. Tapi tetap direkomendasikan
jahitan dengan locking suture single-layer.
c. jumlah BSC sebelumnya. BSC 2x dibandingkan BSC 1x, risiko rupturnya
2-3k lipat.
d. riwayat ruptur uterus sebelumnya. Kalau ada riwayat, bisa 9-32% lebih
mungkin terjadi ruptur berulang
e. jarak SC 1 dengan SC berikutnya. Kalau ≤ 18 bulan, risiko ruptur 3x lipat
lebih tinggi dibandingkan > 18 bulan. Studi lain ada juga yang melaporkan
< 6 bulan, risiko rupturnya lebih tinggi lagi, hingga 9x > 18 bulan, atau 3x ≥
6 bulan.
f. riwayat PSP. Jelas kalau ada riwayat PSP, semakin tinggi kemungkinan
tercapai VBAC
g. indikasi SC sebelumnya:
i. malpresentasi: sungsang. 90% VBAC
ii. fetal distress: 80% VBAC
h. Ukuran janin. Ini faktor yang sebenarnya tidak begitu meningkat signifikan
i. <4000 gram -> 1% ruptur uteri
ii. >4000 gram -> 1,6% ruptur uteri
iii. >4250 gram -> 2,4% ruptur uteri
i. Gemelli tidak meningkatkan risiko ruptur uteri
j. Obesitas maternal.
i. BMI normal -> 85% VBAC
ii. BMI 25-30 -> 78% VBAC
iii. BMI 30-50 -> 70% VBAC
iv. BMI ≥ 40 -> 61% VBAC
k. Ada juga scoring dengan VBAC Score
4. Jadi apa kontraindikasi TOLAC?
a. Kontraindikasi VBAC
b. Kontraindikasi PSP secara umum
c. Prior classical incision
d. Jenis luka T terbalik or unknown incision
e. Luka parut pada otot rahim di luar SBU
f. Riwayat rupture uterus
g. Kontraindikasi relative, panggul sempit relative
h. Dua atau lebih luka parut transversal di SBU
i. Kehamilan ganda
j. jarak <18 bulan
k. riwayat SC > 2 kali
5. Kalau dia kandidat TOLAC, kapan sebaiknya dilakukan?
a. tunda TOLAC kalau belum 39 minggu, ERCD juga seperti itu
b. USG saat usia kehamilan 20 minggu juga harus mendukung usia kehamilan
saat ini > 39 minggu
c. DJJ sudah terdeteksi selama 30 minggu dengan USG doppler
d. b-hCG sudah positif selama > 36 minggu
6. Apakah induksi boleh dilakukan pada VBAC?
Menurut rekomendasi ACOG, induksi masih merupakan sebuah pilihan untuk
TOLAC namun memiliki peningkatan risiko rupture uteri. NAMUN, induksi juga
sebenarnya berhubungan dengan meningkatnya risiko kegagalan TOLAC.
a. Misoprostol tidak boleh digunakan untuk pemtangan serviks dan induksi
TOLAC
b. Oksitosin juga meningkatkan risiko ruptur uteri
7. Tanda-tanda ruptur uteri
a. Fetal bradycardia
b. Increased uterine contractions
c. Vaginal bleeding
d. Loss of fetal station
e. New onset intense uterine pain
8. Penilaian VBAC?
● Flamm-Geiger VBAC scoring

Distosia Bahu
1. Definisi & Diagnosis distosia bahu
Distosia bahu dapat dikenali apabila didapatkan adanya:
● Kepala bayi sudah lahir, tetapi bahy tertahan dan tidak dapat dilahirkan
● Kepala bayi sudah lahir, tetapi tetap menekan vulva dengan kencang
● Dagu tertarik dan menekan perineum
● Traksi pada kebapla tidak berhasil melahirkan bahu yang tetap tertahan di
kranial simfisis pubis
● Turtle sign: penarikan kembali kepala terhadap perineum sehingga tampak
masuk kembali ke dalam vagina.

2. Berapa lama bayi harus lahir sebelum terjadi hipoksia pada otak?
Setelah kepala lahir akan terjadi penurunan pH arteria umbilikalis. Dengan
demikian pada bayi yang sebelumnya tidak mengalami hipoksia tersedia waktu
antara 4-5 menit untuk melakukan manuver untuk melahirkna bahu.
3. Apa saja manuver yang dilakukan pada keadaan distosia bahu?
Manuver ALARMER
● Ask for help
● Lift/ hyperexflex woman’s leg (Mc Robert)
● Memposisikan ibu dalam posisi telentang, kedua paha fleksi, sehingga
lutut menjadi sedekat mungkin ke dada, rotasikan kedua kaki ke arah luar
(abduksi).
Lalu dilakukan tekanan suprapubik (Manuver Massanti)
● Anterior shoulder disimpaction (Suprapubic pressure – abdominal
approach)
● Rotation of posterior shoulder (Manuver Rubin dan Manuver Wood
Corkscrew – vaginal approach)
Manuver rubin: Masih dalam posisi McRobert, masukkan tangan pada
bagian posterior vagina, tekanlah daerah ketiak bayi sehingga bahu
berputar menjadi posisi oblik atau transversa
Manuver Wood Corkscrew: Masukkan tangan penolong yang
bersebrangan dengan punggung bayi.
Temukan bahu posterior, telusuri lengan atas dan buatlah sendi siku
menjadi fleksi. Peganglah lengan bawah dan buatlah gerakkan mengusap
kea rah dada bayi.
Langkah ini akan membuat bahu posterior lahir dan memberikan ruang
cukup bagi bahu asnterior masuk kebawah simfisis.
Dengan bantuan tekanan suprasimfisis kea rah posterior, lakukan tarikan
kepala ka arah posterokaudal dengan mantap untuk melahirkan bahu
anterior

● Manual removal of posterior arms


● Episiotomy
● Roll woman over onto “all fours”


CPD
1. Kapan diagnosis CPD ditegakkan?
Saat intrapartum atau percobaan persalinan, dimana kepala janin tidak kunjung
masuk PAP serta adanya dilatasi dan penipisan yang lambat, meskipun kontraksi
uterus baik (ACOG). Williams menggambarkan bagaimana saat ini CPD
diakibatkan malposisi kepala janin terhadap pelvis (asynclitism) atau karena
kontraksi uterus yang kurang efektif.
Dasar diagnosis CPD sendiri bisa dari tanda klinis seperti inspeksi abdomen,
Leopold, Osborn test, Muller-Munro Kerr test, USG, maupun pelvimetri
2. Kriteria diagnosis CPD?
● Dilatasi serviks ≥3 cm (ACOG) atau ≥4 cm (RTCOG) dan pendataran
serviks 100% (ACOG) atau 80% (RTCOG)
● Ditemukan penyebab-penyebab CPD (absolut dan relatif)
● Kontraksi uterus adekuat selama minimal 2 jam
● Kurva persalinan abnormal
● Pemanjangan Kala I dan kala II pada ibu yang sudah mendapat oksitosin
● Molase bertambah tanpa penurunan kepala bayi, pada leopold kepala
melayang di atas PAP atau di lateral salah satu fossa illiaca
● Tes Osborn/Obturator - pemeriksaan abdomen:
o Pasien tidur terlentang
o Kepala janin dipegang oleh tangan kiri pemeriksa
o Dua jari dari tangan kanan pemeriksa berada di atas simfisis,
permukaan jari berada di permukaan anterior pada simfisis
o Tekan kepala bayi ke bawah dan kebelakang
o Tentukan derajat tumpang tindih
Interpretasi:
o Kepala dapat ditekan ke dalam panggul dan tidak dapat tumpang
tindih dari parietal pada simfisis pubis ’ CPD (-)
o Kepala dapat ditekan sedikit, terdapat sedikit tumpang tindih dari
tulang parietal sekitar 0,5 cm → CPD sedang, lanjutkan Muller test.
o Kepala tidak dapat dimasukkan ke dalam tulang panggul, tulang
parietal menggantung di atas simfisis dengan dibatasi jari ’ CPD
(+) atau CPD berat
● Tes Muller-Munro Kerr - pemeriksaan abdominovaginal untuk menguji
apakah kepala janin dapat masuk ke PAP. Cara:
o Pasien tidur terlentang
o Satu tangan memegang kepala dari luar atas simfisis
o Dua jari dari tangan lain masuk ke vagina, sampai PAP
o Tangan luar mendorong ke arah panggul (bawah)
o jari telunjuk dan tengah tangan kanan di dalam vagina merasakan
penurunan kepala dan ibu jari di bagian luar simfisis pubis.
Interpretasi:
o Kepala bayi teraba oleh kedua jari, kepala bayi dapat didorong
setinggi spina ischial, tidak terdapat tumpang tindih ’ CPD (-)
o Kepala bayi teraba oleh jari, dapat didorong sedikit, tidak setinggi
spina ischial, terdapat sedikit tumpang tindih → CPD ringan sedang
o Kepala bayi tidak teraba oleh kedua jari, dan tidak dapat didorong→
CPD (+) atau CPD berat
● Xray pelvimetri: pelvis kecil
● Cephalometry dengan USG dan MRi
3. Klasifikasi Berdasarkan Penyebab CPD
a. Absolut CPD - true mechanical obstruction
i. kontraktur panggul
1. kontraktur PAP: konjugata vera <10cm, atau diameter
transversa <12 cm, atau konjugata diagonalis <11cm.
2. kontraktur midpelvis: lebih sering karena kapasitas midpelvis
< PAP disertai defleksi atau posisi abnormal kepala janin.
Kontrantur midpelvis suspek kalau:
a. jumlah antara distansia spinarum + diameter sagittal
posterior midpelvis <= 13cm (normalnya 15 cm).
b. distansia spinarum <10cm → suspek
c. distansia spinarum <8cm → jelas kecil
d. spina ischiadica menonjol hingga dinding pelvis
bertemu
3. kontraktur PBP: diameter intertuberosum <= 8cm, arkus pubis
<90o → yang bisa bikin kepala bayi bukannya turun ke
simfisis pubis bawah, tapi ke rami ischiopubic dan
menyebabkan distensi dan laserasi perineum
ii. pelvis sempit
iii. tumor di jalan lahir (anterior sacrococcygeal tumor)
iv. spondylolisthesis
v. pelvic exostoses
vi. Hidrosefalus
vii. Bayi besar, seperti postmatur atau bayi pada ibu dengan DM
b. Relative CPD
i. malpresentasi janin: presentasi alis, presentasi wajah
ii. malposisi janin: persistent occupitoposterior position, kepala
defleksi
4. Grading pada CPD
grading berdasarkan pemeriksaan pelvimetri (PAP):
- CPD berat: konjugata vera <7,5cm
- CPD borderline: konjugata vera 9,5-10cm, diameter anteroposterior
<10cm, diameter transversal <12cm
grading molase:
1. tulang terpisah, sutura teraba
2. sutura menutup, tidak overlap
3. tulang overlap, bisa dipisahkan
4. tulang overlap, tidak bisa dipisahkan
derajat CPD:
1. Kepala bayi di bagian bawah simfisis pubis
2. kepala bayi di bagian atas simfisis pubis
3. kepala bayi tidak menyentuh bagian atas simfisis pubis
5. Faktor Risiko CPD
Faktor ibu:
- nullipara, lebih jarang pada wanita multipara, terutama yang PSP normal.
CPD pada wanita dengan multipara dapat terjadi bila persalinan
sebelumnya diketahui ada CPD juga, atau kehamilan sekarang punya
faktor risiko tambahan seperti bayi besar, kepala besar, atau malposisi
- panggul sempit bisa dari tubuh yang kecil dan tinggi badan <145 cm
karena secara statistik memiliki panggul sempit
- riwayat fraktur pelvis → malalignment
- diabetes mellitus → meningkatkan risiko makrosomia pada janin
Faktor janin:
- Kepala besar
- Malposisi (asinklitismus, occiput posterior, presentasi muka, alis)
6. Sikap Terhadap CPD
- kontraktur minor: PSP, partus percobaan atau trial of labor. Risiko ruptur
uteri dan TOL gagal: insisi SC sebelumnya klasik/vertikal, riwayat SC >1,
SC sebelumnya <18cm, BMI>40, makrosomia >4kg
- kontraktur sedang-berat”
- induksi persalinan preterm
- SC saat aterm
- tidak direkomendasikan TOL
- CPD akibat kontraktur midpelvis atau PBP
- SC elektif
- PSP dengan vakum atau forsep pada kontraksi minimal
(kontroversial)
7. Tanda Inpartu pada CPD
- pemanjangan kala 1
- molase terus bertambah, tapi kepala tidak turun
- dilatasi serviks sangat pelan, atau tidak ada bahkan statis >2 jam
- bisa KPD karena kepala bayi yang tertahan di PAP akan memberikan gaya
tekan memecah selaput ketuban


PF (deskripsinya)
1. Cara pemeriksaan Leopold
Leopold I
● Pemeriksa menghadap ke bagian kepala ibu
● Letakan tangan pada fundus uteri (biasanya bokong bayi)
● Bokong (massa besar, nodular, empuk)/Kepala (keras bulat, ballotenement
positif)
● Pengukuran TFU
Leopold II
● Pemeriksa menghadap ke bagian kepala ibu
● Letakan tangan pada sisi kanan dan kiri perut ibu pada ketinggian yang sama
● Punggung (rata & memanjang)/ ekstremitas (bagian-bagian kecil)
Leopold III
● Pemeriksa menghadap ke kepala ibu
● Ibu jari tangan di supra simfisis
● Bagian terendah janin dicekap secara lembut dan rasakan bagian tersebut
Leopold IV
● Pemeriksa menghadap ke kaki ibu
● Ujung-ujung kedua jari tangan berada ditepi atas simfisis
● Temukan kedua ibu jari, kemudia rapatkan semua jari-jari tangan yang
meraba dinding bawah uterus
● Perhatikan sudut yang dibentuk oleh jari-jari (konvergen/divergen)
● Memeriksa sampai seberapa jauh bagian terendah janin masuk PAP (1-5/5)
2. Inspekulo apa yang dinilai
3. VT apa yang dinila
Hipertensi Dalam kehamilan
1. Jenis HT dalam kehamilan & definisinya
● Hipertensi kronik
HT yang timbul sebelum umur kehamilan 20 minggu (bisa juga sebelum
hamil) atau HT yang pertama kali didiagnosa pada kehamilan 20 minggu
dan menetap sampai 12 minggu pasca persalinan. Biasanya sudah ada
end organ damage juga karena sudah kronik
● Preeklamsia – eclampsia
PE adalah hipertensi yang timbul setelah 20 minggu kehamilan disertai
dengan proteinuria, menetap sampai 12 minggu pasca persalinan. PE =
pregnancy specific syndrome that can affect virtually every organ system.
Eklampsia adalah PE yang disertai dengan kejang dan/atau koma, tanpa
penyebab lain.
● Hipertensi kronik dengan superimposed preeklampsia
HT kronik disertai peningkatan tekanan darah (biasanya setelah 24 minggu)
+ new onset proteinuria
● Hipertensi gestasional
HT yang timbul pada kehamilan (bisa dibilang setelah 20 minggu) tanpa
disertai proteinuria dan menetap hingga 12 minggu pasca persalinan,
kehamilan dengan tanda-tanda PE tetapi tanpa proteinuria
2. Apa definisi proteinuria pada preeklampsia?
Proteinuria adalah adanya ditemukannya pada pemeriksaan penunjang 300 mg
protein dalam urin selama 24 jam atau sama dengan ≥ +1 dipstick
3. Bagaimana diagnosa Preeklampsia ditegakkan
4. Apa saja faktor risiko PE?
● Primigravida, primipaternitas, nullipara
● Hiperplasentosis; mola hidatidosa, multiple pregnancy, DM, hydrops
fetalis, bayi besar
● Umur yang ekstrim (terlalu muda, atau terlalu tua)
● Jarak dengan kehamilan sebelumnya 10 tahun tau lebih
● Riwayat kehamilan sebelumnya dengan PE (meningkat hingga 7x menurut
POGI atau 23-40% menurut Williams)
● Riwayat keluarga dengan PE (kalau ibunya PE, anaknya risiko PE
meningkat hingga 26%)
● Penyakit ginjal dan HT sebelum hamil
● Obesitas
5. Apa yang dimaksud dengan impending eclampsia
Impending eklampsia adalah preeklampsia dengan tanda berat berat yang
disertai gejala-gejala subjektif berupa nyeri kepala hebat, gangguan visus, nyeri
epigastrium dan kenaikan progresif tekanan darah.
6. Kriteria EDEN pada PE dengan gejala berat?
PE dengan gejala berat apabila terdapat satu atau lebih kriteria:
- koma >= 6 jam
- suhu >= 39oC
- nadi > 120x/menit
- TD sistolik > 200mmJg
- Anuria/oligouria
- RR>40x/menit
- konvulsi>10x
7. Patogenesis PE
a. kelainan vaskularisasi plasenta (remodelling a. spiralis terjadi secara
abnormal): invasi trofoblas ke dalam lapisan otot A. spiralis dan ke
jaringan matriks sekitar A. spiralis tidak terjadi → lapisan otot tetap kaku
dan keras → vasokonstriksi relatif A. spiralis (diameter 200 micron) →
penurunan aliran darah uteroplasenta → hipoksia dan iskemia plasenta
b. Iskemia plasenta → menghasilkan radikal bebas/oksidan yang merupakan
senyawa penerima elektron.
c. Hidroksil sebagai salah satu radikal bebas bersifat sangat toksik →
menyerang secara khusus membran sel endotel -> toxaemia.
d. Hidroksil → merusak membran sel yang banyak asam lemak tidak jenuh
→ terbentuk peroksida lemak → semakin merusak membran sel itu
sendiri, merusak nukleus dan merusak protein sel endotel → disfungsi
endotel
e. disfungsi endotel menyebabkan vasokonstriksi melalui:
i. gangguan metabolisme prostaglandin (vasodilator menurun
aktivitasnya) → pembuluh darah kehilangan daya refrakter
terhadap vasopressor sehingga menjadi sangat peka
ii. agregasi trombosit pada endotel rusak → trombosit kemudian
memproduksi tromboksan (vasokonstriksi meningkat aktivitasnya)
iii. perubahan khas pada sel endotel kapiler glomerulus
iv. peningkatan permeabilitas kapiler
v. peningkatan produksi endotelin (vasokonstriktor)
vi. penurunan produksi NO (vasodilator)
vii. peningkatan faktor koagulasi
f. Teori imunologi: HLA-G yang memiliki fungsi modulasi respon imun
sehingga ibu tidak menolak hasil konsepsi dan plasenta yang bersifat
asing serta melindungi trofoblas janin dari lisis oleh sel NK ibu
(mempermudah invasi sel trofoblas ke dalam jaringan desidua ibu) →
ekspresi HLA-G menurun sebagai maladaptasi imun → HLA-G juga
berkurang pada desidua daerah plasenta → menghambat invasi sel
trofoblas ke dalam jaringan desidua ibu → remodelling A. spiralis terjadi
abnormal
g. genetik: faktor keturunan dan familial dengan model gen tunggal. Jadi
risiko anak mengalami PE kalau ibu nya PE adalah 26%
h. diet defisiensi gizi: minyak ikan mengandung banyak asam lemak tak
jenuh → menghambat aktivasi trombosit dan produksi tromboksan →
menghambat vasokonstriksi
i. inflamasi: banyaknya sel trofoblas plasenta pada hamil ganda atau
plasenta besar → proses inflamasi berlebihan
8. Patofisiologi PE
a. disfungsi endotel (once again) menyebabkan:
i. gangguan metabolisme prostaglandin + agregasi trombosit dan
tromboksan meningkat + endotelin meningkat dan NO menurun →
vasokonstriksi
ii. Vasokonstriksi efeknya ke:
1. HT
2. nekrosis periporta → peningkatan enzim hepar dan
perdarahan pada sel periportal lobus perifer → edema →
kapsul teregang → nyeri epigastrik
3. peregangan selaput meninges → sakit kepala
4. kegagalan autoregulasi otak → vasospasme, iskemia dan
edema pada otak → kejang
5. spasme dan vasogenic edema pada a. retina atau menekan
N.II → gangguan penglihatan / pandangan kabur
6. Perdarahan otak di occipital → occipital blindness
(amaurosis)
7. Iskemi, infark, ablasio retina → purtscher retinopathy →
gangguan penglihatan
iii. kerusakan sel glomerulus → permeabilitas membran basalis
meningkat → kebocoran protein ke dalam urine → hipoalbuminemia
iv. Hipoalbuminemia + permeabilitas kapiler yang meningkat →
tekanan onkotik menurun → perpindahan cairan intravaskuler ke
interseluler → edema
v. endotel vaskular yang rusak juga bakal mengkonsumsi platelet
dalam jumlah banyak → low platelet count
vi. eritrosit didominasi yang bentuknya kurang baik → mudah hemolisis
→ LDH meningkat
9. Bagaimana membedakan nyeri kepala karena impending eklamsia / karena
HT / karena efek samping MgSO4?
Impending eklamsia lebih banyak di frontal (71%) dan berat banget (50%) serta
tidak membaik dengan obat
Kalau karena hipertensi → suborbital pressure + additional signs (sinus pressure,
aura, sensitivitas cahaya, denyut nadi ireguler)
Kalau karena MgSO4 → ada early sign (nausea, sumeng-sumeng hangat, flishing,
somnolen, double vision, slurred speech)
10. Penyakit ibu hamil dengan HT dan proteinuria selain PE apa?
HT + kelainan ginjal seperti sindrom nefrotik, nefritik atau gagal ginjal. Serum
kreatinin diketahui meningkat tanpa tanda proteinuria
11. Komplikasi PE
- Ibu: eklamsia, kejang, cerebral hemorrhage, DIC, trombositopeni, gagal
ginjal, kerusakan hepar, edema paru
- kehamilan: insufisiensi uteroplasenta, solusio plasenta, persalinan preterm,
SC
- janin: fetal distress, IUGR, oligohidramnios
12. Bagaimana cara pemberian MgSO4?



Loading dose/initial dose:
4 gram MgSO4 IV (40% dalam 10 cc) dalam 100mL RL/NaCl/D5 selama 15 menit
ATAU dalam 10mL RL selama 20 menit (1mL/menit = 20 tpm)
Maintenance dose:
1-2gram/jam → 24gram/24jam → 24 gram dalam 500cc RL → 524mL / 24 jam =
0,36mL/menit = 7 tpm (apasih ini)
ATAU
Infus 6 gram dalam RL/ 6 jam atau diberikan 4 atau 5 gram IM, maintenance dose
selanjutnya 4 gram IM tiap 4-6 jam
BILA KEJANG BERULANG
2 gram bolus IV perlahan dalam 15-20 menit → ambil 5mL MgSO4 dalam 10mL
RL → 15 mL dalam 20 menit → 0,75mL/menit → 15 tpm im I
=
20µm
BERULANG LAGI? m, xtpm
Diazepam 10mg IV selama 2 menit X = 151pm
13. Syarat pemberian MgSO4
● PEB atau E
● Harus tersedia antidotum Ca glukonas 10% diberikan IV 3 menit
● Refleks patella (+)
● Frekuensi pernapasan >16x menit, tidak ada tanda distress napas
● UO > 0,5mL/kgBB/jam
14. Kapan pemberian MgSO4 dihentikan?
● Ada tanda intoksikasi
o Hilangnya refleks tendon
o Henti napas
o Henti jantung
● Setelah 24 jam pasca persalinan atau 24 jam setelah kejang berakhir
15. Dosis terapeutik MgSO4
4-7 mEq/L atau 4,8-8,4 mg/dl
16. Cara pemberian Ca Glukonas
10% Ca glukonas 1 gram (10 cc), IV, selama 3 menit atau 15-20 menit
17. Cara kerja MgSO4
a. vasodilatasi otot polos pembuluh darah perifer dan uterus
b. menghambat reseptor NMDA di otak yang teraktivasi saat (asfiksia → Ca2+
masuk neuron → kejang), dimana magnesium akan menggeser Ca2+
sehingga aliran rangsangan tidak terjadi, transmisi neuromuscular pada
sinaps membutuhkan kalsium
c. antihipertensi
d. tokolitik
18. Obat antihipertensi dalam kehamilan
Untuk PE dengan tanda bahaya (jika TD >= 180/110) :
- Nicardipine 1 ampul 10 mg dalam 50 cc pelarut per jam (tidak boleh RL dan
Na bicarbonat) → 5mg/jam via slow infusion → titrasi 2,5mg/jam tiap 5
menit → maks 10-15mg/jam. dikurangi 3mg/jam setelah sampai target.
Target MAP turun 25%
Untuk PE
● Nifedipin dengan dosis awal 10-20 mg PO, diulangin 30 menit bila perlu.
Dosis maksimum 120 mg/24 jam. Tidak boleh diberikan sublingual karena
efek vasodilatasi sangat cepat. → ini sifatnya short acting, dan targetnya
MAP turun 20%
● Hydralazine loading dose 5 mg IV atau 10 mg IM. Kalau IV dilanjutkan 5-10
mg setiap 15-20 menit bila TD tidak turun. Kalau turun, boleh diulang 3 jam
berikutnya. Max dosenya memang 400mg/hari, tapi kalau sudah 20 mg IV
atau 30 mg IM tapi gak turun-turun → ganti obat aja om
● Labetalol: loading dose 10-20 mg IV → 2-80 mg IV setiap 20-30 menit maks
300mg/hari
Untuk HT kronis:
● Alfa-metildopa: 2-3x250-500mg PO (maks 2-3g/hari) → ini sifatnya long
acting (4-6 jam setelah obat masuk, menetap 10-12 jam) → disekresikan
dari ginjal dan ASI
● Nifedipin: 30-90 mg/hari
19. Cara Kerja metildopa dan nifedipin
Metildopa
Kalo kata Williams, acts by generally reducing sympathetic outflow to effect a
generalized decreased vascular tone.
Nifedipin
Menurunkan systemic vascular resistance dan efek vasodilatasi.
20. Target penurunan tekanan darah pada preeklamsia?
PEB: <160/110 (PNPK) atau 20-25%MAP atau sampai <140/90 mmHg
21. Kortikosteroid?
Dexamethasone IV 2x6mg IM dalam 2 hari → total 24 mg IM.
Betamethasone 1x12mg IM dalam 2 hari
22. Kerja kortikosteroid?
Pematangan paru, dengan cara: merangsang fibroblast paru → hasilin fibroblast
pneumocyte factor → stimulasi pneumosit tipe II untuk sintesis phosphatidylcoline
(dipalmitoyl phosphatidylcoline / lecithin: major component of surfactant)
23. Indikasi pemberian kortikosteroid
a. Pematangan paru pada PEB dibawah 34 minggu (manajemen ekspektatif
maupun aktif), PE dibawah 34 minggu (manajemen aktif), impending
eklamsia dan eklamsia (tapi tidak ada konsensusnya, yang penting tidak
boleh menunda terminasi kehamilannya)
b. HELLP Syndrome, dapat memperbaiki kadar trombosit ,SGOT, SPGT,
LDH, tekanan darah arteri rata-rata dan produksi urin. Dia meredam faktor
antiangioneik dan inflamasi yang banyak muncul di HELLP syndrome
24. LDH itu kenapa bisa meningkat kalau lagi PE atau HELLP Syndrome?
LDH adalah enzim intrasel, dia merubah asam piruvat menjadi asam laktat pada
proses glikolisis. Glikolisis adalah major energy pathway pada plasenta. Ketika
terjadi hipoksia akibat PE → glikolisis meningkat karena ini kan metabolisme
anaerob → LDH jelas meningkat. Jadi LDH meningkat melambangkan kerusakan
dan disfungsi seluler re
injury jaringan

25. Cara melihat lung maturity?


hitung Lecithine/Sphingomyelin Ratio (L/S Ratio) - gold standard.
Lecithin dan Sphingomyelin merupakan komponen surfactant. Sampai 32-33
minggu, kadar Lecithine dan Sphingomyelin kurang lebih sama, kemudian
Lecithine mulai naik sedangkan Sphingomyelin tetap.
Ketika didapatkan L/S Ratio <2 → high risk of RDS
Bagaimana cara mengambil sampel untuk cek L/S Ratio? → amniosentesis, cairan
amnion ditampung dan disentrifugasi 1000rpm 3-5 menit → dilakukan thin layer
chromatography pada supernatant → memisahkan komponen-komponen →
identifikasi Lecithin dan Sphingomyelinnya
Cara lain: shake test (Clement’s Test)
Menilai kemampuan Lecithine dan Sphingomyelin dalam mendukung
pembentukan ring bubbles di permukaan cairan amnion, stabil kah selama 15
menit? caranya: amniosentesis → ambil cairan amnion + NaCl + etanol → shake
15 detik → diamkan 15 menit dan lihat. Hasil positif bila terdapat complete ring of
bubbles di meniscus → paru sudah matang. Kelebihan: lebih mudah dan hasil on
the spot.
26. Terminasi kehamilan pada PE dengan gejala berat dan eklamsia?
PSP dengan induksi cukup adekuat dengan oksitosin.
5IU dilarutkan dalam 500mL RL → 10mIU/m = 10mIU/cc
bila ingin dosis awal 4mU/menit → maka 4/10 cc/menit → 0,4x20tpm = 8 tpm
tiap 15 menit naikkan 4mU (+ 8 tpm) hingga maksimal 20mU/mL atau 40 tpm
(buku merah) atau hingga maksimal 39mU/mL atau 72 tpm (williams)
27. Pemantauan apa yang dilakukan setelah SC pasien dengan impending
eklamsia?
PF: TTV, involusi uteri, perdarahan, observasi eklamsia (bisa late onset, hingga 24
jam post partum)
PP:
- darah lengkap
- fungsi hati
- LDH
- fungsi ginjal
- asam urat (meningkat di serum karena penurunan GFR dan peningkatan
reabsorpsi di tubulus proksimal → ekskresi asam urat berkurang)
28. Klasifikasi HELLP syndrome (Mississippi)

29. Klasifikasi HELLP syndrome (Tennessee)
a. Komplit: trombosit <100.000/uL, LDH >= 600 IU/L, SGOT >= 70 IU/L
b. Inkomplit: hanya terdapat 1 atau 2 tanda inkomplit

KPD
1. Kenapa KPD bikin solusio plasenta
KPD dapat menyebabkan korioamnionitis. Pada kondisi tersebut, terjadi
pelepasan lipopolisakarida dan endotoksin dari agen infeksi dan menginduksi
pembentukan dan penumpukkan sitokin dan bahan oksidan lainnya. Bahan ini
memiliki daya sitotoksik yang menyebabkan iskemia dan hipoksia yang berujung
pada kematian sel. Pada kasus ini kematian sel menyebabkan pemisahan vili-vili

korialis plasenta dari desidua basalis.


2. Kenapa flora normal yang berlebih bisa jadi FR KPD
3. Kapan terminasi KPD
4. Sumber cairan amnion
● Difusi dari pembuluh darah korion
● Urine janin (setelah 20 minggu)
● Rembesan kulit, selaput amnion dan plasenta
5. Apa hubungan koitus dengan KPD
Semen mengandung prostaglandin yang dapat menyebabkan pematangan
serviks/kontraksi dari uterus sehingga menyebabkan selaput ketuban menjadi
lebih mudah untuk pecah.
6. Mengapa diberikan dexamethasone? Bagaimana cara kerjanya?
Untuk pematangan paru
7. Bagaimana cara mengetahui paru-paru sudah matang?
● Lecithin-sphingomyelin (L/S) ratio > 2
(Gold standard)
● foam stability or shake test. Cara: Amniosemtesis tube + alkohol 70%
diamkan. Kalau terbentuk cincin ungu (air dan alkohol) pematangan paru +

● TDx-FLM= fluorescence polarization test, untuk mengukur rasio surfaktan
terhadap albumin (S/A ratio) 

● AmnioStat-FLM= untuk mengetahui adanya phosphatidylglycerol (PG) pada
surfaktan paru. Apabila ada aglutinasi antibodi pada PG di cairan amnion PG
ada! = matur
e.
● Lamellar Body Count= surfaktan dibungkus oleh granul yang disebut lamellar
bodies dan disekresi dari sel ke alveoli. Hal ini dapat ditentukan dengan
menghitung lamelar bodies pada amniosentesis. Kalau jumlahnya tinggi, fetal
lung mature

Kehamilan Ektopik
1. Definisi
Kehamilan ektopik: kehamilan yang pertumbuhan sel telur yang telah dibuahi
tidak menempel pada dinding endometrium kavum uteri. Implantasi bisa terjadi
dimanapun, padahal seharusnya setelah fertilisasi dan mobilisasi sel telur yang
telah dibuahi pada tuba fallopi, blastokista normalnya akan menempel pada
endometrial lining.
Kehamilan ektopik terganggu: kegawatdaruratan obstetri dimana implantasi hasil
konsepsi tidak terjadi pada dinding endometrium kavum uteri, dan tempat nidasi
tersebut tidak dapat menyesuaikan dengan pertumbuhan hasil konsepsi sehingga
terjadi ruptur.
2. Faktor Risiko
50% wanita dengan KE dan KET tidak memiliki faktor risiko.
Etiologi dan Faktor Risiko:
a. kondisi yang menyebabkan transportasi hasil nidasi ke kavitas uterus
terganggu, bisa karena gangguan anatomi atau gangguan fungsional.
b. Risiko meningkat 5x bila ada KE sebelumnya
c. Faktor tuba:
i. anatomi tuba falopi yang abnormal:
1. kongenital: misalkan divertikel saluran tuba, hipoplasia
uterus, saluran tuba berkelok-kelok
2. acquired:
a. infeksi: salpingitis kronis, infeksi menular seksual,
salpingitis isthmica nodosa, apendisitis
b. iatrogenik: pasca operasi rekanalisasi tuba, sterilisasi
tuba
c. tumor di sekitar tuba, mioma uteri, kista ovarium →
merubah patensi tuba
d. peritubal adhesion: karena endometriosis, infeksi,
apendisitis
e. PID
d. Faktor janin, janin terlalu cepat dan besar tumbuhnya.
e. Faktor ovarium: ketika hasil nidasi keluar, malah masuk ke tuba
kontralateral
f. Infertilitas
g. Iatrogenik:
i. penggunaan pil KB progesteron → menurunkan motilitas tuba falopi
ii. penggunaan IUD → infeksi
iii. IVF
h. Merokok → menurunkan motilitas tuba falopi
i. ras kulit hitam
j. usia >35 tahun
3. Gambaran klinis
ada yang khas di masing-masing klasifikasi tuba, tapi trias klasik:
- amenorrhea
- nyeri abdomen
- perdarahan pervaginam
bila ruptur:
- nyeri abdomen dan pelvis berat, tajam, tusuk-tusuk
- tenderness
- defans muskular
- nyeri goyang porsio → slinger sign/sliding pijn
- nyeri menjalar hingga ke leher/pundak bila mendesak subdiafragma &
nervus frenikus, terutama saat inspirasi → kehr sign
- forniks posterior akan menonjol, darah pada kavum douglasi / rectouterina
cul-de-sac. nyeri tekan juga
- vasomotor → vertigo, sinkop
- decidual cast bisa lepas dari endometrium → harus dicek, apakah ada hasil
konsepsi, gestational sac di decidual cast.
- silent shock, karena perdarahan bisa berkumpul di forniks posterior.
- uterus membesar sedikit saja
4. Klasifikasi KE
● Tuba (95%): dapat terjadi di ampulla (70%), isthmus (12%), fimbrae (11%),
interstisial dan kornual (2%)
o Biasanya KE tuba tidak berkembang lebih dari 8 minggu karena
vaskularisasi tidak sebaik di endometrium, dindingnya tipis,
lumennya sempit.
o hati-hati, ruptur sering terjadi ketika coitus atau VT
o Khusus untuk interstisial/kornual:
▪ trofoblas itu nempelnya di proksimal tuba di dalam dinding
muskular uterus. Sulit dibedakan dengan IUP yang eksentrik,
apalagi kalau ada anomali Mullerian, makanya IUP yang
eksentrik itu considered meliputi kehamilan kornual dan
kehamilan angular.
▪ USG bisa bantu tegakkan KE interstisial: uterus kosong atau
tidak, gestational sac terpisah dari endometrium >1 cm,
miometrium tipis <5mm yang meliputi gestational sac,
adanya interstitial line sign atau ekstensi endometrium ke
ujung gestational sac. Bisa juga USG3D.

▪ Untuk kehamilan eksentrik jelas kriterianya


o outcome pregnancynya bisa:
▪ ruptur tuba, biasa terjadi di ampulla, isthmus (ruptur di early
gestational age), interstisial/kornual (ruptur di late gestational
age).
▪ abortus tuba
▪ kematian mudigah dengan resolusi
o Tatalaksana
▪ Ekspektatif: kalau bhCG kurang <1.500 stabil dan cenderung
turun, tidak ada tanda-tanda ruptur termasuk gejala, massa
nya <3,5 cm, DJJ (-)
▪ MTX: hemodinamik stabil, dan sesuai indikasi:
● HARUS COMPLIANT
● KE tuba only
● Beta hCG <5.000 mIU/ml
● Belum ruptur
● Tidak ada DJJ
● massa <3,5 cm
● Methotrexate (MTX): antagonis asam folat bekerja
dengan cara mengikat dihidrofolat reduktase —> blok
reduksi dihidrofolat ke tetrahidrofolat (bentuk aktif
asam folat) —> sintesis purin dan pirimidin de novo
terhambat —> sintesis protein, DNA, RNA berhenti —
> stop tissue proliferation.
● Efek samping: bahaya ke sumsum tulang belakang,
mukosa GI, epitel pernafasan, ginjal dan hepatosit.
Toksisitas dapat diatasi dengan Lucovorin (asam
folat). Efek samping lainnnya ya nyeri.
● Dosis 50mg/m2 body surface area (single dose) atau
1 mg/kgBB (multidose, hari ke 1,3,5,7) + Leucovorin
asam folat. Dosis 0,1 mg/kgBB, hari ke 2,4,6,8
(multidose)

▪ Surgery: hemodinamik tidak stabil, tidak membaik dengan


MTX (beta hCG plateau atau meningkat), KE ruptur!, ada
peritoneal bleeding
▪ Laparotomi atau laparoskopi. Laparoskopi kalo pasiennya
secara hemodinamik stabil. Abis itu bisa dilanjutin either
salpingostomy (kalo massa belum rupture, ukuran < 2 cm,
posisi di 1/3 distal tuba) atau salpingectomy
▪ Salpingektomi indikasi:
● indikasi surgery
● kondisi tuba buruk, misalkan cicatrix, udah compang
camping
● bila menginginkan kehamilan, tuba kontralateral masih
sehat, atau merencanakan IVF
● cukup anak
▪ Salpingotomi/salpingostomi biasanya dilakukan kalau KE
infundibulum (fimbrae) atau ampulla.
▪ Partial salpingektomi + reanastomosis tuba biasanya
dilakukan pada KE isthmus.
▪ Wedge resection atau cornuectomy (<4cm) atau
cornuostomy (<3,5cm) bahkan histerektomi biasanya
dilakukan pada KE interstisial/kornual, kombinasi dengan
MTX sesuai dosis

● Abdominal (1%)
o Dapat terjadi secara primer akibat penanaman balstokista secara
direk di abdomen, atau secara sekunder akibat abortus tuba
(ekspulsi embrio dari ostium tuba pars abdominalis, atau terjadi
ruptur tuba, ruptur uteri, ruptur kornu) atau bahkan fertilisasi
intraabdominal dan IVF. Kemudian bisa menempel di mesovarium,
mesenterium, omentum, dll (intinya di kavitas peritoneum). Tidak
jarang ditemukan plasenta masih menempel pada uterus atau
adneksa, sementara janin berada di kavitas peritoneum
o Bisa diketahui early (<20 minggu usia gestasi), atau late (>20 minggu
usia gestasi)
o Tanda-tanda:
▪ posisi janin dapat dipalpasi, diketahui posisi abnormal
▪ serviks lokasinya tidak seperti biasanya
▪ tenderness pada abdomen
▪ USG sering miss hanya 50% akurasinya (agak lebih akurat
kalau di awal-awal kehamilan dilakukannya), misalkan:
● oligohidramnios (sering tapi tidak spesifik)
● Allibone et al:
o janin dalam gestasional sac terpisah dengan
uterus
o tampak pelvic atau abdominal mass, dengan
dugaan uterus terpisah dengan fetus
o tidak ada miometrium antara janin dengan
dinding abdomen anterior atau kandung kemih
o tampak janin sangat dekat dengan dinding
abdomen anterior
o lokalisasi plasenta diluar kavum uterus
● janin posisinya eksentrik pada pelvis
● bowel loops mengelilingi gestasional sac
● lebih akurat dengan TVUS
▪ Alfafetoprotein bisa meningkat.
▪ ketika dilakukan Medical Termination of Pregnancies (MTP),
janin tetap hidup dan aktif gerak
▪ bogginess pada kavum Douglas
o Terapi:
▪ tatalaksana tergantung usia gestasi
▪ terminasi segera setelah didiagnosis ATAU tunggu janin
viabel (24 minggu) dengan risiko perdarahan
▪ konservatif: berbahaya karena risiko perdarahan hebat dan
akut
▪ operatif: laparoskopi (kalau usia gestasi masih muda, dia bisa
berhentiin perdarahan dengan cauter bipolar, ultrasonic
surgical cutting dan coagulation device, hemostatic matrix
sealant agent, vasopressin analogue dll yang penting
mencapai hemostasis) dan laparotomi
● cari janin, lahirkan janin
● identifikasi plasenta, bila plasenta dapat diidentifikasi
dan yakin dapat dilahirkan dengan aman ATAU bila
sudah ada perdarahan yang berlangsung → lahirkan
plasenta. Ligasi pembuluh darahnya dahulu, bisa
dilakukan juga preoperative angiographic embolization
atau pemasangan kateter di dekat a. uterina untuk
mengurangi hilangnya darah dalam jumlah yang
banyak
● hindari eksplorasi yang tidak penting
● bila plasenta tidak dapat dilahirkan dengan aman, atau
kontraindikasi melahirkan plasenta lainnya,
direkomendasikan biarkan plasenta di dalam dengan
risiko infeksi, abses, adhesi, obstruksi ileus dan uretra
serta dehiscence. Pantau dengan USG dan b-hCG
(biasanya angkanay turun bulan hingga tahunan).
Targetnya adalah resorpsi plasenta, meski dapat
terjadi bertahun-tahun. Hal ini menghindari
pendarahan akut yang hebat, terlebih karena tidak ada
miometrium yang akan kontraksi menutup pembuluh
darah besar yang tervasodilatasi seperti pada
kehamilan normal.
● Berikan MTX (MASIH KONTROVERSIAL, karena bisa
mempercepat destruksi plasenta dengan jaringan
nekrosis dan infeksi serta pembentukan abses.
o Komplikasi:
▪ mortalitas maternal hingga 20%, biasanya karena
perdarahan hebat, perforasi, fistula, obstruksi GIT, DIC.
Apalagi kalau plasenta ditinggalkan didalam
▪ mortalitas fetal hingga 85-90%, yang selamat 20-90%
ditemukan janin yang malformasi dan dengan deformitas
karena kompresi (cairan amnion sedikit), dan gangguan
vaskuler
● Ovarium (3%)
o dapat ditegakkan dengan kriteria Spiegelberg (1878) dan ini
ditentukan intraoperasi
▪ tuba falopi ipsilateral intak
▪ gestational sac nya terletak di area ovarium
▪ kehamilan ektopiknya terhubung dengan uterus oleh lig.
ovarii proprium
▪ jaringan ovarium terdeteksi secara histologis pada dinding
gestational sac
o dibantu juga ditegakkan dengan TV-USG:
▪ uterus kosong
▪ kista dengan lingkaran echogenik yang lebar di ovarium atau
di sekitar ovarium
o Terapi: wedge resection / cystectomy atau bahkan ooforektomi
● Hysterectomy scar pregnancy:
o 1:2000 wanita BSC
o sulit dibedakan dengan IUP cervicoisthmic
o makanya dibantu dengan USG: uterus kosong, canalis cervicalis
kosong, massa intrauterine di depan isthmus uterus, miometrium di
antara kandung kemih dan gestational sac tidak ada atau tipis (1-
3mm)
o Terapi:
▪ MTX
▪ Histerektomi
▪ Bila masih ingin hamil: histeroskopi, visually guided suction
curretage, emboli a. uterine untuk mengurangi perdarahan)
● Heterotopic pregnancy: kehamilan intrauterin dan ekstrauterin
● Servikal:
o Gejala lebih sering perdarahan pervaginam yang banyak tanpa nyeri
o tidak ada sliding sign dan aliran darah sekitar gestational sac
dengan doppler
o uterus kosong
o barrel-shaped cervix
o ada gestational sac di bawah level sekitar os servikal internal
o Terapi: MTX, atau bila ekstensif bisa sampai pembedahan D&C atau
histerektomi
● Intraligamen

5. Goal dari Terapi


Selesaikan etiologi.
Jangan sampai ada trofoblas yang persisten.
Cek POD-1 b-hCG, bila penurunan <50% → masih ada trofoblas persisten.
Pilihannya operasi lagi atau MTX single-dose. Kecuali bila masih ada tanda-tanda
KET.

6. Pemeriksaan Penunjang KET


1st to Order

Trias KET (amenorrhea, nyeri abdominal, vaginal


Anamnesis bleeding/spotting)
Faktor risiko

tanda syok, anemia, nyeri tekan abdomen, defens


PF muskular, Kehr sign, nyeri goyang serviks, uterus
membesar tapi di bawah usia kehamilan

(+)
Alat testpack dengan ELISA itu biasanya bisa deteksi
Urine Pregnancy Test
serendah 20-25mIU/mL b-hCG di urine atau <=
5mIU/mL b-hCG di darah serum

- Pseudogestational sac
- Gestational sac di luar kavum uteri
- Free fluid
High-Resolution Transvaginal Ultrasound - Hemaperitoneium @kavum douglas
TVUS / TAUS (biasanya bila perdarahan >50cc)
- Decidual cyst
- Trilaminar pattern
- Doppler - ring of fire (kalau KET tuba)

Up to 30.000/uL

“Mean platelet volume seems to be higher in ectopic


pregnancy and this finding evokes a possible role of
increased platelet activity in the pathophysiology
Leukocytosis may occur more apparently in EP cases
Leukocyte with tubal rupture. “

“In patients with vaginal bleeding related


to ectopic pregnancy or miscarriage, an increased
neutrophil-to-lymphocyte ratio in the absence of
infection maybe used as an early diagnostic marker
for ectopic pregnancy.“ Normalnya NLR = 1-3

To Consider
● Above the discriminatory zone (3510
mIU/mL)

● Increase in level over 48h or plateau. Jadi


biasanya kalau meragukan (diatas
discriminatory zone, tapi gaada IUP ATAU
dibawah discriminatory zone), diulang 48
jam kemudian + TVUS. Penundaan ini harus
Serial Blood hCG
memperhatikan risiko ruptur.

● Karena TVS kedeteksi biasanya b-hCG


<1500, kalau TAS bisa deteksi, biasanya b-
HCG >6000
● kalau dia PUL, b-hCG juga meragukan →
D&C atau tunda 48 jam untuk cek lagi TVUS
dan b-hCG nya

● Indicator of viability of pregnancy, not


location tho

Serum Progesterone
● Failing pregnancy (non living IUP) or ectopic
pregnancy < 5 nanograms/mL,, yang
penting paham bahwa serum progesteron
>5 ng/mL itu 92% bukan KE.

(+): ditemukan darah kecoklatan sampai hitam yang


tidak membeku, atau yang berupa bekuan kecil-kecil.
Darah ini menunjukan adanya hematokel retrouterin.

(-): cairan jernih (dapat berasal dari cairan peritoneum


normal atau kista ovarium yang pecah), nanah (dapat
Culdocentesis (pastikan pasien sudah ½ duduk berasal dari penyakit radang pelvis atau radang
selama 15 menit) apendiks yang pecah, darah segar berwarna merah
yang dalam beberapa menit membeku, berasal dari
arteri atau vena yang tertusuk

Buat cek hemaperitoneum, ada studi yang


rekomendasiin pakai culdocentesis aja dibanding
TVUS
Hb dan Ht jarak 5 menit atau jarak 1 jam.
Melihat selisihnya untuk menentukan
kemungkinan occult blood loss.
Fonslani Test (Hb serial) Sejauh ini recent studies bilang fonslani test
itu tidak begitu membedakan yang mana yang
butuh intervensi, yang mana yang tidak butuh
intervensi.

7. Differential Diagnosis

Conditions Differentiating Sign/Symptom Differentiating Investigations


● Includes anembryonic
gestation, threatened ● Ultrasound: intrauterine pregnancy
abortion, incomplete
● Pelvic exam: dilation of cervix, as
abortion, complete
well as presence of tissue at
abortion, and missed
cervical os
Miscarriage abortion
● Vaginal bleeding in first
● Serum chorionic gonadotropin: do
not rise appropriately
trimester accompanied by
abdominal discomfort ● Progesterone: < 5ng/mL
secondary to uterine
contractions

● Sudden onset, severe,


unilateral lower abdominal
● Sonograph: ovarian enlargement
secondary to impaired venous and
pain that worsens
lymphatic drainage
Ovarian Torsion intermittently over many
hours ● Absence of arterial blood flow, but
● Peritoneal signs are often absent in early stages of torsion
absent

● Palpation: lower
abdominal, adnexal, ● Rare in pregnancy but can occur in
cervical motion tenderness the first 12 weeks of gestation
PID or Tubo-ovarian before decidua seals off the uterus
Abscess ● Oral temperature might be from ascending bacteria
> 38.3°C
● WBC: > 10 000 cells/mm3
● Abnormal cervical or
vaginal discharge

● Nonspecific nausea,
vomiting, low-grade fever,
pelvic pain, which is often
Ruptured Corpus
sharp, intermittent, sudden ● Doppler ultrasonography:
onset, and severe diagnostic
Luteal Cyst or Follicle
● At times of rupture: may
lead to profuse bleeding
and result in hemorrhagic
shock

Plasenta Previa
1. Komplikasi Plasenta previa
● Perdarahan anemia dan shock
● Plasenta aktreta spektrum (akreta, increta, perkreta)
● Perdarahan banyak saat dilakukan operasi krn serviks dan SBR rapuh dan
kaya pembuluh darah.
● Malpresentasi janin
2. Kapan bisa lahir PSP
● Plasenta tipe low-lying, perdarahan sedikit, presentasi kepala amniotomi
PSP
3. Beda solusio plasenta & Plasenta previa

4. Kapan harus diterminasi kehamilan?


● Usia cukup bulan
● Janin mati atau menderita anomali atau kondisi yang mengurangi
kelangsungan hidup (anensefali)
● Perdrahan aktif dan banyak, segera diterapi aktif tanpa memandang
kehamilan
PPH
1. Apa definisi PPH?
Perdarahan yang melebihi 500 ml (PSP)/1000 ml (SC) setelah bayi lahir. Secara
umum, jika terdapat perdarahan yang lebih dari normal apalagi telah menyebabkan
tanda vital, sesak napas, maka harus segera dilakukan penanganan

Hyperemesis Gravidarum
1. Bagaimana batas normal nausea dan vomiting pada ibu hamil?
2. Definisi HEG
HEG adalah bentuk nausea dan vomiting terberat pada ibu hamil yang dikaitkan
dengan muntah persisten, hipovolemia, ketosis, gangguan elekrolit, dan
penurunan berat badan (biasanya >5% BB)
3. Etiologi HEG
Bila betul HEG< seharusnya dia unknown dan mulifaktorial, oleh karena itu harus
dicari dahulu penyebab nausea and vomiting lainnya sebelum menegakkan HEG.
Ada yang bilang HEG adalah “diagnosis of exclusion”
- Berhubungan dengan peningkatan hormon yang tinggi dan dengan cepat.
Pelakunya diduga hCG, estrogen, progesteron, leptin dan placental growth
hormone, prolaktin, tiroksin, dan hormon adenokortikal. Sumber lain ada
juga yang bilang ghrelin, leptin, nesfatin-1, peptide YY (3-36)
- hCG tinggi berhubungan dengan wanita yang memiliki estrogen
tinggi
- progesteron diketahui berhubungan dengan penurunan motilitas
GIT
- faktor biologis dan lingkungan. Salah satu penjelasannya adalah HEG
sebagai respon protektif ibu dan janin dari substansi yang berpotensi
membahayakan
- faktor psikologis
4. Faktor Risiko HEG
a. hipertiroid
b. DM
c. asma
d. infeksi H. pylori → masih inkonklusif
e. Kebiasaan merokok kronis → cannabinoid hyperemesis syndrome
f. komplikasi obstetri:
i. mola hidatidosa
ii. persalinan preterm
iii. solusio plasenta
iv. preeklamsia
5. Komplikasi HEG

selain tabel diatas:


- defisiensi vitamin K → koagulopati maternal dan perdarahan intrakranial
janin, embriopati vitamin K
6. Manajemen HEG
a. Antiemetik untuk mild dan moderate yang aman, tidak teratogenik:
- Doxylamine
- Pyridoxine
- Ondansetron, dia bahkan lebih efektif dibandingkan Doxylamine
dan Pyridoxine, tapi jarang digunakan karena ES prolonged-QT
dan serotonin syndrome
b. Untuk severe dan tidak terkontrol:
i. Resusitasi cairan dengan kristaloid 1L + thiamine 100mg untuk
mencegah Wernicke Encephalopathy
ii. Antiemetik seperti metoclopramide, promethazine.

*teratogenik:
- glukokortikosteroid
-

Anemia
1. Berapa cutoff untuk anemia pada masing-masing trimester?
Trimester 1&3 – 11.0 g/dL
Trimester 2 – 10.5 g/dL
2. Mengapa pada trimester 2 cutoff Hb lebih rendah?
Pada trimester 2 terdapat ekspansi volume plasma dibandingkan dengan
peningkatan sel darah merah sehingga terjadi hemodilusi.
Volume plasma akan meningkat kira-kira 40-45%, sementara jumlah sel darah
merah akan meningkat sebanyak 20-30%
3. Indikasi transfusi pada anemia dalam kehamilan
Transfusi dengan sel darah merah tetap dilakukan Ketika tingkat Hb adalah 7-10 g/dL
pada kondisi:
● Terjadi perdarahan terus-menerus
● Terdapat tanda-tanda penurunan daya-angkut oksigen (PPOK, penyakit
kardiovaskular) selama pembedahan.
● Menurunnya eritropoesis
● Ketika transfusi autologous akan digunakan. Setiap unit sel darah merah
yang di transfusi akan meningkatkan Hb +/- 1 g/dL (Ht 1-3%) pada seorang
perempuan dengan berat badan 70 kg.
4. Berapa kebutuhan besi, asam folat dan vitamin B12 dalam kehamilan?
● Fe – 6-7mg/hari
● B12 2,2 mcg/hari
● Folat 400 mcg/hari
5. Berapa dosis besi yang diberikan pada kehamilan?
Tablet tambah darah: 60 mg besi elemental + 400 microgram asam folat
Diberikan segera setelah mual/muntah berkurang selama kehamilan.
1. Kenapa ketuban bisa berwarna hijau
Placental insufficiency, maternal hypertension, preeclampsia, oligohydramnios,
infection, acidosis, and maternal drug abuse especially use of tobacco and
cocaine ’ fetal hypoxic stress → stimulasi neural kepada GIT janin
’ meconium passage (ketuban berwarna hijau) ’ reducing antibacterial
activity, increasing risk of fetal bacterial infection, irritate skin, and meconium
aspiration syndrome (MAS). MAS ’ airway obstruction, surfactant
dysfunction, chemical pneumonitis, pulmonary hypertension → hypoxia ’ RDS.

2. Cairan ketuban
98% terdiri dari air dan bertambah terus sesuai dengan usia kehamilan sampai
setelah 40 minggu baru akan berkurang 8% setiap minggunya. Cairan ketuban
diukur dengan indeks cairan amnion yang normalnya 5cm – 24/25cm (4 kuadran)
atau 2-8cm (single deepest pocket)

3. Oligohydramnios
Kriteria diagnosis: indeks cairan amnion < 5cm (4 kuadran) atau single deepest
pocket dari cairan amnion < 2cm.
Etiologi:
- Early onset: ketika dideteksi cairan ketuban berkurang pada
trimester kedua awal, dapat disebabkan gangguan pada fetal (gangguan
sekresi urine), gangguan pada insufisiensi plasenta (gangguan perfusi)
- After mid pregnancy: ketika dideteksi cairan ketuban berkurang
pada trimester kedua akhir atau trimester ketiga, dapat disebabkan
abnormalitas plasenta, faktor maternal seperti preeklamsia atau
penyakit vaskuler yang berujung pada IUGR dan gangguan sekresi urine
- Gangguan kongenital: seringkali gangguan kongenital yang
menyebabkan gangguan pada ginjal dan traktus urinarius menyebabkan
gangguan sekresi urine
- Medikasi:
● ACE inhibitors → ganggu RAAS → hipotensi fetal →
hipoperfusi renal ’ iskemia renal ’ gangguan sekresi urine
● NSAIDs ’ konstriksi fetal ductus arteriosus → gangguan
sekresi urine
Ketika indeks cairan amnion 5 – 8 cm, dapat dikategorikan “borderline”
oligohydramnios tapi ada studi yang menyatakan kalau borderline itu tidak lebih
berbahaya dibandingkan jumlah cairan ketuban normal (>8 cm) dalam hal
komplikasi stillbirth, kematian neonatus ataupun hipertensi maternal. Studi lain
ada yang melaporkan borderline ini tinggi angka IUGR nya.

Terapi: observasi ibu dan janin, hidrasi ibu



Stillbirth, Kematian Mudigah, Abortus
1. Definitions
Stillbirth = IUFD = kematian janin intrauterine sebelum ekspulsi atau ekstraksi
produk konsepsi dari rahim ibu setelah 20 minggu usia kehamilan atau berat
badan janin 350/500 gram.
Kematian Mudigah = abortus = kematian janin intrauterine sebelum ekspulsi atau
ekstraksi produk konsepsi dari rahim ibu sebelum 20 minggu usia kehamilan atau
berat badan janin < 350/500 gram.
Macam-macam abortus:
- Abortus dibagi menjadi abortus spontan dan provokatus. Abortus
provokatus dibagi menjadi abortus medisinalis dan abortus kriminalis
- Abortus dapat juga dibagi menjadi:
- Abortus iminens:
- Abortus insipiens
- Abortus kompletus
- Abortus inkompletus
- Abortus tertunda (missed abortion)
- Abortus habitualis
- Abortus infeksiosus / septik
2. Sdsds


Preeklamsia
1. Antiphospholipid Antibody (APLA) Syndrome / Hughes Syndrome
Diketahui berhubungan dengan KET. APS-related complication pada wanita hamil
tidak hanya KET, tapi juga abortus rekuren, preterm, oligohidramnios, IUGR,
trombosis fetal/neonatal, preeklamsia, eklamsia, HELLP, insufisiensi lasenta.
Antibodi antifosfolipid itu banyak, tapi intinya antibodi yang menyerang fosfolipid
dan membuat tubuh lebih mudah membentuk trombus (prothrombotic) sehingga
terbentuk blood clots di arteri dan vena. Mekanismenya bermacam-macam, bisa
meningkatkan ekspresi molekul adhesi, tissue factor, monosit, ekspresi dan
sintesis thromboxane A2, dan akhirnya menyebabkan procoagulant state.
Antibodinya ada Lupus Anticoagulants (LA), Anticardiolipin Antibodies (aCL), Anti-
2-Glycoprotein I antibodies (APL subset). Sebenarnya antibodi ini paling banyak
di orang dengan SLE, gangguan jaringan ikat, dan sebagian kecil orang normal
tanpa fenotip juga bisa tapi gak bergejala. Stimulusnya yang diketahui adalah
infeksi. Bisa sampai katastrofik (Catastrophic Antiphospholipid Antibody
Syndrome)
Kalau sudah ada resiko pembekuan darah begini, biasanya terjadi:
1. aktivasi prokoagulan
2. inaktivasi antikoagulan natural
3. aktivasi komplemen
4. inhibisi diferensiasi sinsitiotrofoblas
pada akhirnya menyebabkan vaskulopati desidua, infark plasenta, IUGR,
preeklamsia onset dini dan recurrent IUFD
Terapi:
1. Kalau ada riwayat tromboemboli → heparin profilaksis selama kehamilan
dan heparin atau warfarin hingga 6 minggu postpartum
2. Kalau tidak ada riwayat tromboemboli → observasi ketat dengan/tanpa
heparin dosis intermediate. ada juga penelitian yang bilang pakai
hidroksiklorokuin. Sebenarnya pemberian heparin itu kontroversial, tapi
kalau titer ACA atau aktivitaas LAC nya tinggi, ya lebih baik diberikan
apalagi kalau ada riwayat pregnancy loss. ACOG rekomendasiin juga kasih
Aspirin 60-80mg/hari karena minimal efek samping kombinasi dengan
Hidroksiklorokuin.
3. Kalau sudah ada CAPS, terapi agresif dengan antikoagulan, kortikosteroid,
plasma, IVIG, rituximab.

Kegawatdaruratan Obstetri
1. Terdiri atas:
a. Hipertensi dalam kehamilan
b. infeksi
c. perdarahan
d. lain-lain
2. Perdrarahan :
a. antepartum early:
i. abortus
1. iminens
2. insipiens
3. inkomplit
4. komplit
5. missed
6. habitualis
ii. KET
1. ampulla
2. fimbrae
3. interstisial
4. ismika
5. serviks uteri
6. ovarium
7. cornu uteri
8. rongga obdamen
iii. Mola
b. Antepartum late:
i. vasa previa
ii. placenta previa
iii. solusio placenta
c. Intrapartum:
i. ruptur uteri
ii. robekan portio
iii. varises vagina pecah
iv. robekan jalan lahir
v. solusio plasenta
vi. trauma
d. postpartum:
i. tonus
ii. trauma
iii. tissue
iv. thrombin

Kontrasepsi
KB Ideal:
intinya tiap wanita punya kepentingan terhadap hidupnya
1 menunda kehamilan
2 menjarakkan kehamilan
3 sterilisasi

pil KB kombinasi, cincin vagina, dan koyo KB, penggunaan KB ini dari 21 hari pasca
melahirkan. Namun, hal ini bisa dilakukan hanya bila Anda sedang tidak menyusui bayi.
Sementara itu, apabila ternyata Anda menyusui bayi, sebaiknya Anda menunggu waktu
lebih lama untuk mulai menggunakan salah satu dari tiga alat KB tersebut setelah
melahirkan. Ini karena ketiga alat KB tersebut mengandung hormon sintetis, seperti
estrogen. Permasalahannya yakni hormon sintetis ini dapat memengaruhi produksi ASI
di dalam tubuh Anda selama masa menyusui. Jika Anda ingin menggunakan KB suntik,
pil KB mini, diafragma, atau kap serviks, Anda bisa menggunakan salah satu alat
kontrasepsi tersebut sekitar 6 minggu setelah melahirkan. Kerja dari pil KB kombinasi
memang dapat menghambat produksi ASI, tapi melansir dari Mayo Clinic, pil KB mini
justru tidak masalah diminum ibu menyusui. Keempat alat KB tersebut termasuk aman
dan tidak memiliki pengaruh terhadap produksi ASI Anda. Jika Anda ingin menggunakan
alat KB IUD (KB spiral), pemasangan KB ini bisa dilakukan segera setelah melahirkan.
Pemasangan KB IUD setelah melahirkan ini dilakukan agar Anda tidak perlu merasa nyeri
berulang kali. Cara ini juga termasuk salah satu cara yang praktis. Di samping itu, IUD
tidak akan mengganggu produksi ASI.

Anda mungkin juga menyukai